ROMANIAN MATHEMATICAL MAGAZINE R.M.M. · pentru ∈ z∗, pe care am rezolvat-o așa cum urmează,...

78
S.S.M.ROMÂNIA - Filiala Mehedinți 2017 1 ROMANIAN MATHEMATICAL MAGAZINE NR. 18 SOCIETATEA DE ȘTIINȚE MATEMATICE DIN ROMÂNIA Filiala Mehedinți ROMANIAN MATHEMATICAL MAGAZINE R.M.M. Nr.18-2017

Transcript of ROMANIAN MATHEMATICAL MAGAZINE R.M.M. · pentru ∈ z∗, pe care am rezolvat-o așa cum urmează,...

Page 1: ROMANIAN MATHEMATICAL MAGAZINE R.M.M. · pentru ∈ z∗, pe care am rezolvat-o așa cum urmează, am păstrat algoritmul și pentru calculul altor sume, unele putând fi calculate

S.S.M.ROMÂNIA - Filiala Mehedinți 2017

1 ROMANIAN MATHEMATICAL MAGAZINE NR. 18

SOCIETATEA DE ȘTIINȚE MATEMATICE DIN ROMÂNIA

Filiala Mehedinți

ROMANIAN MATHEMATICAL MAGAZINE

R.M.M.

Nr.18-2017

Page 2: ROMANIAN MATHEMATICAL MAGAZINE R.M.M. · pentru ∈ z∗, pe care am rezolvat-o așa cum urmează, am păstrat algoritmul și pentru calculul altor sume, unele putând fi calculate

S.S.M.ROMÂNIA - Filiala Mehedinți 2017

2 ROMANIAN MATHEMATICAL MAGAZINE NR. 18

SOCIETATEA DE ȘTIINȚE MATEMATICE DIN ROMÂNIA

Filiala Mehedinți

COLECTIVUL DE REDACȚIE ROMANIAN MATHEMATICAL MAGAZINE

1 DANIEL SITARU Redactor șef ; Machetare grafică ; Copertă 2 GHEORGHE CĂINICEANU Redactor principal 3 DAN NĂNUȚI Redactor 4 EMILIA RĂDUCAN Redactor 5 MARIA UNGUREANU Redactor 6 DANA PAPONIU Redactor 7 LEONARD GIUGIUC Redactor 8 GIMOIU IULIANA Redactor clasa a V-a 9 ELENA RÎMNICEANU Redactor clasa a VI-a

10 DRAGA TĂTUCU MARIANA Redactor clasa a VII-a 11 DANIEL STRETCU Redactor clasa a VIII-a 12 CLAUDIA NĂNUȚI Redactor clasa a IX-a 13 DAN NEDEIANU Redactor clasa a X-a 14 GABRIELA BONDOC Redactor clasa a XI-a 15 OVIDIU TICUȘI Redactor clasa a XII-a

Page 3: ROMANIAN MATHEMATICAL MAGAZINE R.M.M. · pentru ∈ z∗, pe care am rezolvat-o așa cum urmează, am păstrat algoritmul și pentru calculul altor sume, unele putând fi calculate

S.S.M.ROMÂNIA - Filiala Mehedinți 2017

3 ROMANIAN MATHEMATICAL MAGAZINE NR. 18

CUPRINS

Aflarea a două numere naturale cărora le cunoaștem suma puterilor asemenea și cel mai mic multiplu comun - Ștefan Marica..........................................................................................................4

O metodă de calcul a sumelor de „n” termeni prin deplasarea termenilor în linie – Carmen – Victorița Chirfot.....................................................................................................................6

Generalizări ale unor ecuații diofantice- Angelica Boitan, Nicolae Papacu.............................10

Power sums – Daniel Mackleen……………………………………........................................................13

Hlawka’s inequalities for a class of functions – Daniel Sitaru, Leonard Giugiuc……………..….15

Generator probleme de gimnaziu pentru concursuri – Laviniu Bejenaru.............................18

În legătură cu o problemă dată în Azerbaijan 2016, TST, BMO – Marin Chirciu...................20

Problemă comentată 27272, G.M. 9/2016 -Ioan Șerdean....................................................22

GPF Inequality – Nguyen Anh Duc.....................................................................................................................24

Factorizarea unor determinanți – Daniel Sitaru ..................................................................................31

On some triangle inequalities – Nguyen Viet Hung .......................................................................34

Problemă comentată GMB 5/2016 –Daniel Sitaru ....................................................................39 Refinements of the inequality between the AM-GM means –Mihaly Bencze….………...……..40

Solved problem-Leonard Giugiuc…………………………………………………………………………………….…41

Probleme propuse………………………………………..…………………………………………………………………...42

Autorii materialelor din RMM-18………………………………………………………………………………………………79

Page 4: ROMANIAN MATHEMATICAL MAGAZINE R.M.M. · pentru ∈ z∗, pe care am rezolvat-o așa cum urmează, am păstrat algoritmul și pentru calculul altor sume, unele putând fi calculate

S.S.M.ROMÂNIA - Filiala Mehedinți 2017

4 ROMANIAN MATHEMATICAL MAGAZINE NR. 18

AFLAREA A DOUĂ NUMERE NATURALE CĂRORA LE CUNOAȘTEM SUMA PUTERILOR ASEMENEA ȘI CEL MAI

MIC MULTIPLU COMUN By Ștefan Marica-Romania

Dacă notăm 푺풏 = 풂풏 + 풃풏 unde 풏 ∈ ℕ∗ și 푴 = [풂;풃] pentru determinarea numerelor naturale 풂 și 풃 vom proceda astfel: 푺풏 = 풂풏 + 풃풏 = 푫풏 ⋅ 풂ퟏ풏 + 푫풏 ⋅ 풃ퟏ풏 = 푫풏 ⋅ (풂ퟏ풏 + 풃ퟏ풏), unde 푫 este cel mai mare divizor comun 푫 = (풂;풃), iar (풂ퟏ;풃ퟏ) = ퟏ (sunt numere prime între ele). În aceste

condiții 푴 = 푫 ⋅ 풂ퟏ ⋅ 풃ퟏ, iar 풃ퟏ = 푴푫⋅풂ퟏ

. 푺풏 = 푫풏 풂ퟏ풏 + 푴푫풏⋅풂ퟏ

풏 =

= 푫풏 ⋅ 풂ퟏ풏 + 푴풏

풂ퟏ풏 sau 푺풏 ⋅ 풂ퟏ풏 = 풂ퟏퟐ풏 ⋅ 푫풏 + 푴풏 ecuație în necunoscuta 풂ퟏ풏

푫풏 ⋅ 풂ퟏퟐ풏 − 푺풏 ⋅ 풂ퟏ풏 +푴풏 = ퟎ (1) ecuație de gradul doi în necunoscuta 풂ퟏ풏.

휟 = 푺풏ퟐ − ퟒ ⋅ 푫풏 ⋅ 푴풏, iar soluția ecuație (1) este: 풂ퟏ풏 = 푺±√휟ퟐ⋅푫풏

, 풃ퟏ풏 = 푴풏

푫⋅풂ퟏ풏

Numerele naturale căutate sunt: 풂 = 푫 ⋅ 풂ퟏ, 풃 = 푫 ⋅ 풃ퟏ

În continuare vom aplica cele stabilite anterior pentru 풏 = ퟏ,풏 = ퟐ și 풏 = ퟑ.

1. Să se afle 풂 și 풃 știind că 푺ퟏ = 풂 + 풃 = ퟏퟔ și 푴 = [풂;풃] = ퟑퟎ. Dacă descompunem în factori primi avem 푆 = 16 = 2 și 푀 = 3 ⋅ 2 ⋅ 5. Cel mai mare divizor comun 퐷 = 2

푆 = (푎 + 푏 ) ⋅ 2 = 2 ⋅ 푎 +⋅

sau 2푎 − 푆푎 + 30 = 0.

훥 = 푆 − 4 ⋅ 퐷 ⋅ 푀 = 256− 4 ⋅ 2 ⋅ 30 = 16 = 4

Soluțiea ecuației este: 푎 =

푏 =⋅

⇒푎 = = 5

푏 = = 3. Numerele 푎 și 푏 sunt:

푎 = 퐷 ⋅ 푎 = 2 ⋅ 5 = 10푏 = 퐷 ⋅ 푏 = 2 ⋅ 3 = 6 , 푎 = 10

푏 = 6

2. Să se afle 풂 și 풃 știind: 푺ퟐ = 풂ퟐ + 풃ퟐ = ퟓퟒퟒ și 푴 = [풂;풃] = ퟔퟎ.

Cel mai mare divizor comun 퐷 = (544; 60) = (2 ⋅ 17; 2 ⋅ 3 ⋅ 5) = 4

푆 = (4푎 ) + (4푏 ) = 16(푎 + 푏 ) = 16푎 +60푎

16푎 − 544푎 + 60 = 0 (ecuație de gradul doi în raport cu 푎 )

훥 = 544 − 4 ⋅ 16 ⋅ 60 = 2 ⋅ 17 − 2 ⋅ 3 ⋅ 5 = 2 ⋅ (289− 225) = 2 de aici √훥 = 2 =256 ; 푎 =

⋅= = 25 = 5 ; 푏 =

⋅= ⋅ ⋅

⋅= 9 = 3

numerele cerute sunt: 푎 = 퐷 ⋅ 푎푏 = 퐷 ⋅ 푏 ⇒ 푎 = 20

푏 = 12

Page 5: ROMANIAN MATHEMATICAL MAGAZINE R.M.M. · pentru ∈ z∗, pe care am rezolvat-o așa cum urmează, am păstrat algoritmul și pentru calculul altor sume, unele putând fi calculate

S.S.M.ROMÂNIA - Filiala Mehedinți 2017

5 ROMANIAN MATHEMATICAL MAGAZINE NR. 18

3. Să se afle 풂 și 풃 știind că: 푺ퟑ = 풂ퟑ + 풃ퟑ = ퟑퟓ și 푴 = [풂;풃] = ퟔ

Cel mai mare divizor comun 퐷 = (35; 60) = 1. În acest caz 푎 = 푎 și 푏 = 푏 .

푆 = 푎 + 푏 = 푎 + , 35 = 푎 + ecuația în 푎 este: (푎 ) − 35푎 + 216 = 0 (ecuație de

gradul doi în 푎 ) , 훥 = 35 − 4 ⋅ 1 ⋅ 216 = 1225 − 864 = 361 = 19

Soluția este: 푎 = = = 27 = 3

푏 = = = 8 = 2⇒ 푎 = 3

푏 = 2.

Observație: La acest exemplu am urmărit însușirea metodei de lucru. Cele două numere se observă direct: 풂ퟑ + 풃ퟑ = ퟑퟑ + ퟐퟑ = ퟑퟓ. În continuare ne propunem să formulăm următoarea problemă:

Să se afle numerele naturale 풂 și 풃 cărora le cunoaștem suma 푺 și cel mai mare divizior comun 푫. În cazul 푛 = 1 avem 푆 = 푎 + 푏 și 퐷 = (푎; 푏)

푆 = 푎 + 푏 = 퐷 ⋅ 푎 +퐷 ⋅ 푏 = 퐷(푎 + 푏 ) unde 퐷 este cel mai mic dintre 퐷 și 퐷

푀, cel mai mic multiplu comun va fi: 푀 = [푎; 푏] = 퐷 ⋅ 푎 ⋅ 푏ʹ iar 푏ʹ =⋅

푆 = 퐷 ⋅ 푎 + 퐷 ⋅ 푏ʹ iar 푏ʹ =⋅

; 푆 = 퐷 ⋅ 푎 + 퐷 ⋅ 푏ʹ = 퐷 ⋅ 푎 +

퐷 ⋅ 푎 − 푆 ⋅ 푎 + 푀 = 0 (ecuație de gradul doi în 푎 ) ; 훥 = 푆 − 4 ⋅ 퐷 ⋅ 푀 și numerele 푎 și 푏 se află

astfel: 푎 = √

푏ʹ =⋅

iar 푎 = 퐷 ⋅ 푎푏 = 퐷 ⋅ 푏ʹ

Aplicație: Să se afle aria și perimetrul unui triunghi dreptunghic știind că

푺 = 풂 + 풃 = ퟑퟔ și 푫 = (풂;풃) = ퟐ, unde 풂 și 풃 reprezintă lungimea ipotenuzei și a unei catete. 푆 = 푎 + 푏 = 2 ⋅ 푎 + sau 2 ⋅ 푎 − 36 ⋅ 푎 +푀 = 0 (ecuație de gradul doi)

푎 − 18푎 +푀 unde 푀 = 2 ⋅ 푀 , 훥 = 81−푀 unde 푀 se află ușor. 푀 = 65

훥 = 81− 65 = 16 = 4 ; 2 ⋅ 푎 − 36푎 + 130 = 0 ;

푎 = 9 + 4 = 13푏ʹ = 9− 4 = 5 ⇒ 푎 = 13 ⋅ 2 = 26

푏 = 5 ⋅ 2 = 10 ; Cateta 푐 este 푐 = 24. Triunghiul dreptunghic are:

푃 = 10 + 24 + 26 = 60 (cm) ; 푆 = ⋅ = 120 (cm2) ; Aplicații:

1. Să se afle numerele naturale 풂 și 풃, care au: 푺 = ퟑퟖ și 푫 = ퟐ

2. Să se afle numerele naturale 풂 și 풃, cărora le cunoaștem 푺 = ퟑퟖ și 푴 = ퟏퟖퟎ

3. Lungimea diagonalei unui dreptunghi este de ퟒퟎ cm, iar cel mai mic multiplu comun al dimensiunilor dreptunghilor este ퟗퟔ cm. Să se afle aria și perimetrul dreptunghiului.

Bibliografie: Gazeta de Matematică și Fizică nr. 10 din 1957.

Page 6: ROMANIAN MATHEMATICAL MAGAZINE R.M.M. · pentru ∈ z∗, pe care am rezolvat-o așa cum urmează, am păstrat algoritmul și pentru calculul altor sume, unele putând fi calculate

S.S.M.ROMÂNIA - Filiala Mehedinți 2017

6 ROMANIAN MATHEMATICAL MAGAZINE NR. 18

O METODĂ DE CALCUL A SUMELOR DE N TERMENI PRIN DEPLASAREA TERMENILOR ÎN LINIE

By Carmen – Victorița Chirfot-Romania

Plecând de la o problemă elementară, și anume, calcularea sumei ퟏ + ퟐ + ퟐퟐ + ퟐퟑ +⋯+ ퟐ풏 pentru 풏 ∈ 푵∗, pe care am rezolvat-o așa cum urmează, am păstrat algoritmul și pentru calculul altor sume, unele putând fi calculate prin același algoritm de deplasare a primului termen al sumei cu o poziție, păstrând ordinea celorlalți termeni. În cele ce urmează propun un set de probleme rezolvate prin această metodă.

1. Să se calculeze suma ퟏ + ퟐ + ퟐퟐ +⋯+ ퟐ풏,풏 ∈ 푵∗.

Soluție: Scriem suma astfel: 푆 = 2 + 2 + 2 + ⋯+ 2 + 1

푆 = 1 + 2 + 2 + ⋯+ 2 + 2

− − −− − −− −− − −− − − −

0 = (2 − 1) + 2(2− 1) + 2 (2− 1) +⋯+ 2 (2− 1) + 1− 2 ⇒

⇒ 0 = 1 + 2 + 2 + ⋯+ 2 + 1− 2 ⇒

⇒ 2 − 1 = 1 + 2 + 2 + ⋯+ 2 ⇒ 2 − 1 + 2 = 1 + 2 + 2 + ⋯+ 2 + 2 ⇒

2 − 1 = 푆

2. Să se calculeze suma 푺 = ퟏ + 풂 + 풂ퟐ + 풂ퟑ + ⋯+ 풂풏,풏 ∈ 푵∗,풂 ∈ 푹∗ − {ퟏ} fixat.

Soluție: 푆 = 푎 + 푎 + 푎 + ⋯+ 푎 + 1

푆 = 1 + 푎 + 푎 + 푎 +⋯+ 푎

−− −− − − −− − −− −− −

0 = (푎 − 1) + 푎(푎 − 1) + 푎 (푎 − 1) + ⋯+ 푎 (푎 − 1) + 1− 푎 ⇒

⇒ 0 = (푎 − 1)(1 + 푎 + 푎 + ⋯+ 푎 ) + 1− 푎 ⇒푎 − 1푎 − 1

= 1 + 푎 + 푎 + ⋯+ 푎 ⇒

⇒ 푆 =푎 − 1푎 − 1

,∀푛 ∈ 푁∗

3. Să se calculeze suma ퟏ + ퟑ + ퟓ + ퟕ + ⋯+ (ퟐ풏 − ퟏ),풏 ∈ 푵∗.

Soluție:

푆 = 1 + 2 + 3 + ⋯+ (푛 − 1) + 푛

푆 = 2 + 2 +⋯+ (푛 − 1) + 푛 + 1

− − − −− − −− −− − −− − − −−

0 = (2 − 1 ) + (3 − 2 ) + (4 − 3 ) +⋯+ [푛 − (푛 − 1) ] + 1 − 푛 ⇒

Page 7: ROMANIAN MATHEMATICAL MAGAZINE R.M.M. · pentru ∈ z∗, pe care am rezolvat-o așa cum urmează, am păstrat algoritmul și pentru calculul altor sume, unele putând fi calculate

S.S.M.ROMÂNIA - Filiala Mehedinți 2017

7 ROMANIAN MATHEMATICAL MAGAZINE NR. 18

⇒ 0 = 3 + 5 + 7 + ⋯+ (2푛 − 1) + 1− 푛 ⇒ 1 + 3 + 5 + 7 + ⋯+ (2푛 − 1) = 푛 .

4. Să se calculeze suma ퟏ + ퟐ + ퟑ + ⋯+ 풏,풏 ∈ 푵∗.

Soluție: Considerăm suma 푆 = 1 ⋅ 2 + 2 ⋅ 3 + ⋯+ (푛 − 1)푛 + 푛(푛 + 1). Atunci

푆 = 2 ⋅ 3 + 3 ⋅ 4 +⋯+ 푛(푛 + 1) + 1 ⋅ 2. Scădem cele două sume și avem

0 = 2 ⋅ (3− 1) + 3(4− 2) +⋯+ 푛(푛 + 1 − 푛 + 1) + 1 ⋅ 2 − 푛(푛 + 1) ⇒

⇒ 푛(푛 + 1) = 2 ⋅ 2 + 3 ⋅ 2 +⋯+ 푛 ⋅ 2 + 1 ⋅ 2 ⇒ 푛(푛 + 1) = 2(1 + 2 + 3 + ⋯+ 푛) ⇒

⇒ 퐴 =푛(푛 + 1)

2.

5. Să se calculeze suma 푨 = ퟏ ⋅ ퟐ + ퟐ ⋅ ퟑ + ퟑ ⋅ ퟒ +⋯+ 풏(풏+ ퟏ),풏 ∈ 푵∗.

Soluție: Considerăm suma 푆 = 1 ⋅ 2 ⋅ 3 + 2 ⋅ 3 ⋅ 4 + ⋯+ (푛 − 1)푛(푛 + 1) + 푛(푛 + 1)(푛 + 2). Atunci 푆 = 2 ⋅ 3 ⋅ 4 + 3 ⋅ 4 ⋅ 5 + ⋯+ 푛(푛 + 1)(푛 + 2) + 1 ⋅ 2 ⋅ 3. Scădem cele două sume și avem

0 = 2 ⋅ 3(4− 1) + 3 ⋅ 4(5− 2) + ⋯+ 푛(푛 + 1)(푛 + 2− 푛 + 1) + 1 ⋅ 2 ⋅ 3− 푛(푛 + 1)(푛 + 2) ⇒

⇒ 푛(푛 + 1)(푛 + 2) = 1 ⋅ 2 ⋅ 3 + 2 ⋅ 4 ⋅ 3 + 3 ⋅ 4 ⋅ 3 +⋯+ 푛(푛 + 1) ⋅ 3 ⇒

⇒푛(푛 + 1)(푛 + 2)

3= 1 ⋅ 2 + 2 ⋅ 3 + 3 ⋅ 4 + ⋯+ 푛(푛 + 1) ⇒ 퐴 =

푛(푛 + 1)(푛 + 2)3

.

6. Să se calculeze suma 푨 = ퟏ ⋅ ퟐ ⋅ ퟑ + ퟐ ⋅ ퟑ ⋅ ퟒ + ퟑ ⋅ ퟒ ⋅ ퟓ + ⋯+ 풏(풏+ ퟏ)(풏+ ퟐ),풏 ∈ 푵∗.

Soluție: Considerăm suma

푆 = 1 ⋅ 2 ⋅ 3 ⋅ 4 + 2 ⋅ 3 ⋅ 4 ⋅ 5 + ⋯+ (푛 − 1)푛(푛 + 1)(푛 + 2) + 푛(푛 + 1)(푛 + 2)(푛 + 3). Atunci 푆 = 2 ⋅ 3 ⋅ 4 ⋅ 5 + 3 ⋅ 4 ⋅ 5 ⋅ 6 + ⋯+ 푛(푛 + 1)(푛 + 2)(푛 + 3) + 1 ⋅ 2 ⋅ 3 ⋅ 4. Scădem cele două sume și avem: 0 = 2 ⋅ 3 ⋅ 4(5− 1) + 3 ⋅ 4 ⋅ 5(6− 2) + ⋯+ 푛(푛 + 1)(푛 + 2)(푛 + 3 − 푛 + 1) +

+1 ⋅ 2 ⋅ 3 ⋅ 4−−푛(푛 + 1)(푛 + 2)(푛 + 3) ⇒

⇒ 푛(푛 + 1)(푛 + 2)(푛 + 3) = 1 ⋅ 2 ⋅ 3 ⋅ 4 + 2 ⋅ 3 ⋅ 4 ⋅ 4 + 3 ⋅ 4 ⋅ 5 ⋅ 4 +⋯+ 푛(푛 + 1)(푛 + 2) ⋅ 4 ⇒

⇒푛(푛 + 1)(푛 + 2)(푛 + 3)

4= 1 ⋅ 2 ⋅ 3 + 2 ⋅ 3 ⋅ 4 + 3 ⋅ 4 ⋅ 5 + ⋯+ 푛(푛 + 1)(푛 + 2) ⇒

⇒ 퐴 =푛(푛 + 1)(푛 + 2)(푛 + 3)

4.

Observație: În mod analog, dacă suma 퐵 este

퐵 = 1 ⋅ 2 ⋅ 3 ⋅… ⋅ 푝 + 2 ⋅ 3 ⋅ 4 ⋅ … ⋅ (푝 + 1) + 3 ⋅ 4 ⋅ 5 ⋅ … ⋅ (푝 + 2) + ⋯+ 푛(푛 + 1)(푛 + 2) ⋅… ⋅

⋅ (푛 + 푝 − 1),푝,푛 ∈ 푁∗, deducem 퐵 = ( )( )( )⋅…⋅( ) ,∀푝,푛 ∈ 푁∗. Scriind sub formă de

aranjamente, avem 퐴 = 퐴 + 퐴 + 퐴 +⋯+ 퐴 ,∀푝,푛 ∈ 푁∗.

7. Folosind suma 푺 = ퟏퟏ

+ ퟏퟐ

+ ퟏퟑ

+ ퟏퟒ

+ ⋯+ ퟏ풏 ퟏ

+ ퟏ풏

,풏 ∈ 푵∗, aplicând același algoritm, să se calculeze o nouă sumă.

Page 8: ROMANIAN MATHEMATICAL MAGAZINE R.M.M. · pentru ∈ z∗, pe care am rezolvat-o așa cum urmează, am păstrat algoritmul și pentru calculul altor sume, unele putând fi calculate

S.S.M.ROMÂNIA - Filiala Mehedinți 2017

8 ROMANIAN MATHEMATICAL MAGAZINE NR. 18

Soluție:

푆 =11

+12

+13

+⋯+1

푛 − 1+

1푛

푆 =12

+13

+14

+ ⋯+1푛

+ 1

− −− − −− −− − −− − −

0 =11−

12

+12−

13

+13−

14

+ ⋯+1

푛 − 1−

1푛

+1푛− 1 ⇒

⇒ 0 =1

1 ⋅ 2+

12 ⋅ 3

+1

3 ⋅ 4+ ⋯+

1(푛 − 1) ⋅ 푛

+1푛− 1 ⇒

11 ⋅ 2

+1

2 ⋅ 3+

13 ⋅ 4

+ ⋯+1

(푛 − 1) ⋅ 푛=

= 1−1푛

,푛 ∈ 푁∗.

8. Folosind suma 푺 = ퟏퟏ⋅ퟐ

+ ퟏퟐ⋅ퟑ

+ ퟏퟑ⋅ퟒ

+ ⋯+ ퟏ(풏 ퟏ)⋅풏

+ ퟏ풏⋅(풏 ퟏ) ,풏 ∈ 푵∗ și aplicând același algoritm, să

se calculeze o nouă sumă.

Soluție:

푆 =1

1 ⋅ 2+

12 ⋅ 3

+1

3 ⋅ 4+ ⋯+

1(푛 − 1) ⋅ 푛

+1

푛 ⋅ (푛 + 1)

푆 =1

2 ⋅ 3+

13 ⋅ 4

+1

4 ⋅ 5+ ⋯+

1푛 ⋅ (푛 + 1) +

11 ⋅ 2

−− − −− − − −− − −− −− − −− − − −− −−− −

0 =12

11−

13

+13

12−

14

+14

13−

15

+⋯+1푛

1푛 − 1

−1

푛 + 1+

1푛 ⋅ (푛 + 1) −

11 ⋅ 2

⇒ 0 =2

1 ⋅ 2 ⋅ 3+

22 ⋅ 3 ⋅ 4

+2

3 ⋅ 4 ⋅ 5+ ⋯+

2(푛 − 1) ⋅ 푛 ⋅ (푛 + 1) +

1푛 ⋅ (푛 + 1)−

12⇒

⇒1

1 ⋅ 2 ⋅ 3+

12 ⋅ 3 ⋅ 4

+1

3 ⋅ 4 ⋅ 5+⋯+

1(푛 − 1) ⋅ 푛 ⋅ (푛 + 1) =

14−

12푛(푛 + 1).

9. Să se calculeze pentru orice 풌,풏 ∈ 푵∗ suma:

푨 =ퟏ

ퟏ ⋅ ퟐ ⋅… ⋅ 풌+

ퟏퟐ ⋅ ퟑ ⋅… ⋅ (풌 + ퟏ) +

ퟏퟑ ⋅ ퟒ ⋅ … ⋅ (풌 + ퟐ) +⋯+

ퟏ(풏 − ퟏ) ⋅ 풏 ⋅ … ⋅ (풏+ 풌− ퟐ) +

+ퟏ

풏 ⋅ (풏+ ퟏ) ⋅ … ⋅ (풏+ 풌 − ퟏ)

Soluție:

푆 =⋅ ⋅…⋅( ) +

⋅ ⋅…⋅+⋯+

⋅( )⋅…⋅( ) + ( )⋅( )⋅…⋅( ). Scriem suma 푆 sub forma

푆 =1

2 ⋅ 3 ⋅ … ⋅ 푘+ ⋯+

1푛 ⋅ (푛 + 1) ⋅… ⋅ (푛 + 푘 − 2) +

1(푛 + 1) ⋅ (푛 + 2) ⋅… ⋅ (푛 + 푘 − 1) +

Page 9: ROMANIAN MATHEMATICAL MAGAZINE R.M.M. · pentru ∈ z∗, pe care am rezolvat-o așa cum urmează, am păstrat algoritmul și pentru calculul altor sume, unele putând fi calculate

S.S.M.ROMÂNIA - Filiala Mehedinți 2017

9 ROMANIAN MATHEMATICAL MAGAZINE NR. 18

+1

1 ⋅ 2 ⋅ … ⋅ (푘 − 1)

Prin diferență, obținem 0 =⋅…⋅( ) − +

⋅…⋅− + ⋯+

+ ( )⋅…⋅( ) − + ( )⋅( )⋅…⋅( ) − ⋅ ⋅…⋅( ), adică

0 =푘 − 1

1 ⋅ 2 ⋅ … ⋅ 푘+

푘 − 12 ⋅ 3 ⋅ … ⋅ (푘 + 1) + ⋯+

푘 − 1푛(푛 + 1) ⋅ … ⋅ (푛 + 푘 − 1) +

+ ( )⋅( )⋅…⋅( ) − ⋅ ⋅…⋅( ). Rezultă că

1푘 − 1

11 ⋅ 2 ⋅… ⋅ (푘 − 1)−

1(푛 + 1) ⋅ (푛 + 2) ⋅… ⋅ (푛 + 푘 − 1) =

=⋅ ⋅…⋅

+⋅ ⋅…⋅( ) + ⋯+ ( )⋅…⋅( ), deci

퐴 =1

푘 − 11

1 ⋅ 2 ⋅ … ⋅ (푘 − 1)−1

(푛 + 1) ⋅ (푛 + 2) ⋅… ⋅ (푛 + 푘 − 1) .

10. Folosind suma 푺 = ퟑ⋅ퟒퟏ⋅ퟐ

+ ퟒ⋅ퟓퟐ⋅ퟑ

+ ퟓ⋅ퟔퟑ⋅ퟒ

+⋯+ (풏 ퟏ)(풏 ퟐ)(풏 ퟏ)⋅풏

,풏 ∈ 푵∗ și aplicând același

algoritm, să se calculeze o nouă sumă.

Soluție:

푆 = ⋅⋅

+ ⋅⋅

+ ⋅⋅

+ ⋯+ ( )( )( )⋅

și 푆 = ⋅⋅

+ ⋅⋅

+ ⋯+ ( )( )( )⋅

+ ⋅⋅

0 =42

31−

53

+53

42−

64

+ ⋯+푛 + 1푛 − 1

⋅푛

푛 − 2−푛 + 2푛

+(푛 + 1)(푛 + 2)

(푛 − 1) ⋅ 푛−

3 ⋅ 41 ⋅ 2

⇒ 0 =4

1 ⋅ 2 ⋅ 3+

4 ⋅ 52 ⋅ 3 ⋅ 4

+ ⋯+4(푛 + 1)

(푛 − 2)(푛− 1)푛+

(푛 + 1)(푛 + 2)(푛 − 1) ⋅ 푛

−3 ⋅ 41 ⋅ 2

⇒32−

(푛 + 1)(푛 + 2)4(푛 − 1) ⋅ 푛

=4

1 ⋅ 2 ⋅ 3+

52 ⋅ 3 ⋅ 4

+ ⋯+푛 + 1

(푛 − 2)(푛 − 1)푛

11. Să se calculeze suma trigonometrică 퐬퐢퐧 풂ퟐ

+ 퐬퐢퐧 ퟑ풂ퟐ

+ 퐬퐢퐧 ퟓ풂ퟐ

+ ⋯+ 퐬퐢퐧 (ퟐ풏 ퟏ)풂ퟐ

,

풂 ∈ 푹 − {ퟐ풌흅|풌 ∈ 풁},풏 ∈ 푵∗.

Soluție: 푆 = 1 + cos 푎 + cos 2푎 + ⋯+ cos (푛 − 1)푎 + cos 푛푎. Dar

푆 = cos 푎 + cos 2푎 + cos 3푎 + ⋯+ cos 푛푎 + 1. Scăzând cele două sume, obținem:

0 = 2 sin푎2

+ 2 sin푎2

sin3푎2

+ 2 sin푎2

sin5푎2

+ ⋯+ 2 sin푎2

sin(2푛− 1)푎

2+ cos 푛푎 − 1 ⇒

⇒ 1− cos푛푎 = 2 sin푎2

sin푎2

+ sin3푎2

+ sin5푎2

+ ⋯+ sin(2푛 − 1)푎

2⇒

Page 10: ROMANIAN MATHEMATICAL MAGAZINE R.M.M. · pentru ∈ z∗, pe care am rezolvat-o așa cum urmează, am păstrat algoritmul și pentru calculul altor sume, unele putând fi calculate

S.S.M.ROMÂNIA - Filiala Mehedinți 2017

10 ROMANIAN MATHEMATICAL MAGAZINE NR. 18

⇒ = sin + sin + sin +⋯+ sin ( ) . Dacă notăm

푎 = 2푏 ⇒ ( ) = sin푏 + sin 3푏 + sin 5푏 + ⋯+ sin (2푛 − 1)푏 , 푏 ∈ 푅 − {푘휋|푘 ∈ 푍}, 푘 ∈ 푍,

푛 ∈ 푁∗. Probleme propuse:

1. Să se calculeze suma 퐴 = 1 + 2 + 3 +⋯+ 푛 ,푛 ∈ 푁∗.

2. Să se calculeze suma 퐴 = 1 ⋅ 3 + 2 ⋅ 4 + 3 ⋅ 5 + ⋯+ 푛(푛 + 2),푛 ∈ 푁∗.

3. Să se rezolve ecuația ⋅ ⋅

+⋅ ⋅

+⋅ ⋅

+⋯+ ( )⋅ ⋅( ) = −⋅ ⋅

,푛 ∈ 푁∗.

4. Să se calculeze suma 퐴 = tg + tg + ⋯+ tg ,푛 ∈ 푁∗

5. Să se calculeze suma 퐴 = tg⋅

+ tg⋅

+ tg⋅

+ ⋯+ tg ,푛 ∈ 푁∗.

6. Să se calculeze suma 퐴 =⋅ ⋅

+⋅ ⋅

+ ⋯+ ( ) ⋅ ⋅( ) ,푛 ∈ 푁∗.

Bibliografie: [1]. Colecția Revistei de Matematică din Mehedinți. [2]. Colecția Gazetei Matematice.

GENERALIZĂRI ALE UNOR ECUAȚII DIOFANTICE

By Angelica Boitan, Nicolae Papacu-Romania

În prezenta lucrare vom rezolva în numere naturale ecuațiile: (푝 + 2) − 2 = 푦 , (푝 + 2) −2 = 푧 , (푝 − 2) + 2 = 푦 , (푝 − 2) + 2 = 푧 ,푝 este un număr natural impar, ecuații care reprezintă generalizări ale unor ecuații diofantice (spre exemplu: 11 − 2 = 푦 ,

27 − 2 = 푦 , 3 − 2 = 푦 , 3 − 2 = 푧 ) din diverse reviste și cărți de matematică.

Lemă. Dacă 풂,풃 sunt două numere impare, iar (풂,풃) cel mai mare divizor comun, atunci (풂 − 풃,풂 + 풃) = ퟐ(풂,풃). În caz particular, dacă 풂,풃 sunt impare și prime între ele atunci (풂 − 풃,풂 + 풃) = ퟐ.Demonstrația este imediată.

Ecuația 풑ퟐ + ퟐ풙− ퟐ풙 = 풚ퟐ.

Rezolvăm pentru început în numere naturale ecuația: (푝 + 2) − 2 = 푦 (1) unde 푝este un număr natural impar. Deoarece 푝 este un număr natural impar, atunci 푝 ≡ 1(mod8), deci 푝 + 2 ≡ 3(mod8) și atunci (푝 + 2) ≡ 1(mod8) pentru 푥 număr natural și (푝 + 2) ≡ 3(mod8) dacă 푥 este număr natural impar. În (1), pentru 푥 = 0, avem 푦 = 0, iar pentru 푥 = 1, avem 푦 = 푝.

Presupunem în continuare că 푥 ≥ 2 și observăm că 푦 este număr natural impar. Vom demonstra că 푥 este număr natural par. Dacă 푥 ≥ 3 este număr natural impar, avem (푝 + 2) ≡ 3(mod8), dar cum 2 ≡ 0(mod8), deci (푝 + 1) − 2 ≡ 3(mod8), iar 푦 ≡ 1(mod8) și prin urmare

(푝 + 2) − 2 ≠ 푦 . Așadar 푥 = 2푡, 푡 ∈ 푁∗ și atunci ecuația (1) devine: (푝 + 2) − 2 = 푦 (2)

Demonstrăm că ecuația (2) nu are soluții naturale.

Soluție. Ecuația (2) se mai scrie ((푝 + 2) − 푦)((푝 + 2) + 푦) = 2 . Fie

Page 11: ROMANIAN MATHEMATICAL MAGAZINE R.M.M. · pentru ∈ z∗, pe care am rezolvat-o așa cum urmează, am păstrat algoritmul și pentru calculul altor sume, unele putând fi calculate

S.S.M.ROMÂNIA - Filiala Mehedinți 2017

11 ROMANIAN MATHEMATICAL MAGAZINE NR. 18

푑 = ((푝 + 2) − 푦, (푝 + 2) + 푦) = 2((푝 + 2) ,푦). Avem 푑|2푦 și deoarece 푑 |2 și 푑 număr natural par, 푦 număr natural impar, rezultă că 푑 = 2. Atunci din ((푝 + 2) − 푦)((푝 + 2) + 푦) =

2 și 푑 = 2, rezultă (푝 + 2) − 푦 = 2

(푝 + 2) + 푦 = 2 și prin adunarea celor două ecuații, obținem ecuația

(푝 + 2) = 2 + 1 (3) . Vom demonstra că ecuația (3) nu are soluții naturale. Pentru 푡 = 1 obținem 푝 = 0, ceea ce nu este posibil, iar pentru 푡 ≥ 2, avem 2 + 1 ≡ 1(mod4) și prin urmare (푝 + 2) ≡ 1(mod4), ceea ce înseamnă că 푡 este număr par. Fie 푡 = 2푤,푤 ∈ 푁∗ și atunci ecuația (3) devine (푝 + 2) = 2 + 1, sau ((푝 + 1) − 2 )((푝 + 2) + 2 ) = 1, de unde

(푝 + 2) − 2 = (푝 + 2) + 2 = 1, ceea ce nu este posibil. În concluzie, ecuația (푝 + 2) − 2 = 푦 are soluțiile 푥 = 푦 = 0 și 푥 = 1,푦 = 푝.

Ecuația 풑ퟐ + ퟏ풙− ퟐ풚 = 풛ퟐ

În mod natural se pune problema generalizării ecuației (1), adică ecuația 풑ퟐ + ퟐ풙− ퟐ풚 = 풛ퟐ

(4) unde 풑 este un număr natural impar, se poate rezolva în numere naturale.

Răspunsul este afirmativ. Analizăm pentru început cazurile 푥 ∈ {0,1} și 푦 ∈ {0,1}

În (4), pentru 푥 = 0, avem 1− 2 = 푧 și atunci 푦 = 푧 = 0, iar pentru 푥 = 1, avem 푝 + 2 − 2 =푧 . Deoarece 푝 ≡ 1(mod8) și 푧 ≡ 1(mod8), atunci 2 − 2 ≡ 0(mod8), deci 푦 = 1 și atunci 푧 = 푝. Așadar avem soluțiile 푥 = 푦 = 푧 = 0 și 푥 = 푦 = 1, 푧 = 푝. Pentru 푦 = 0, ecuația (4) devine (푝 + 2) − 1 = 푧 . Dacă 푥 ≥ 2 este impar, avem (푝 + 2) ≡ 3(mod8), sau (푝 + 2) − 1 ≡2(mod8) și prin urmare am avea 푧 ≡ 2(mod8), ceea ce nu este posibil. Deci 푥 este par, 푥 = 2푡, 푡 ∈푁∗ și avem ecuația (푝 + 2) − 1 = 푧 sau ((푝 + 2) − 푧)((푝 + 2) + 푧) = 1, adică (푝 + 2) −푧 = (푝 + 2) + 푧 = 1, imposibil. Pentru 푦 = 1 ecuația (4) devine (푝 + 2) − 2 = 푧 (5). Dacă 푥 este un număr natural par, cum (푝 + 2) ≡ 1(mod8), iar 푧 + 2 ≡ 3(mod8), atunci ecuația (5) nu are soluții. Pentru 푥 = 1, avem 푧 = 푝 și deci soluția 푥 = 푦 = 1, 푧 = 푝. Dacă 푥 este număr natural impar, 푥 ≥ 3, atunci 푥 are cel puțin un factor prim impar notat 푞, deci 푥 = 푞 ⋅ 푡 și atunci (푝 + 2) =푤 , unde 푤 = (푝 + 2) . În aceste condiții, ecuația (5) devine 푧 + 2 = 푤 cu 푞 prim impar, ecuație care este rezolvată în [1], pagina 256, problema 9 și care are soluția 푧 = 5,푞 = 3,푤 = 3, deci 푤 = (푝 + 2) = 3, adică 푡 = 푝 = 1 și imediat ecuația (5) are pentru 푝 = 1 soluția 푥 = 3,푦 = 1,푧 =5.Presupunem în continuare 푥,푦 ≥ 2 și observăm că 푧 este număr natural impar. Vom demonstra că 푥 este număr natural par. Dacă 푥 ≥ 3 este impar, avem (푝 + 2) ≡ 3(mod8), dar cum 2 ≡0(mod4), atunci (푝 + 1) − 2 ≡(mod4), iar 푦 ≡ 1(mod8) și prin urmare (푝 + 2) − 2 ≠ 푦 . Așadar 푥 = 2푡, 푡 ∈ 푁∗ și atunci ecuația (5) devine (푝 + 2) − 2푦 = 푧 (6)

Ecuația (6) se mai scrie 풑ퟐ + ퟐ풕− 풛 풑ퟐ + ퟐ

풕+ 풛 = ퟐ풚.

Fie 푑 = ((푝 + 2) − 푧, (푝 + 2) + 푧) = 2((푝 + 2) ,푧) și imediat obținem 푑|2푧. De asemenea din 푑|((푝 + 2) − 푧),푑|((푝 + 2) + 푧) și ((푝 + 2) − 푧)((푝 + 2) + 푧) = 2 , rezultă că 푑 |2 .

Avem 푑|2푧,푑 |2 ,푑 număr par, 푧 număr impar, deci 푑 = 2. Atunci ((푝 + 2) − 푧)((푝 + 2) +

푧) = 2 și 푑 = 2, rezultă (푝 + 2) − 푧 = 2

(푝 + 2) + 푧 = 2 (7). Din (7), rezultă prin scăderea ecuațiilor că

푧 = 2 − 1 și prin adunare că (푝 + 2) = 2 + 1 (8) . Dacă 푦 = 2, avem (푝 + 2) = 2, imposibil pentru că 푝 ≥ 1. Dacă 푦 = 3, avem (푝 + 2) = 3 și atunci 푝 = 1, 푡 = 1,푥 = 2푡 = 2 și 푧 = 2 − 1 = 1. Dacă 푦 = 4, avem (푝 + 2) = 5, care nu are soluții naturale. Pentru 푦 ≥ 5, avem 2 + 1 ≡ 1(mod8) și atunci (푝 + 2) ≡ 1(mod8), deci 푡 este un număr natural par. Fie

Page 12: ROMANIAN MATHEMATICAL MAGAZINE R.M.M. · pentru ∈ z∗, pe care am rezolvat-o așa cum urmează, am păstrat algoritmul și pentru calculul altor sume, unele putând fi calculate

S.S.M.ROMÂNIA - Filiala Mehedinți 2017

12 ROMANIAN MATHEMATICAL MAGAZINE NR. 18

푡 = 2푣, 푣 ∈ 푁∗, atunci ecuația (8), devine (푝 + 2) = 2 + 1 (9) sau (푝 + 2) − 1 = 2 , adică ((푝 + 2) − 1)((푝 + 2) + 1) = 2 , deoarece ((푝 + 2) − 1, (푝 + 2) + 1) = 2, avem

(푝 + 2) − 1 = 2(푝 + 2) + 1 = 2

. Din prima ecuație avem (푝 + 2) = 3 și imediat 푝 = 1 și 푣 = 1. Atunci (9)

devine 9 = 2 + 1 și deci 푦 = 5. Prin urmare 푡 = 2, adică 푥 = 4 și 푧 = 2 − 1 = 7. Așadar pentru 푝 = 1, avem 푥 = 4,푦 = 5, 푧 = 7. În concluzie ecuația (푝 + 2) − 2 = 푧 are soluțiile: 푥 = 푦 = 푧 = 0, 푥 = 푦 = 1, 푧 = 푝, iar pentru 푝 = 1 mulțimea soluțiilor ecuației 3 − 2 = 푧 este

{(0,0,0), (1,1,1), (3,1,5), (2,3,1), (4,5,7)}

Studiem în continuare rezolvarea în numere naturale a ecuației 풑ퟐ − ퟐ풙

+ ퟐ풙 = 풚ퟐ (10)

unde 풑 ≥ ퟏ este un număr natural impar.

Pentru 푝 = 1, avem ecuația (−1) + 2 = 푦 . Pentru 푥 = 1, avem 푦 = 1. Presupunem în continuare că 푥 ≥ 2. Dacă 푥 = 2푡, 푡 ∈ 푁∗, atunci ecuația de mai înainte devine 1 + 2 = 푦 sau (푦 − 2 )(푦 + 2 ) = 1, deci 푦 − 2 = 푦 + 2 = 1, ceea ce nu este posibil. Dacă 푥 = 2푡 + 1, 푡 ∈ 푁∗, ecuația de mai înainte devine −1 + 2 = 푦 . Deoarece −1 + 2 ≡ 7(mod8) și 푦 ≡ 1(mod8), rezultă că −1 + 2 ≠ 푦 . Presupunem în continuare că 푝 ≥ 3. În (10), pentru 푥 = 0, avem 푦 ∉푁, iar pentru 푥 = 1, avem 푦 = 푝. Presupunem în continuare că 푥 ≥ 2 și observăm că 푦 este număr natural impar. Vom demonstra că 푥 este număr natural par. Dacă 푥 ≥ 3 este număr natural impar, avem (푝 − 2) ≡ 7(mod8), dar cum 2 ≡ 0(mod8), deci (푝 − 2) + 2 ≡ 7(mod8), iar 푦 ≡1(mod8) și prin urmare (푝 − 2) + 2 ≠ 푦 .

Așadar 풙 = ퟐ풕, 풕 ∈ 푵∗ și atunci ecuația (1) devine : 풑ퟐ − ퟐퟐ풕

+ ퟐퟐ풕 = 풚ퟐ (11)

Demonstrăm că ecuația (2) nu are soluții naturale.

Soluție. Dacă 푝 ≥ 3 avem ((푝 − 2) ) < (푝 − 2) + 2 < ((푝 − 2) + 1) , pentru că (푝 − 2) + 2 < ((푝 − 2) + 1) ⇔ 2 < 2(푝 − 2) + 1, iar 2(푝 − 2) + 1 > 7 > 2 .

Pentru 푝 = 1, ecuația (11) devine 1 + 2 = 푦 , care nu are soluții naturale, pentru că ecuația de mai înainte se scrie (푦 − 2 )(푦 + 2 ) = 1 ⇔ 푦 − 2 = 푦 + 2 = 1 ceea ce nu este posibil.

În concluzie, ecuația (푝 − 2) + 2 = 푦 are soluția 푥 = 1,푦 = 푝.

Ecuația 풑ퟐ − ퟐ풙

+ ퟐ풚 = 풛ퟐ

În mod natural se pune problema generalizării ecuației (10), adică dacă ecuația

(푝 − 2) + 2 = 푧 (12) unde 푝 este un număr natural impar, se poate rezolva în numere naturale. Vom arăta că răspunsul este afirmativ cu o singură excepție. Analizăm pentru început cazurile 푥 ∈ {0,1} și 푦 ∈ {0,1}. În (12), pentru 푥 = 0, avem 1 + 2 = 푧 sau (푧 − 1)(푧 + 1) = 2 . Deoarece (푧 − 1,푧 + 1) = 2 și 푧 − 1 < 푧 + 1 atunci avem 푧 − 1 = 2, 푧 + 1 = 2 , de unde 푦 = 3, 푧 = 2. În (13) pentru 푥 = 1, avem 푝 − 2 + 2 = 푧 . Deoarece 푝 ≡ 1(mod8) și

푧 ≡ 1(mod8), atunci 2 − 2 ≡ 0(mod8), deci 푦 = 1 și atunci 푧 = 푝. Așadar avem soluțiile 푥 = 0,푦 = 3,푧 = 2 și 푥 = 푦 = 1, 푧 = 푝. Pentru 푦 = 0, ecuația (4) devine (푝 − 2) + 1 = 푧 sau 푧 − (푝 − 2) = 1 și atunci 푧 = 3,푝 − 2 = 2,푥 = 3 (Teorema Catalan-Mihăilescu: Ecuația 풙풚 − 풛풕 = ퟏ,풙,풚,풛, 풕 ∈ 푵∗ − {ퟏ} are doar soluția 풙 = 풕 = ퟑ,풚 = 풛 = ퟐ), ceea ce înseamnă că 푝 =2, contradicție cu 푝 impar. Pentru 푦 = 1 ecuația (12) devine (푝 − 2) + 2 = 푧 (13)

Page 13: ROMANIAN MATHEMATICAL MAGAZINE R.M.M. · pentru ∈ z∗, pe care am rezolvat-o așa cum urmează, am păstrat algoritmul și pentru calculul altor sume, unele putând fi calculate

S.S.M.ROMÂNIA - Filiala Mehedinți 2017

13 ROMANIAN MATHEMATICAL MAGAZINE NR. 18

Dacă 푥 este număr natural par, cum (푝 − 2) + 2 ≡ 3(mod8), iar 푧 ≡ 1(mod8), atunci ecuația (13) nu are soluții. Pentru 푥 = 1, avem 푧 = 푝 și deci soluția 푥 = 푦 = 1,푧 = 푝. Rămâne problemă deschisă rezolvarea ecuației (14) pentru 푥 ≥ 3 număr natural impar, adică ecuația

풑ퟐ − ퟐ풙

+ ퟐ = 풛ퟐ . (eventual pe un caz particular: ퟕ풙 + ퟐ = 풛ퟐ)

Presupunem în continuare 푥,푦 ≥ 2 și observăm că 푧 este număr natural impar. Vom demonstra că 푥 este număr natural par. Dacă 푥 ≥ 3 este impar, avem (푝 − 2) ≡ 7(mod8), dar cum 2 ≡ 0(mod4), atunci (푝 − 2) + 2 ≡ 3(mod4), iar 푦 ≡ 1(mod8) și prin urmare (푝 − 2) + 2 ≠ 푧 . Așadar 푥 = 2푡, 푡 ∈ 푁∗ și atunci ecuația (14) devine (푝 − 2) + 2 = 푧 (14) . Ecuația (6) se mai scrie (푧 − (푝 − 2) )(푧 + (푝 − 2) ) = 2 . Fie 푑 = (푧 − (푝 − 푡) , 푧 + (푝 − 2) ), care este număr natural par și 푑|2푧. De asemenea din 푑|(푧 − (푝 − 푧) ),푑|(푧 + (푝 − 2) ) și (푧 − (푝 − 2) )(푧 +푝2−2푡=2푦, rezultă că 푑2|2푦. Avem 푑2푧, 푑22푦,푑 număr par, 푧 număr impar, deci 푑=2.

Atunci din (푧 − (푝 − 2) )(푧 + (푝 − 2) ) = 2 și 푑 = 2, rezultă 푧 − (푝 − 2) = 2

푧 + (푝 − 2) = 2 (15)

Din (15), rezultă prin adunarea ecuațiilor că 푧 = 2 + 1 și prin scădere că (푝 − 2) = 2 − 1 (16) . Dacă 푦 = 2, avem (푝 − 2) = 0, imposibil. Dacă 푦 = 3, avem (푝 − 2) = 1 și atunci, 푡 = 0,

imposibil. Dacă 푡 = 1, avem 2 = 푝 − 1, de unde 푝 − 1 = 2

푝 + 1 = 2 , adică 푝 = 3 și 푦 = 5. Pentru

푦 ≥ 4 și 푡 ≥ 2 ecuția (16) care se mai scrie 2 − (푝 − 2) = 1 nu are soluții (teorema Catalan-Mihăilescu).

Bibliografie: [1] I. Cucurezeanu, Ecuații în numere întregi, Editura Aramis, București, 2006

[2] I. Cucurezeanu, Pătrate și cuburi perfecte de numere întregi, Editura Gil, Zalău, 2007

[3] W. Sierpinski, Elementary theory of numbers, Warszawa, 1964

POWER SUMS

By Daniel Mackleen-India

Abstract: In this mathematical note we build a recurrence method for calculus of natural power sums. Keywords: sums, power.

Let be 풎,풏 ∈ ℕ∗. 푭풎 = ퟏ풎 + ퟐ풎 + ⋯+ 풏풎, 푵 = (풏 풏ퟐ 풏ퟑ … 풏풎 ퟏ) ∈ 푴ퟏ,풏 ퟏ(ℝ)

푩 =

푭ퟏ푭ퟐ⋮

푭풎 ퟏ

∈ 푴풎 ퟏ,ퟏ(ℝ)

We consider the linear system:

(푺):

⎩⎨

⎧ 풙ퟏ ⋅ ퟏퟏ + 풙ퟐ ⋅ ퟏퟐ + 풙ퟑ ⋅ ퟏퟑ + ⋯+ 풙풎 ퟏ ⋅ ퟏ풎 ퟏ = 푭ퟏ풙ퟏ ⋅ ퟐퟏ + 풙ퟐ ⋅ ퟐퟐ + 풙ퟑ ⋅ ퟐퟑ + ⋯+ 풙풎 ퟏ ⋅ ퟐ풎 ퟏ = 푭ퟐ… … … … … … … … … … … … … … … … … … … … … … …

풙ퟏ ⋅ (풎 + ퟏ)ퟏ + 풙ퟐ ⋅ (풎+ ퟏ)ퟐ + 풙ퟑ ⋅ (풎+ ퟏ)ퟑ +⋯+ 풙풎 ퟏ(풎+ ퟏ)풎 ퟏ = 푭풎 ퟏ

Page 14: ROMANIAN MATHEMATICAL MAGAZINE R.M.M. · pentru ∈ z∗, pe care am rezolvat-o așa cum urmează, am păstrat algoritmul și pentru calculul altor sume, unele putând fi calculate

S.S.M.ROMÂNIA - Filiala Mehedinți 2017

14 ROMANIAN MATHEMATICAL MAGAZINE NR. 18

Let be: 푨 =

ퟏퟏ ퟏퟐ ퟏퟑ … ퟏ풎 ퟏ

ퟐퟏ ퟐퟐ ퟐퟑ … ퟐ풎 ퟏ

⋮ ⋮ ⋮ ⋱ ⋮(풎 + ퟏ)ퟏ (풎+ ퟏ)ퟐ (풎+ ퟏ)ퟑ … (풎 + ퟏ)풎 ퟏ

∈ 푴풎 ퟏ(ℝ)

and 푫 = 퐝퐞퐭 푨. The solution of the system is: 풙ퟏ = 푫ퟏ푫

; 풙ퟐ = 푫ퟐ푫

; … ; 풙풎 ퟏ = 푫풎 ퟏ푫풏

푫풊 =

ퟏퟏ … ퟏ풊 ퟏ 푭ퟏ ퟏ풊 ퟏ … ퟏ풎 ퟏ

ퟐퟏ … ퟐ풊 ퟏ 푭ퟐ ퟐ풊 ퟏ … ퟐ풎 ퟏ

⋮ ⋮ ⋮ ⋮ ⋮ ⋮ ⋮(풎 + ퟏ)ퟏ … (풎+ ퟏ)풊 ퟏ 푭풎 ퟏ (풎 + ퟏ)풊 ퟏ … (풎+ ퟏ)풎 ퟏ

(푺):

ퟏퟏ ퟏퟐ … ퟏ풎 ퟏ

ퟐퟏ ퟐퟐ … ퟐ풎 ퟏ

⋮ ⋮ ⋱ ⋮(풎 + ퟏ)ퟏ (풎 + ퟏ)ퟐ … (풎+ ퟏ)풎 ퟏ

풙ퟏ풙ퟐ⋮

풙풎 ퟏ

=

푭ퟏ푭ퟐ⋮

푭풎 ퟏ

(푺):푨

풙ퟏ풙ퟐ⋮

풙풎 ퟏ

=

푭ퟏ푭ퟐ⋮

푭풎 ퟏ

풙ퟏ풙ퟐ⋮

풙풎 ퟏ

= 푨 ퟏ

푭ퟏ푭ퟐ⋮

푭풎 ퟏ

(풏 풏ퟐ 풏ퟑ … 풏풎 ퟏ)

풙ퟏ풙ퟐ⋮

풙풎 ퟏ

= 푵푨 ퟏ푩

풏풙ퟏ + 풏ퟐ풙ퟐ +⋯+ 풏풎 ퟏ풙풎 ퟏ = 푵푨 ퟏ푩

We write this equality for 풏 = ퟏ,ퟐ, …

⎩⎨

⎧ 풙ퟏ + 풙ퟐ +⋯+ 풙풎 ퟏ = 푵푨 ퟏ푩ퟐ풙ퟏ + ퟐퟐ풙ퟐ +⋯+ ퟐ풎 ퟏ풙풎 ퟏ = 푵푨 ퟏ푩… … … … … … … … … … … … … … … … … …

풎풙ퟏ + 풏ퟐ풙ퟐ +⋯+ 풏풎 ퟏ풙풎 ퟏ = 푵푨 ퟏ푩

By adding: 푭ퟏ + 풏푭ퟐ +⋯+ 풏풎푭풎 ퟏ = 풎 ퟏ풏푵푨 ퟏ푩

HLAWKA’S INEQUALITIES FOR A CLASS OF FUNCTIONS

By Daniel Sitaru, Leonard Giugiuc-Romania

Abstract: In this article we build inequalities similar of classical Hlawka’s inequality

Keywords and phrases: Hlawka; Inequalities

Hlawka’s inequality is well known for 풙,풚,풛 ∈ ℂ:

|풙| + |풚| + |풛| + |풙+ 풚 + 풛| ≥ |풙 + 풚| + |풚+ 풛| + |풛+ 풙|

Processing an idea of Dan Ștefan-Marinescu, Leonard Giugiuc proved the next assertion:

Page 15: ROMANIAN MATHEMATICAL MAGAZINE R.M.M. · pentru ∈ z∗, pe care am rezolvat-o așa cum urmează, am păstrat algoritmul și pentru calculul altor sume, unele putând fi calculate

S.S.M.ROMÂNIA - Filiala Mehedinți 2017

15 ROMANIAN MATHEMATICAL MAGAZINE NR. 18

Proposition 1: Let 푓: [0,∞) → ℝ be a differentiable function such that 푓 is convex and 푓(0) = 0. For any nonnegative numbers 푥,푦, 푧 holds the following inequality:

푓(푥) + 푓(푦) + 푓(푥 + 푦 + 푧) ≥ 푓(푥 + 푦) + 푓(푥 + 푧) + 푓(푦 + 푧) (1)

Proof : (Leonard Giugiuc) Let 푥,푦, 푧 be fixed nonnegative numbers. Let be 푔: [0,∞) → ℝ,

푔(푡) = 푓(푡) + 푓(푦) + 푓(푧) + 푓(푡 + 푦 + 푧)− 푓(푡 + 푦)− 푓(푡 + 푧)− 푓(푦 + 푧)

푔 (푡) = 푓 (푡) + 푓 (푡 + 푦 + 푧) − 푓 (푡 + 푦) − 푓 (푡 + 푧). We observe that 푔(0) = 0.

Lemma: Let ℎ: [0,∞) → ℝ be a convex function. If 푎, 푏, 푐,푑 ∈ ℝ such that:

0 ≤ 푎 ≤ 푏, 푐 ≤ 푑 and 푎 + 푑 = 푏 + 푐, then: ℎ(푎) + ℎ(푑) ≥ ℎ(푏) + ℎ(푐).

Lemma’s proof: 푏 ∈ [푎,푑] ⇒ (∃)푘 ∈ [0,1]; 푏 = 푘푎+ (1− 푘)푑

From 푎 + 푑 = 푏 + 푐 we get 푐 = 푎 + 푑 − 푏, 푐 = 푎 + 푑 − 푘푎 − (1− 푘)푑 ⇒ 푐 = (1− 푘)푎+ 푘푑

ℎ(푏) = ℎ(푘푎 + (1 − 푘)푑) ≤ 푘ℎ(푎) + (1− 푘)ℎ(푑) (2)

푐 ∈ [푎,푑] ⇒ (∃)푘 ∈ [0,1]; 푐 = (1 − 푘)푎 + 푘푑,

ℎ(푐) = ℎ (1 − 푘)푎 + 푘푑 ≤ (1− 푘)ℎ(푎) + 푘ℎ(푑) (3)

By adding (2); (3): ℎ(푏) + ℎ(푐) ≤ ℎ푘(푎) + (1− 푘)ℎ(푎) + (1− 푘)ℎ(푑) + 푘ℎ(푑) ≤

≤ (1 + 푘 − 푘)ℎ(푎) + (1 − 푘 + 푘)ℎ(푑) = ℎ(푎) + ℎ(푑)

Back to proposition’s 1 proof: We apply the lemma for the convex function 푓 and:

푎 = 푡; 푏 = 푡 + 푦; 푐 = 푡 + 푧; 푑 = 푡 + 푦 + 푧

푓 (푡) + 푓 (푡 + 푦 + 푧) ≥ 푓 (푡 + 푦) + 푓 (푡 + 푧), 푓 (푡) + 푓 (푡 + 푦 + 푧) − 푓 (푡 + 푦)− 푓 (푡 + 푧) ≥ 0

푔 (푡) ≥ 0 ⇒ 푔 is increasing ⇒ 푔(푥) ≥ 푔(0) = 0

푓(푥) + 푓(푦) + 푓(푧) + 푓(푥 + 푦 + 푧)− 푓(푥 + 푦) − 푓(푦 + 푧)− 푓(푧 + 푥) ≥ 0

푓(푥) + 푓(푦) + 푓(푧) + 푓(푥 + 푦 + 푧) ≥ 푓(푥 + 푦) + 푓(푦+ 푧) + 푓(푧 + 푥)

Proposition 2 (Marian Dincă): If 풙,풚, 풛 ∈ [ퟎ,∞) then:

√풙 + 풚 + √풛 + 풙 + 풚 + 풛 ≥ 풙 + 풚 + 풚 + 풛 + √풛 + 풙

Proof: Let be 푓: [0,∞) → ℝ; 푓(푥) = √푥;푓(0) = 0.

푓 (푥) =1

2√푥;푓 (푥) =

−14푥√푥

;푓 (푥) =4√푥 +

√16푥

> 0

From proposition 1: 푓(푥) + 푓(푦) + 푓(푧) + 푓(푥 + 푦 + 푧) ≥ 푓(푥 + 푦) + 푓(푦 + 푧) + 푓(푧 + 푥)

Page 16: ROMANIAN MATHEMATICAL MAGAZINE R.M.M. · pentru ∈ z∗, pe care am rezolvat-o așa cum urmează, am păstrat algoritmul și pentru calculul altor sume, unele putând fi calculate

S.S.M.ROMÂNIA - Filiala Mehedinți 2017

16 ROMANIAN MATHEMATICAL MAGAZINE NR. 18

√푥 + 푦 + √푧 + 푥 + 푦 + 푧 ≥ 푥 + 푦 + 푦 + 푧 + √푧 + 푥

Proposition 3 (Daniel Sitaru): If 풙,풚,풛 ∈ [ퟎ,∞) then:

ퟐ풙 + ퟐ풚 + ퟐ풛 + ퟐ풙 풚 풛 ≥ ퟐ풙 풚 + ퟐ풚 풛 + ퟐ풛 풙 + ퟏ

Proof: Let be 푓: [0,∞) → ℝ;푓(푥) = 2 − 1;푓(0) = 0, 푓 (푥) = 2 ln 2 ; 푓 (푥) = 2 ln 2 ;

푓 (푥) = 2 ln 2 , From proposition 1:

푓(푥) + 푓(푦) + 푓(푧) + 푓(푥 + 푦 + 푧) ≥ 푓(푥 + 푦) + 푓(푦+ 푧) + 푓(푧 + 푥)

2 − 1 + 2 − 1 + 2 − 1 + 2 − 1 ≥ 2 − 1 + 2 − 1 + 2 − 1

2 + 2 + 2 + 2 ≥ 2 + 2 + 2 + 1

Proposition 4 (Daniel Sitaru):

Let 풇: [ퟎ,∞) → ℝ be a differentiable, increasing function such that 풇 is convex and 풇(ퟎ) = ퟎ. For any nonnegative numbers 풙,풚,풛 holds the following inequalities:

풇(풙) + 풇(풚) + 풇(풛) + 풇(풙 + 풚 + 풛) ≥ 풇 ퟐ 풙풚 + 풇 ퟐ 풚풛 + 풇 ퟐ√풛풙

풇(풙) + 풇(풚) + 풇(풛) + 풇(풙 + 풚 + 풛) ≥ 풇ퟒ풙풚풙 + 풚

+ 풇ퟒ풚풛풚 + 풛

+ 풇ퟒ풛풙풛 + 풙

Proof: By proposition 1: 푓(푥) + 푓(푦) + 푓(푧) + 푓(푥 + 푦 + 푧) ≥ 푓(푥 + 푦) + 푓(푦 + 푧) + 푓(푧 + 푥)

푓 increasing and: 푥 + 푦 ≥ 2 푥푦 ⇒ 푓(푥 + 푦) ≥ 푓 2 푥푦

Analogous: 푓(푦 + 푧) ≥ 푓 2 푦푧 ;푓(푧 + 푥) ≥ 푓 2√푧푥 . By adding:

푓(푥 + 푦) + 푓(푦 + 푧) + 푓(푧 + 푥) ≥ 푓 2 푥푦 + 푓 2 푦푧 + 푓 2√푧푥 (4)

From (1); (4) ⇒ 푓(푥) + 푓(푦) + 푓(푧) + 푓(푥 + 푦 + 푧) ≥ 푓 2 푥푦 + 푓 2 푦푧 + 푓 2√푧푥 (5)

푓 increasing and 푥 + 푦 ≥ ⇒ 푓(푥 + 푦) ≥ 푓 . Analogous:

푓(푦 + 푧) ≥ 푓4푦푧푦 + 푧

;푓(푧 + 푥) ≥ 푓4푧푥푧 + 푥

By adding: 푓(푥 + 푦) + 푓(푦 + 푧) + 푓(푧 + 푥) ≥ 푓 + 푓 + 푓 (6)

From (1); (6) ⇒ 푓(푥) + 푓(푦) + 푓(푧) + 푓(푥 + 푦 + 푧) ≥ 푓 + 푓 + 푓 (7)

Proposition 5 (Daniel Sitaru): If 풙,풚, 풛 ∈ [ퟎ,∞) then:

ퟐ풙 + ퟐ풚 + ퟐ풛 + ퟐ풙 풚 풛 ≥ ퟒ 풙풚 + ퟒ 풚풛 + ퟒ√풛풙 + ퟏ

Page 17: ROMANIAN MATHEMATICAL MAGAZINE R.M.M. · pentru ∈ z∗, pe care am rezolvat-o așa cum urmează, am păstrat algoritmul și pentru calculul altor sume, unele putând fi calculate

S.S.M.ROMÂNIA - Filiala Mehedinți 2017

17 ROMANIAN MATHEMATICAL MAGAZINE NR. 18

ퟐ풙 + ퟐ풚 + ퟐ풛 + ퟐ풙 풚 풛 ≥ ퟏퟔ풙풚풙 풚 + ퟏퟔ

풚풛풚 풛 + ퟏퟔ

풛풙풛 풙 + ퟏ

Proof: Let be 푓: [0,∞) → ℝ;푓(푥) = 2 − 1;푓(0) = 0

푓 (푥) = 2 ln 2 ;푓 (푥) = 2 ln 2 ; 푓 (푥) = 2 ln 2

By proposition (4): 2 − 1 + 2 − 1 + 2 − 1 + 2 − 1 ≥ 2 √ − 1 + 2 √ − 1 + 2 √ − 1

2 + 2 + 2 + 2 ≥ 4√ + 4√ + 4√ + 1

By proposition (5): 2 − 1 + 2 − 1 + 2 − 1 + 2 − 1 ≥ 2 − 1 + 2 − 1 + 2 − 1

2 + 2 + 2 + 2 ≥ 16 + 16 + 16 + 1

Corolary: ퟐ풙 + ퟐ풚 + ퟐ풛 + ퟐ풙 풚 풛 ≥ √ퟏퟔ풙풚풙 풚

+ √ퟏퟔ풚풛풚 풛

+ √ퟏퟔ풛풙풛 풙 + ퟏ

Bibliography:

[1] – Daniel Sitaru, “Math Phenomenon” – Paralela 45 – Publishing House – Pitesti – Romania – 2016

[2] – Daniel Sitaru, Radu Gologan, Leonard Giugiuc, “300 Romanian Mathematical Challenges” – Paralela 45 – Publishing House – Pitesti – Romania – 2016

[3] – Daniel Sitaru, Claudia Nănuți, Leonard Giugiuc, Diana Trăilescu, “Inequalities” – Ecko – Print – Publishing House - Dr. Tr. Severin – Romania - 2015

GENERATOR PROBLEME DE GIMNAZIU PENTRU CONCURSURI

By Laviniu Bejenaru-Romania

La a 67-a Olimpiadă Națională de Matematică, Faza Finală din anul 2016 au fost prezentate următoarele probleme numerice pentru gimnaziu:

Clasa a V-a, Problema 2: Determinați numerele naturale 푎, 푏, 푐 cu proprietatea că 푎 + 푏 + 푐 = 푎푏푐.

Clasa a VI-a, Problema 1: Un număr natural se numește superb dacă este multiplul numărului divizorilor săi (spre exemplu, 12 este număr superb deoarece are 6 divizori și 12 este multiplu al lui 6). Determinați cel mai mare număr superb de două cifre.

Clasa a VII-a,Problema 3: Determinați numerele naturale 푝 cu proprietatea că suma primelor 푝 numere naturale nenule este un număr natural de 4 cifre având descompunerea în factori primi 2 ⋅ 3 ⋅ (푚 + 푛), unde 푚, 푛 ∈ ℕ∗.

Din punct de vedere al calculelor, se impun condiții succesive asupra numerelor și/sau cifrelor urmate de calcule efective, obținându-se astfel rezultatul dorit, rezultat ce este reprezentat de multe

Page 18: ROMANIAN MATHEMATICAL MAGAZINE R.M.M. · pentru ∈ z∗, pe care am rezolvat-o așa cum urmează, am păstrat algoritmul și pentru calculul altor sume, unele putând fi calculate

S.S.M.ROMÂNIA - Filiala Mehedinți 2017

18 ROMANIAN MATHEMATICAL MAGAZINE NR. 18

ori prin soluții multiple. Pentru clasa a V-a Problema 2, în soluția matematică prezentată în Gazeta Matematică Seria B, nr.6-7-8/2016, paginile 320-321, ca răspuns final scris apare doar 푎 = 1;푏 =2; 푐 = 3. Soluțiile sunt date de tripletul (1,2,3) și permutările sale, deci (1,3,2); (2,1,3); (2,3,1); (3,1,2); (3,2,1). Datorită simetriei relației din problemă, ne putem rezuma doar la prima soluție, caz ce apare dacă cerem ca numerele 푎, 푏, 푐 să fie crescătoare.

Este bine-cunoscut algoritmul lui Euclid pentru aflarea cmmdc a două numere, ca metodă algoritmică utilizată în matematică. În cazul nostru, pentru exactitatea și completitudinea soluțiilor, putem folosi metode algoritmice; soluția algoritmică brută presupune testarea automată a numerelor pe intervale de numere naturale foarte mari și identificarea exact a acelor numere care verifică relația dată, timpul de calcul necesar fiind relativ mare. În acest fel obținem cele 6 soluții de mai sus. O metodă algoritmică condiționată presupune deducerea din relația dată a anumitor condiții limitative pentru intervalele de căutare, caz în care timpul de calcul poate fi semnificativ diminuat.

În cazul problemei 2 din clasa a V-a de mai sus, putem avea condiția 푎 < 푏 < 푐 obținând astfel doar (1,2,3). Pentru clasa a VI-a Problema 1, punctul a., soluția este 96, din condițiile de număr superb. Numărul 96 are 12 divizori și 12|96. Metoda algoritmică asigură cu mare ușurință obținerea acestui răspuns. Trebuie spus că obținerea tuturor numerelor definite de respectivul autor ca fiind superbe, pot fi obținute imediat, cele cu două cifre fiind următoarele: 12 (are 6 divizori și 6|12); 18 (are 6 divizori și 6|18); 24 (are 8 divizori și 8|24); 36 (are 9divizori și 9|36); 40 (are 8 divizori și 8|40); 56 (are 8 divizori și 8|56); 60 (are 12 divizori și 12|60); 72 (are 12 divizori și 12|72); 80 (are 10 divizori și 10|80); 84 (are 12 divizori și 12|84); 88 (are 8 divizori și 8|88); 96 (are 12 divizori și avem 12|96).

Pentru clasa a VII-a Problema 3, din condițiile impuse obținem soluția 푝 = 63, numărul de exact 4 cifre fiind 2016. Soluția algoritmică generează, fără mult efort, același rezultat, desigur. Dacă ne uităm asupra condițiilor puse în soluția algoritmică, vedem că trebuie să impunem 푚 + 푛 să fie număr prim (deoarece suma se descompune în numere prime).

Variantă generalizată propusă pentru clasa a VII-a Problema 3: Determinați numerele naturale p cu proprietatea că suma primelor p numere naturale nenule este un număr natural de nc cifre având descompunerea în factori primi 푎 ⋅ 푏 (푚 + 푛), unde 푚,푛 ∈ ℕ∗ iar 푎 și 푏 sunt numere prime arbitrare. Pentru 푛푐 = 4,푎 = 2 și 푏 = 3 se obține problema originală. Un prim aspect este dat de faptul că pentru 푛푐 = 4, deci suma 푆 = ∑ 푖 va avea 4 cifre, avem libertatea de a alge dintre mai multe numere prime. Punând condițiile ca 푚 + 푛 să fie număr prim, iar 푎 și 푏 nu pot depăși suma 푆 = ∑ 푖 , vom obține următoarele soluții:

푝 = 63,푎 = 2,푚 = 5, 푏 = 3,푛 = 2, deci 푆 = 2016 = ∑ 푖 = 2 ⋅ 3 ⋅ (5 + 2);

푝 = 53,푎 = 3,푚 = 2, 푏 = 53,푛 = 1 deci 푆 = 1431 = ∑ 푖 = 3 ⋅ 53 ⋅ (2 + 1)

푝 = 50,푎 = 5,푚 = 2, 푏 = 17,푛 = 1, deci 푆 = 1275 = ∑ 푖 = 5 ⋅ 17 ⋅ (2 + 1)

푝 = 74,푎 = 5,푚 = 2, 푏 = 37,푛 = 1, deci 푆 = 2775 = ∑ 푖 = 5 ⋅ 37 ⋅ (2 + 1)

푝 = 52,푎 = 13,푚 = 1,푏 = 53,푛 = 1, deci 푆 = 1378 = ∑ 푖 = 13 ⋅ 53 ⋅ (1 + 1)

Page 19: ROMANIAN MATHEMATICAL MAGAZINE R.M.M. · pentru ∈ z∗, pe care am rezolvat-o așa cum urmează, am păstrat algoritmul și pentru calculul altor sume, unele putând fi calculate

S.S.M.ROMÂNIA - Filiala Mehedinți 2017

19 ROMANIAN MATHEMATICAL MAGAZINE NR. 18

푝 = 67,푎 = 17,푚 = 1,푏 = 67,푛 = 1, deci 푆 = 2278 = ∑ 푖 = 17 ⋅ 67 ⋅ (1 + 1)

Plecând de la această problemă, se pot genera noi probleme ale căror soluții complete pot fi depistate prin calcul algoritmic, ținând cont de condițiile rezultae din relație.

PROBLEME PROPUSE + SOLUȚII: Reformulăm problema 3 din faza finală a olimpiadei naționale de matematică astfel:

Problema G1: Determinați numerele naturale 푝 cu proprietatea că suma primelor 푝 numere naturale nenule este un număr natural de 4 cifre având descompunerea în factori primi 2 ⋅ 3 ⋅ 11 ⋅(푚 + 푛 + 푞), unde 푚,푛, 푞 ∈ ℕ∗.

Soluție: Desigur, este impusă condiția de număr prim pentru 푚 + 푛 + 푞, precum și celelalte condiții de limitare, obținându-se soluția푝 = 120,푚 = 2,푛 = 1, 푞 = 2 deci 푆 = 7260 = ∑ 푖 = 2 ⋅ 3 ⋅11 ⋅ (2 + 1 + 2). Trebuie precizat că la această problemă soluția este unică (dincolo de eventualele permutări ale elementelor mulțimii {2,3,11}).

Problema G2: (caz general la G1) Determinați numerele naturale 푝 cu proprietatea că suma primelor 푝 numere naturale nenule este un număr natural de 4 cifre având descompunerea în factori primi 푎 ⋅ 푏 ⋅ 푐 ⋅ (푚 + 푛 + 푞), unde 푚,푛, 푞 ∈ ℕ∗ iar 푎, 푏, 푐 sunt numere prime având o singură cifră.

Soluție: Numerele prime cu o singură cifră sunt 2,3,5 și 7. Condiția suplimentară este ca inclusiv 푚 + 푛 + 푞 să fie număr prim, iar ∑ 푖 să aibă forma dorită. Această generalizare acceptă ca soluție unică푝 = 125, cu 푆 = 7875 = ∑ 푖 = 3 ⋅ 5 ⋅ 7 ⋅ (2 + 2 + 1), deci 푚 = 2,푛 = 2și 푞 = 1.

Cazul generalizat de mai sus în care cerem ca numerele prime 푎, 푏, 푐 să aibă mai multe cifre conduce la mai multe soluții greu de calculat matematic. De exemplu, dacă numerele prime 푎, 푏, 푐 pot avea cel mult două cifre, atunci vom obține 20 de soluții, respectiv pentru valorile

푝 ∈ {120,59,60,83,51,68,92,123,125,45,89,62,117,69,105,65,110,129,114,77}

Problema G3: Vom defini un nou concept de număr. Vom spune că un număr natural n este articulat dacă el este format din cifre distincte și are proprietatea că toate grupele de câte două cifre alăturate, privite ca număr, divid numărul dat. Exemplu de număr articulat: 1248 deoarece 1248 = 12 × 104 ⇒ 12|1248, 1248 = 24 × 52 ⇒ 24|1248, 1248 = 48 × 26 ⇒ 48|1248 și are toate cifrele distincte.Determinați toate numerele articulate de patru cifre.Determinați toate numerele articulate de cinci cifre. Soluție: Cele de patru cifre sunt 1248; 1428; 2346; 4692. Cazul numerelor articulate de cinci cifre are soluție unică, anume 31248.

ÎN LEGĂTURĂ CU O PROBLEMĂ DATĂ ÎN AZERBAIJAN 2016, TST, BMO

By Marin Chirciu-Romania

Pentru pregătirea Lotului Național în vederea participării la Olimpiada Balcanică de Matematică din 2016, Azerbaijan în cadrul unui Test de Selecție al echipei a fost propusă următoarea problemă:

Page 20: ROMANIAN MATHEMATICAL MAGAZINE R.M.M. · pentru ∈ z∗, pe care am rezolvat-o așa cum urmează, am păstrat algoritmul și pentru calculul altor sume, unele putând fi calculate

S.S.M.ROMÂNIA - Filiala Mehedinți 2017

20 ROMANIAN MATHEMATICAL MAGAZINE NR. 18

1. Dacă 풂,풃, 풄 ≥ ퟎ și 풂풃+ 풃풄 + 풄풂 = ퟑ demonstrați că:

ퟏ(풂 + 풃)ퟐ + ퟒ

+ퟏ

(풃 + 풄)ퟐ + ퟒ+

ퟏ(풄 + 풂)ퟐ + ퟒ

≤ퟑퟖ.

Azerbaijan BMO 2016, Preparation Exam

Soluție: „Întorc”. Inegalitatea ⇔ ∑( ) ≤ |⋅4 ⇔ ∑

( ) ≤ ⇔ ∑ ( ) ( )( ) ≤ ⇔

⇔ ∑ 1 − ( )( ) ≤ ⇔ ∑ ( )

( ) ≥ , adevărată din inegalitatea lui Bergström:

푀 =(푏 + 푐)

4 + (푏 + 푐) ≥4(푎 + 푏 + 푐)

12 + ∑(푏 + 푐) =4(∑푎 + 2∑푏푐)

4∑푏푐 + 2∑푎 + 2∑푏푐=

2∑푎 + 4∑푏푐∑푎 + 3∑푏푐

≥( ) 3

2=

= 푀 , unde (1) ⇔ ∑푎 + 8∑푏푐 ≥ 3∑ + 9∑푏푐 ⇔ ∑푎 ≥ ∑푏푐, evident. Egalitatea are loc dacă și numai dacă 푎 = 푏 = 푐 = 1. Cu această inegalitate se poate obține următoarea aplicație în triunghi:

∑ ퟏ

ퟒ ퟑ 퐭퐠푩ퟐ 퐭퐠푪ퟐퟐ ≤

ퟑퟖ. Soluție: Avem ∑ tg tg = 1 ⇒ ∑ 3 tg tg = 3;푥 = √3 tg și aplicând

inegalitatea (*) obținem: ∑ ( ) = ∑√ √

= ∑ ≤ . Similar problemei

din Azerbaijan într-o revistă de matematică din China este propusă:

2. Dacă 풂,풃, 풄 ≥ ퟎ și 풂풃+ 풃풄 + 풄풂 = ퟑ demonstrați că ∑ ퟏퟐ (풃 풄)ퟐ ≤

ퟏퟐ.

Mathematics Bulletin, China, Beijing 5/2016

Soluție: „Întorc”inegalitatea:

⇔1

2 + (푏 + 푐) ≤12

|⋅2 ⇔2

2 + (푏 + 푐) ≤ 1 ⇔2 + (푏 + 푐) − (푏 + 푐)

2 + (푏 + 푐) ≤ 1 ⇔

⇔ ∑ 1 − ( )( ) ≤ 1 ⇔ ∑ ( )

( ) ≥ 2, adevărată din inegalitatea lui Bergström:

푀 =(푏 + 푐)

2 + (푏 + 푐) ≥4(푎 + 푏 + 푐)6 +∑(푏 + 푐) =

4(∑푎 + 2∑ 푏푐)2∑푏푐 + 2∑푎 + 2∑푏푐

=2∑푎 + 4∑ 푏푐∑푎 + 2∑푏푐

= 2 =

= 푀 . Egalitatea are loc dacă și numai dacă 푎 = 푏 = 푐 = 1. În baza acestui rezultat obținem inegalitatea în triunghi:

∑ ퟏ

ퟐ ퟑ 퐭퐠푩ퟐ 퐭퐠푪ퟐퟐ ≤

ퟏퟐ . Soluție: Avem ∑ tg tg = 1 ⇒ ∑ 3 tg tg = 3;푥 = √3 tg și aplicând

inegalitatea 2) obținem:

12 + (푦 + 푧) =

1

2 + √3 tg + √3 tg=

1

2 + 3 tg + tg≤

12.

Page 21: ROMANIAN MATHEMATICAL MAGAZINE R.M.M. · pentru ∈ z∗, pe care am rezolvat-o așa cum urmează, am păstrat algoritmul și pentru calculul altor sume, unele putând fi calculate

S.S.M.ROMÂNIA - Filiala Mehedinți 2017

21 ROMANIAN MATHEMATICAL MAGAZINE NR. 18

Articolul își propune generalizarea acestor rezultate și obținerea unor inegalități în triunghi.

3. Dacă 풂,풃, 풄 ≥ ퟎ și 풂풃+ 풃풄 + 풄풂 = ퟑ arătați că ∑ ퟏ풏 (풃 풄)ퟐ ≤

ퟑ풏 ퟒ

, unde 풏 ≥ ퟐ. By Marin Chirciu

Soluție: „Întorc”. Inegalitatea ⇔ ∑( ) ≤ |⋅푛 ⇔ ∑

( ) ≤ ⇔

⇔푛 + (푏 + 푐) − (푏 + 푐)

푛 + (푏 + 푐) ≤3푛푛 + 4

⇔ 1−(푏 + 푐)

푛 + (푏 + 푐) ≤3푛푛 + 4

⇔ ∑ ( )( ) ≥ , adevărată din inegalitatea lui Bergström:

푀 =(푏 + 푐)

푛 + (푏 + 푐) ≥4(푎 + 푏 + 푐)

3푛 + ∑(푏 + 푐) =4(∑푎 + 2∑푏푐)

푛∑푏푐 + 2∑푎 + 2∑푏푐=

= ∑ ∑∑ ( )∑ ≥

( )= 푀 , unde (2) ⇔ (푛 + 4)∑푎 + (2푛 + 8)∑ 푏푐 ≥

≥ 6∑푎 + (3푛 + 6)∑푏푐 ⇔ (푛 − 2)∑푎 ≥ (푛 − 2)∑푏푐, evident pentru 푛 ≥ 2. Egalitatea are loc pentru 푎 = 푏 = 푐 = 1. Cu inegalitatea de mai sus obținem următoarele inegalități în triunghi:

∑ ퟏ

풏 ퟑ 퐭퐠푩ퟐ 퐭퐠푪ퟐퟐ ≤

ퟑ풏 ퟒ

, unde 풏 ≥ ퟐ.

Soluție: Avem ∑ tg tg = 1 ⇒ ∑ 3 tg tg = 3;푥 = √3 tg și aplicând inegalitatea 4) obținem:

1푛 + (푦 + 푧) =

1

푛 + 3 tg + tg≤

3푛 + 4

.

∑ ퟏ풏 ퟑ(퐜퐭퐠푩 퐜퐭퐠 푪)ퟐ ≤

ퟑ풏 ퟒ

, unde 풏 ≥ ퟐ, în triunghiul ascuțitunghic.

Soluție: Avem ∑ ctg퐵 ctg퐶 = 1 ⇒ ∑3 ctg퐵 tg퐶 = 3;푥 = √3 ctg퐴 și aplicând inegalitatea 4)

obținem: ∑ ( ) = ∑( ) ≤ . Obs. Este necesar ca triunghiul 퐴퐵퐶 să fie

ascuțitunghic deoarece 푥,푦, 푧 ≥ 0,푥 = √3 ctg퐴, 푦 = √3 ctg퐵 , 푧 = √3 ctg퐶.

Bibliografie: 1. Azerbaijan BMO 2016, Preparation Exam.2. Mathematics Bulletin, China Beijing 5/2016. 3. Marin Chirciu, Inegalități algebrice, de la inițiere la performanță, Editura Paralela 45, Pitești, 2014. 4. Marin Chirciu, Inegalități geometrice, de la inițiere la performanță, Editura Paralela 45, Pitești, 2015. 5. Marin Chirciu, Inegalități trigonometrice, de la inițiere la performanță, Editura Paralela 45, Pitești, 2016.

PROBLEMĂ COMENTATĂ 27272, G.M. 9/2016 By Ioan Șerdean-Romania

Fie 풂,풃, 풄 numere reale pozitive cu 풂풃풄 = 풂+ 풃+ 풄 + ퟐ. Să se arate că:

Page 22: ROMANIAN MATHEMATICAL MAGAZINE R.M.M. · pentru ∈ z∗, pe care am rezolvat-o așa cum urmează, am păstrat algoritmul și pentru calculul altor sume, unele putând fi calculate

S.S.M.ROMÂNIA - Filiala Mehedinți 2017

22 ROMANIAN MATHEMATICAL MAGAZINE NR. 18

√퐚ퟐퟑ + √퐛ퟐퟑ + √퐜ퟐퟑ

ퟐ≥

ퟏ√퐚ퟑ +

ퟏ√퐛ퟑ +

ퟏ√퐜ퟑ

Soluția 1: Rezolvarea pleacă de la analiza condiției problemei: 푎푏푐 = 푎 + 푏 + 푐 + 2 (1),

푎, 푏, 푐 ∈ (0,∞). Această condiție este „uzuală” deja pentru cei care au anumită experiența a acestui domeniu. O primă idee este că relația (1) permite existența numerelor 푥,푦,푧 ∈ (0,∞) astfel încât

푎 = , 푏 = , 푐 = (2). De aici descriem o soluție, bazată pe (2), anume acea în care mai folosim următoarele rezultate:

Propoziția 1: Avem: 풙ퟏ 풙ퟐ ⋯ 풙풏풏

≥ 풙ퟏ풙ퟐ … 풙풏풏 ,풙ퟏ ∈ (ퟎ,∞), 풊 = ퟏ,풏 (inegalitatea mediilor)

Propoziția 2: Avem: 풂ퟐ + 풃ퟐ + 풄ퟐ ≥ 풂풃+ 풂풄 + 풃풄,풂,풃, 풄 ∈ (ퟎ,∞)

Folosind (2), relația de arătat revine la:

+ + ≥ + + (3)

Pentru a arătă (3) folosim metoda intercalării. Avem

+ + ≥⏟( )

+ + =√

(4)

Pe de altă avem: + + ≤√

+√

+√

=√

√ √ √√

(5)

Arătăm acum că: √⋅ ≥

√⋅ √ √ √

√ (6)

Într-adevăr (6) se scrie echivalent cu:

푥푦 + 푦푧 + 푧푥 ≥ 푥 푦푧 + 푦√푥푧 + 푧 푥푦 ⇔ 푥푦 + 푦푧 + √푧푥

≥ 푥푦 푦푧 + 푥푦 √푧푥 + 푦푧 √푧푥 , inegalitate adevărată conform cu propoziția 2.

Din (6) și (5) obținem (3):

12

푦 + 푧푥

+푥 + 푧푦

+푥 + 푦푧

≥푥

푦 + 푧+

푦푥 + 푧

+푧

푥+ 푦

Solutia 2: O altă variațiune bazată pe transformarea (2) este și următoarea rezolvare, în care mai

folosim următorul rezultat: Propoziția 3. Avem: 풂ퟑ 풃ퟑ

ퟑ≥ 풂 풃

ퟐ,풂,풃 ∈ (ퟎ,∞). Ca și în cazul

precedent, inegalitatea de arătat revine la:

ퟏퟐ

풚 풛풙

ퟐퟑ+ 풙 풛

ퟐퟑ+ 풙 풚

ퟐퟑ≥ 풙

풚 풛ퟑ + 풚

풙 풛ퟑ + 풛

풙 풚ퟑ (7)

Page 23: ROMANIAN MATHEMATICAL MAGAZINE R.M.M. · pentru ∈ z∗, pe care am rezolvat-o așa cum urmează, am păstrat algoritmul și pentru calculul altor sume, unele putând fi calculate

S.S.M.ROMÂNIA - Filiala Mehedinți 2017

23 ROMANIAN MATHEMATICAL MAGAZINE NR. 18

Din propoziția (1) găsim, pe rând că: + √4 ≥ 2 2 (8), + √4 ≥ 2 2 (9)

+ √4 ≥ 2 2 (10) . Adunând membru cu membru (8), (9), (10) găsim

+ + + 3√4 ≥ 2√2 + + (11)

Pe de altă parte, folosind propoziția (1) găsim

√2 + + ≥ 3 8 ( )( )( ) ≥⏟( ț )

3 8 ⋅ √ ⋅ √ ⋅ √ ≥ 3√4 = 3√4 (12)

푦 + 푧푥

+푥 + 푧푦

+푥 + 푦푧

+ √2푦 + 푧푥

+푥 + 푧푦

+푥 + 푦푧

≥⏟( )

푦 + 푧푥

+푥 + 푧푦

+푥 + 푦푧

+ 3√4 ≥⏟( )

2√2푦 + 푧푥

+푥 + 푧푦

+푥 + 푦푧

În final găsim că: + + ≥ √2 + + (13)

Pentru a arăta (7) arătăm că: √2 + + ≥ 2 + + (14)

Pentru a arăta (14) substituim: (푥,푦, 푧) cu (푥 , 푦 , 푧 ). Inegalitatea (14) se rescrie astfel:

√2 + √ + ≥ 2 +√

+ (15)

Folosind propoziția 3 obținem pe rând ( ) ≥√

(16), ( ) ≥√

(17)

( ) ≥√

(18) . Adunând (16), (17) și (18) găsim:

2(푦 + 푧 )푥

+2(푧 + 푥 )

푦+

2(푥 + 푦 )푧

≥1√2

푦+ 푧푥

+푧 + 푥푦

+푥 + 푦푧

=1√2

푥1푦

+1푧

+ 푦1푥

+1푧

+ 푧1푥

+1푦

≥1√2

4푥푦 + 푧

+4푦푥 + 푧

+4푧

푥 + 푦≥

≥ +√

+ = 2 +√

+ , adică relația (14).

Cum (14) este echivalentă cu (7) avem dovedit inegalitatea din enunț.

În continuare vom „exploata” condiția inițială 푎푏푐 = 푎 + 푏 + 푐 + 2 transformând-o într-una echivalentă care folosită în continuare va rezolva „spectaculos” inegalitatea din enunț.

Page 24: ROMANIAN MATHEMATICAL MAGAZINE R.M.M. · pentru ∈ z∗, pe care am rezolvat-o așa cum urmează, am păstrat algoritmul și pentru calculul altor sume, unele putând fi calculate

S.S.M.ROMÂNIA - Filiala Mehedinți 2017

24 ROMANIAN MATHEMATICAL MAGAZINE NR. 18

Lema 1. Dacă 풂 + 풃+ 풄 + ퟐ = 풂풃풄 pentru orce numere reale pozitive, atunci 풂풃풄 ≥ ퟖ.

Demonstrație. Avem șirul de echivalențe: 푎푏푐 = 푎 + 푏 + 푐 + 2 ⇔ 푎푏푐 ≥ 3√푎푏푐 + 2 ⇔

√푎푏푐 − 3√푎푏푐 ⋅ 2 ≥ 0 ⇔ √푎푏푐 − 2 √푎푏푐 − 1 ≥ 0 ⇔ √푎푏푐 − 2 ≥ 0 ⇔ 푎푏푐 ≥ 8.

Folosind lema 1 vom indica, în continuare, alte soluții.

Soluția 3. Avem succesiv:

√푎 + √푏 + √푐2 ≥

√푎 + √푏 + √푐√푎푏푐

≥⏟( ț )

√푎 ⋅ √푏 + √푎 ⋅ √푐+ √푏 ⋅ √푐√푎푏푐

=1√푎

+1√푏

+1√푐

În continuare, „variațiuni pe aceeași temă” cu 푥,푦, 푧 > 0 (19). Imediat inegalitatea de arătat se transcrie astfel:

≥ + + (20)

Dar (20) o putem transcrie sub forma: 푥푦푧(푥 + 푦 + 푧 ) ≥ 2(푥푦 + 푥푧 + 푦푧) (21)

Ultima inegalitate este evidentă căci 푥푦푧 ≥ 2 (22), conform lema 1, și

푥 + 푦 + 푧 ≥ 푥푦 + 푥푧 + 푦푧 (23) conform propoziției 2. Înmulțind membru cu mebru (22) și (23) obținem (21).

Comentarii: 1. Substituția 푎 = , 푏 = , 푐 = (24)

(amintită ca și cunoscută în legătura cu condiția 푎푏푐 = 푎 + 푏 + 푐 + 2) „sugerează” ideea de a folosi lema 1, căci în acest caz, din inegalitatea lui Cesaro: (푥 + 푦)(푦 + 푧)(푧 + 푥) ≥ 8푥푦푧 (25) avem imediat 푎푏푐 ≥ 8. 2. Interesantă este și remarca că dacă admitem adevărată concluzia

√푎 + √푏 + √푐2

≥1√푎

+1√푏

+1√푐

,

pentru orice 푎, 푏, 푐 ∈ (0,∞) atunci putem înlocui condiția problemei cu altele din care să avem 푎푏푐 ≥ 2. Un exemplu ar fi: 4푎푏푐 = 푎 + 푏 + 푐 + 8,푎, 푏, 푐 ∈ (0,∞). „Neajunsul” acestor noi condiții este faptul că în cazul lor nu se pot folosi „celebrele” substituții de la (24). De aici și problema deschisa: date fiind 푎, 푏, 푐 ∈ (0,∞) ce relații se pot stabili între ele astfel încât să existe 푥,푦, 푧 ∈(0,∞) și 푎 = , 푏 = , 푐 = . Sigur o asemenea relație o avem: 푎푏푐 = 푎 + 푏 + 푐 + 2.

GPF INEQUALITY

(GENERALIZATION FOR PRODUCT OF FUNCTIONS INEQUALITY)

By Nguyen Anh Duc –Hanoi-Vietnam

Wen proving inequality [1], I found a lemma for product of functions. Maybe this lemma had been discovered and proved by someone before I found it. So, I will call it by the name: GPF Inequality.

Page 25: ROMANIAN MATHEMATICAL MAGAZINE R.M.M. · pentru ∈ z∗, pe care am rezolvat-o așa cum urmează, am păstrat algoritmul și pentru calculul altor sume, unele putând fi calculate

S.S.M.ROMÂNIA - Filiala Mehedinți 2017

25 ROMANIAN MATHEMATICAL MAGAZINE NR. 18

1. Inequalities between 풇(풙ퟏ)풇(풙ퟐ) ⋅… ⋅ 풇(풙풏) and 풇(풙ퟏ풙ퟐ … 풙풏),풇 풙ퟏ풙ퟐ … 풙풏풏 풏

Theorem 1: Let 풙ퟏ,풙ퟐ, … ,풙풏 be nonnegative real numbers and a positive constant 풌. Given a function 풇(풙) defined on [ퟎ,∞) satisfied: 풇(풙) ≥ ퟎ;풇 (풙) ≥ ퟎ;풇 (풙)풇(풙) ≥ 풇 (풙)ퟐ and 풌풇(풙)풏 ≥ 풇(풙풏) with all 풙 ∈ [ퟎ,∞). Then we will have the following true inequality:

풌풇(풙ퟏ)풇(풙ퟐ) … 풇(풙풏) ≥ 풇(풙ퟏ풙ퟐ … 풙풏).

Proof: The inequality is equivalent to: ln푘 + ln푓(푥 ) + ln푓(푥 ) +⋯+ ln푓(푥 ) ≥ ln푓(푥 푥 … 푥 )

Define 푔(푥) = ln푓(푥). These conditions: ( )( ) ≥ 0, 푓 (푥)푓(푥) ≥ 푓 (푥) give us:

푔 (푥) ≥ 0 and 푔 (푥) ≥ 0. Rewrite the inequality as:

ln푘 + 푔(푥 ) + 푔(푥 ) +⋯+ 푔(푥 ) ≥ 푔(푥 푥 …푥 )

Since 푔 (푥) ≥ 0,푔 (푥) ≥ 0, applying Jensen’s Inequality and AM-GM, we have:

푔(푥 ) + 푔(푥 ) + ⋯+ 푔(푥 ) ≥ 푛푔푥 + 푥 + ⋯+ 푥

푛≥ 푛푔(푥 푥 …푥 )

We need to prove this: 푛 ln푘 + 푛푔(푥 ) + 푛푔(푥 ) +⋯+ 푛푔(푥 ) ≥ 푔(푥 ) + 푔(푥 ) +⋯+ 푔(푥 )

We will prove this one: ln푘 + 푛푔(푥 ) ≥ 푔(푥 ) with all 푛.

↔ 푒 ( ) ≥ 푒 ( ) ↔ 푒 ( ) ⋅ 푒 ≥ 푒 ( ) or 푘푓(푥 ) ≥ 푓(푥 ) (1)

But it is true because we have 1 from the condition.

Theorem 2: Let 풙ퟏ,풙ퟐ, … ,풙풏 ∈ [풎ퟏ,풎ퟐ] be real numbers and a positive constant 풌. Given a

function 풇(풙) defined on [풎ퟏ,풎ퟐ] satisfied: 풇(풙) ≥ ퟎ; 풇 (풙)풇(풙) ≤ ퟎ;풌풇(풎ퟐ)풏 ≥ 풇(풎ퟏ

풏). Then we will

have the following true inequality: 풌풇(풙ퟏ)풇(풙ퟐ) …풇(풙풏) ≥ 풇(풙ퟏ풙ퟐ … 풙풏).

Proof: The inequality is equivalent to:

ln푘 + ln푓(푥 ) + ln푓(푥 ) + ⋯+ ln푓(푥 ) ≥ ln푓(푥 푥 +⋯+ 푥 ) (2)

Define 푔(푥) = ln푓(푥). The condition ( )( ) ≤ 0 gives us 푔 (푥) ≤ 0. Hence, we have:

푔(푥 ) + 푔(푥 ) +⋯+ 푔(푥 ) ≥ 푛(푚 );푔(푥 푥 +⋯+ 푥 ) ≤ 푔(푚 ) (3)

Rewrite 2 as: ln푘 + 푔(푥 ) + 푔(푥 ) +⋯+ 푔(푛) ≥ 푔(푥 푥 …푥 ). Since 2 and 3 we have to prove: ln푘 + 푛푔(푚 ) ≥ 푔(푚 ) ↔ 푒 ( ) ≥ 푒 ( ) ↔ 푘푓(푚 ) ≥ 푓(푚 ) which is the condition.

Theorem 3: Let 풇(풙) be a function defined on 핀 such that: 풇(풙) ≥ ퟎ;풇 (풙) ≥ ퟎ;풇 (풙) ≥ ퟎ. Given 풙ퟏ,풙ퟐ, … ,풙풏 ∈ 핀. Then we will have the following true inequality:

풇(풙ퟏ)풇(풙ퟐ) …풇(풙풏) ≥ 풇 풙ퟏ풙ퟐ …풙풏풏 풏

Page 26: ROMANIAN MATHEMATICAL MAGAZINE R.M.M. · pentru ∈ z∗, pe care am rezolvat-o așa cum urmează, am păstrat algoritmul și pentru calculul altor sume, unele putând fi calculate

S.S.M.ROMÂNIA - Filiala Mehedinți 2017

26 ROMANIAN MATHEMATICAL MAGAZINE NR. 18

Proof: Rewrite the inequality as: 푔(푥 ) + 푔(푥 ) + ⋯+ 푔(푥 ) ≥ 푔 푥 푥 … 푥 with

푔(푥) = ln푓(푥). Applying Jensen’s Inequality and AM-GM, we obtain:

푔(푥 ) + 푔(푥 ) + ⋯+ 푔(푥 ) ≥ 푛푔푥 + 푥 +⋯+ 푥

푛≥ 푛푔 푥 푥 …푥

Theorem 4: Let 풇(풙) be defined on 핀 such that: 풇(풙) ≥ ퟎ;풇 (풙) ≤ ퟎ;풇 (풙) ≤ ퟎ. Given 풙ퟏ,풙ퟐ, … ,풙풏 ∈ 핀. Then we will have the following true inequality:

풇(풙ퟏ)풇(풙ퟐ) …풇(풙풏) ≤ 풇 풙ퟏ풙ퟐ …풙풏풏 풏

Proof: Rewrite the inequality as:

푔(푥 ) + 푔(푥 ) + 푔(푥 ) + ⋯+ 푔(푥 ) ≤ 푔 푥 푥 … 푥 with 푔(푥) = ln푓(푥).

Applying Jensen’s Inequality and AM-GM, we obtain:

푔(푥 ) + 푔(푥 ) + ⋯+ 푔(푥 ) ≤ 푛푔푥 + 푥 + ⋯+ 푥

푛≤ 푛푔 푥 푥 … 푥

You can see that Theorem 4, Theorem 5 are the similar with Theorem 1.

2. Inequalities between 풇(풙ퟏ)풇(풙ퟐ) …풇(풙풏) and 풇 풙ퟏ 풙ퟐ ⋯ 풙풏풏

풏,풇 풙ퟏ 풙ퟐ ⋯ 풙풏

Theorem 5: Let 풇(풙) be a function defined on 핀 such that: 풇 (풙) ⋅ 풇(풙) ≥ 풇 (풙)ퟐ.

Given 풙ퟏ,풙ퟐ, … ,풙풏 ∈ 핀. Then we will have the following true inequality:

풇(풙ퟏ)풇(풙ퟐ) … 풇(풙풏) ≥ 풇풙ퟏ + 풙ퟐ +⋯+ 풙풏

Proof: Define 푔(푥) = ln푓(푥). Rewrite the inequality as:

푔(푥 ) + 푔(푥 ) +⋯+ 푔(푥 ) ≥ 푛푔 ⋯ which is true since 푔 (푥) ≥ 0 and Jensen’s

Inequality. The inequality we need to prove is rewritten as:

↔ ln푓(푥 ) + ln푓(푥 ) +⋯+ 푓(푥 ) ≥ 푛 ln푓푥 + 푥 + ⋯+ 푥

↔ 푒 ( ) ( ) ⋯ ( ) ≥ 푒⋯

↔ 푓(푥 )푓(푥 ) … 푓(푥 ) ≥ 푓푥 + 푥 + ⋯+ 푥

Theorem 6: Let 풇(풙) be a function defined on 핀 such that:

풇(풙) ≥ ퟎ;풇 (풙) ≥ ퟎ;풇 (풙) ⋅ 풇(풙) ≥ 풇 (풙)ퟐ. 풌is a positive real numbers satisfied

풌풇(풙풏)풏 ≥ 풇(풙풏). Given 풙ퟏ,풙ퟐ, … ,풙풏 ∈ 핀. Then we will have the following true inequality:

Page 27: ROMANIAN MATHEMATICAL MAGAZINE R.M.M. · pentru ∈ z∗, pe care am rezolvat-o așa cum urmează, am păstrat algoritmul și pentru calculul altor sume, unele putând fi calculate

S.S.M.ROMÂNIA - Filiala Mehedinți 2017

27 ROMANIAN MATHEMATICAL MAGAZINE NR. 18

풌풇(풙ퟏ)풇(풙ퟐ) … 풇(풙풏) ≥ 풇풙ퟏ + 풙ퟐ +⋯+ 풙풏

Proof: Define 푔(푥) = ln푓(푥). These conditions: ( )( ) ≥ 0;푓 (푥)푓(푥) ≥ 푓 (푥) give us: 푔 (푥) ≥ 0

and 푔 (푥) ≥ 0. Applying Jensen’s Inequality and AM-GM, we have:

ln푘 + 푔(푥 ) + 푔(푥 ) +⋯+ 푔(푥 ) ≥ 푛푔푥 + 푥 +⋯+ 푥

푛≥

푥 + 푥 + ⋯+ 푥푛

We need to prove this:

푛 ln푘 + 푛푔(푥 ) + 푛푔(푥 ) + 푛푔(푥 ) + ⋯+ 푛푔(푥 ) ≥ 푔(푥 ) + 푔(푥 ) + ⋯+ 푔(푥 )

We will prove is true because we have 1 form the condition.This proof is similar with the proof of Theorem 1.

3. Inequalities between 풇(풙ퟏ)풇(풙ퟐ) …풇(풙풏) and

풇풙ퟏ풏 + 풙ퟐ풏 + ⋯+ 풙풏풏

풏,풇

풙ퟏ풏 + 풙ퟐ풏 + ⋯+ 풙풏풏

풏풏

Theorem 7: Let 풇(풙) be a function defined on 핀 such that: 풇(풙) ≥ ퟎ;풇 (풙) ≤ ퟎ;

풇 (풙) ⋅ 풇(풙) ≥ 풇 (풙)ퟐ. Given 풙ퟏ,풙ퟐ, … ,풙풏 ∈ 핀. Then we will have the following true inequality:

Proof: The inequality is equivalent to:

ln푓(푥 ) + ln푓(푥 ) + ⋯+ ln푓(푥 ) ≥ 푛 ln푓푥 + 푥 +⋯+ 푥

Define 푔(푥) = ln푓(푥). Those conditions give us: 푔 (푥) ≤ 0 and 푔 (푥) ≥ 0.

Rewrite the inequality as: 푔(푥 ) + 푔(푥 ) + ⋯+ 푔(푥 ) ≥ 푛푔 ⋯

Applying Jensen’s Inequality and AM-GM, we have:

푔(푥 ) + 푔(푥 ) +⋯+ 푔(푥 ) ≥ 푛푔푥 + 푥 +⋯+ 푥

푛≥ 푛푔

푥 + 푥 +⋯+ 푥푛

Theorem 8: Let 풇(풙) be a function defined on 핀 such that: 풇(풙) ≥ ퟎ;

풇 (풙) ⋅ 풇(풙) ≥ 풇 (풙)ퟐ. 풌 is a positvie real numbers satisfied 풌풇(풙)풏 ≥ 풇(풙풏).

Given 풙ퟏ,풙ퟐ, … ,풙풏 ∈ 핀. Then we will have the following true inequality:

Page 28: ROMANIAN MATHEMATICAL MAGAZINE R.M.M. · pentru ∈ z∗, pe care am rezolvat-o așa cum urmează, am păstrat algoritmul și pentru calculul altor sume, unele putând fi calculate

S.S.M.ROMÂNIA - Filiala Mehedinți 2017

28 ROMANIAN MATHEMATICAL MAGAZINE NR. 18

풌풇(풙ퟏ)풇(풙ퟐ) … 풇(풙풏) ≥ 풇풙ퟏ풏 + 풙ퟐ풏 +⋯+ 풙풏풏

Proof: The inequality is equivalent to:

ln푘 + ln푓(푥 ) + ln푓(푥 ) +⋯+ ln푓(푥 ) ≥ ln푓푥 + 푥 + ⋯+ 푥

Define 푔(푥) = ln푓(푥). The conditions give us: 푔 (푥) ≥ 0. Rewrite the inequality as:

ln푘 + 푔(푥 ) + 푔(푥 ) +⋯+ 푔(푥 ) ≥ 푔푥 + 푥 + ⋯+ 푥

Applying Jensen’s Inequality and AM-GM, we have:

푔(푥 ) + 푔(푥 ) +⋯+ 푔(푥 ) ≥ 푛푔푥 + 푥 + ⋯+ 푥

We need to prove this: 푛 ln푘 + 푛푔(푥 ) + 푛푔(푥 ) +⋯+ 푛푔(푥 ) ≥ 푔(푥 ) + 푔(푥 ) +⋯+ 푔(푥 )

We will prove this one: ln푘 + 푛푔(푥 ) ≥ 푔(푥 )

Or 푒 ( ) ≥ 푒 ( ) or 푒 ( ) ⋅ 푒 ≥ 푒 ( ) or 푘푓(푥 ) ≥ 푓(푥 )

4. Corollaries

Corollary 1: Let 풙ퟏ,풙ퟐ, … ,풙풏 be a positive real numbers such that:

풙ퟏ풙ퟐ … 풙풏 ≤ ퟏ and a positive constant 풌. A function 풇(풙) satisfied:

풇 (풙)풇(풙) ≥ ퟎ; 풇 (풙)풇(풙) ≥ 풇 (풙)ퟐ; 풇(풙) ≥ ퟎ and 풌풇(풙)풏 ퟏ ≥ ퟏ. Then we will have the following true

inequality: 풌풇(풙ퟏ)풇(풙ퟐ) …풇(풙풏) ≥ 풇(풙ퟏ풙ퟐ …풙풏)

Proof: The inequality is equivalent to:

ln푘 + ln푓(푥 ) + ln푓(푥 ) +⋯+ ln푓(푥 ) ≥ ln푓(푥 푥 …푥 )

Define 푔(푥) = ln푓(푥). These conditions: ( )( ) ≥ 0;푓 (푥)푓(푥) ≥ 푓 (푥) gives us:

푔 (푥) ≥ 0 and 푔 (푥) ≥ 0.

Rewrite the inequality as: ln푘 + 푔(푥 ) + 푔(푥 ) + ⋯+ 푔(푥 ) ≥ 푔(푥 푥 … 푥 )

Applying Jensen’s Inequality and AM-GM, since 푔 (푥) ≥ 0 and 푔 (푥) ≥ 0, we have:

푔(푥 ) + 푔(푥 ) + ⋯+ 푔(푥 ) ≥ 푛푔푥 + 푥 + ⋯+ 푥

푛≥ 푛푔 푥 푥 … 푥

Since 푥 푥 … 푥 ≤ 1, we have 푥 푥 …푥 ≥ 푥 푥 … 푥 . Hence, we have:

Page 29: ROMANIAN MATHEMATICAL MAGAZINE R.M.M. · pentru ∈ z∗, pe care am rezolvat-o așa cum urmează, am păstrat algoritmul și pentru calculul altor sume, unele putând fi calculate

S.S.M.ROMÂNIA - Filiala Mehedinți 2017

29 ROMANIAN MATHEMATICAL MAGAZINE NR. 18

푔 푥 푥 …푥 ≥ (푥 푥 …푥 )

We have to prove this: 푛 ln푘 + 푛푔(푥 ) + 푛푔(푥 ) + ⋯+ 푛푔(푥 ) ≥ 푔(푥 ) + 푔(푥 ) +⋯+ 푔(푥 )

Or: 푛 ln푘 + (푛 − 1)푔(푥 ) + (푛 − 1)푛푔(푥 ) + ⋯+ (푛 − 1)푔(푥 ) ≥ 0.

We will prove this: ln푘 + (푛 − 1)푔(푥 ) ≥ 0 with all 푛.

↔ 푒 ( ) ( ) ≥ 1 ↔ 푘 ⋅ 푓(푥 ) ≥ 1 which is true since we considered the condition.

GPF Inequalities isn’t the best way to prove a product of functions Inequality. It must be used with another methods or another inequalities. Because it’s only true in some range of variables. I will show you! We will start from this condition:

푓 (푥)푓(푥) ≥ 푓 (푥)

Corrolary 2: Let 풇(풙) be a polynomial: 풇(풙) = 풂ퟏ풙풏 + 풂ퟐ풙풏 ퟏ + ⋯+ 풂풏풙 + 풂풂 ퟏ

such that 풂ퟏ > 0, 푥 ≥ 0 and 풏 ≥ ퟎ. We will have 풇 (풙) ⋅ 풇(풙) isn’t always bigger than 풇 (풙)ퟐ with all 풙 ≥ ퟎ

Proof: We have 푓 (푥) = (푛푎 푥 + (푛 − 1)푎 푥 + ⋯+ 푎 ) = 푛 푎 푥 + 퐺(푥)

with 퐺(푥) is a polynomial after squaring 푓 (푥).

푓 (푥) ⋅ 푓(푥) = (푛(푛 − 1)푎 푥 + (푛 − 1)(푛 − 2)푎 푥 + ⋯+ 2푎 )(푎 푥 + 푎 푥 + ⋯+ 푎 푥)

with 퐻(푥) is a polynomial after expanding 푓 (푥) ⋅ 푓(푥). We always have 푛 ≥ 푛(푛 − 1). So we obtain: 푆 = 푓 (푥) ⋅ 푓(푥) − 푓 (푥) = −푎 푥 + 퐻(푥) − 퐺(푥)

Then we can conclude that 푆 isn’t always bigger than zero.That is the biggest problem of GPF Inequality. GPF can only help us on some interval 핀 that satisfied the condition. But GPF is the good way to find the best estimation for inequality. Next part of this article is the applications of GPF.

(Note: There are also solutions of following example using uvw method or Cauchy – Schwarz, etc).

5. Applications

As I said, GPF Inequality isn’t always the best way to prove a Inequality with product of functions. When using this theorem, you have to consider some cases.

In my opinion, GPF Inequality can be only a lemma. It isn’t strong enough to be a theorem. But I will show you some applications of this inequality. I hope you enjoy these examples!

1/(Michael Rozenberg) Let 풂,풃 and 풄 be non-negative numbers. Prove that:

ퟏ +ퟐ√ퟑ

풂ퟐ − 풂 + ퟏ 풃ퟐ −풃 + ퟏ 풄ퟐ − 풄 + ퟏ ≥ 풂ퟐ풃ퟐ풄ퟐ −풂풃풄 + ퟏ

Solution: This inequality can be found in [1]. This is a very hard inequality. The original one was a problem in USA TST 2006. I will show you the solution for this case:

Page 30: ROMANIAN MATHEMATICAL MAGAZINE R.M.M. · pentru ∈ z∗, pe care am rezolvat-o așa cum urmează, am păstrat algoritmul și pentru calculul altor sume, unele putând fi calculate

S.S.M.ROMÂNIA - Filiala Mehedinți 2017

30 ROMANIAN MATHEMATICAL MAGAZINE NR. 18

푎, 푏, 푐 ∈12

;1 + √3

2

Define 푓(푥) = 푥 − 푥 + 1. Since 푎, 푏, 푐 ∈ ; √ , we have 푥 ≥ and 푓 (푥) ≥ 0.

The inequality is equivalent to: 푘푓(푎)푓(푏)푓(푐) ≥ 푓(푎푏푐) with 푘 = 1 +√

.

We have: ( )( ) = ;푓 (푥)푓(푥) ≥ 푓 (푥) , 1 +

√(푥 − 푥 + 1) − (푥 − 푥 + 1) ≥ 0 (1)

(1) can be checked by computer. So we have: 푘푓(푥) 푔 ≥ 푓(푥 ).

Applying GPF1 Inequality, we obtain: 푘푓(푎)푓(푏)푓(푐) ≥ 푓(푎푏푐).

2/ (Unknown origin): Let 풂,풃, 풄 be real numbers. Determine the positive constant 풌 such that the following inequality is true: 풌 풂ퟐ + ퟏ 풃ퟐ + ퟏ 풄ퟐ + ퟏ ≥ 풂ퟐ풃ퟐ풄ퟐ + ퟏ.

Solution: For 푎, 푏, 푐 ∈ [0,1]. We have: 푓 (푥) = 2푥;푓 (푥) = 2. Hence, 푓 (푥) ⋅ 푓(푥) ≥ 푓 (푥) .

푘 ⋅ 푓(푥) ≥ 푓(푥 ) if and only if (푘 − 1)푥 + 3푘푥 + 3푘푥 + (푘 − 1) ≥ 0

Applying the first GPF Inequality and we will have 푘 = 1 is the best constant.

3/ (own): Let 풂,풃, 풄 be positive real numbers such that: 풂,풃, 풄 ∈ ퟏퟑ

; ퟐퟑ

. Prove that:

풂ퟑ − ퟐ풂ퟐ + 풂 + ퟏ 풃ퟑ − ퟐ풃ퟐ + 풃 + ퟏ 풄ퟑ − ퟐ풄ퟐ + 풄 + ퟏ ≤ 풂풃풄 − ퟐ 풂ퟐ풃ퟐ풄ퟐퟑ + √풂풃풄ퟑ + ퟏퟑ

Solution: We have: 푓(푥) = 푥 + 2푥 + 푥 + 1 ≥ 0

푓 (푥) = 3푥 − 4푥 + 1 = (3푥 − 1)(푥 − 1) ≤ 0, 푓 (푥) = 6푥 − 4 ≤ 0

Applying the fourth GPF Inequality, we have: 푓(푎)푓(푏)푓(푥) ≤ 푓 √푎푏푐

4/ (Holder’s Inequality): Let 풙ퟏ;풙ퟐ; … ; 풙풏 be nonnegative real numbers. Prove that:

ퟏ + 풙ퟏퟑ ퟏ + 풙ퟐퟑ … ퟏ + 풙풏ퟑ ≥ ퟏ + (풙ퟏ풙ퟐ … 풙풏)ퟑ풏 풏

Solution: This isn’t a good solution. GPF can only prove Holder’s inequality for only one cases!!! For Holder’s inequality, GPF is the weakest. This example will show you the weakness of GPF.We have the function 푓(푥) = 1 + 푥 , 푓 (푥) = 3푥 ≥ 0;푓 (푥) = 6푥 ≥ 0;푓 (푥) ⋅ 푓(푥) − 푓 (푥) =

=6푥 − 3푥 = 3푥(2 − 푥 ). Hence, we have 2 cases: If 푥 ≤ 2. We will have 푓 (푥) ⋅ 푓(푥) ≥ 푓 (푥) .

Applying GPF3 Inequality, we will have: 푓(푥 )푓(푥 ) … 푓(푥 ) ≥ 푓 푥 푥 … 푥 .

Actually, the inequality is true for all positive real numbers! As I said, GPF inequality isn’t worked in some cases! Maybe you want to use another method! In addition, GPF inequality is a good choice for

Page 31: ROMANIAN MATHEMATICAL MAGAZINE R.M.M. · pentru ∈ z∗, pe care am rezolvat-o așa cum urmează, am păstrat algoritmul și pentru calculul altor sume, unele putând fi calculate

S.S.M.ROMÂNIA - Filiala Mehedinți 2017

31 ROMANIAN MATHEMATICAL MAGAZINE NR. 18

a product of functions inequality with ranges of variables. When the inequality doesn’t have any condition, it’s hard to find the range of variables that GPF inequality is worked.

6. Problems for practicing

1. Let 풂,풃, 풄 be a positive real numbers such that: 풂,풃, 풄 ≥ √ퟑퟑ . Prove that:

풂ퟑ − ퟑ 풃ퟑ − ퟑ 풄ퟑ − ퟑ ≤ (풂풃풄 − ퟑ)ퟑ

2. Let 풂,풃, 풄 ∈ ퟏ풆

;ퟏ be positive real numbers, 풌 = 풆ퟑퟑ ퟏ ퟏ

풆ퟐ . Prove that: 풌풂ퟐ풃ퟐ풄ퟐ ≥ (풂풃풄)풂풃풄

3. Let 풂,풃, 풄 ∈ 풎, ퟏ풆

be positive real numbers with 풎 ≠ ퟏ be the root of the equation: 풙 = 풆ퟏ 풙ퟐ

ퟑ풙ퟐ ퟏ.

Let 풌 = 풆ퟑ풎 풎ퟐ ퟏ 퐥퐧풎. Prove that: 풌풂풂풃풃풄풄 ≥ (풂풃풄)풂풃풄

4. Let 풂,풃, 풄 ≥ ퟏ. Prove that: ퟑퟒퟏ + ퟏ

풂ퟏ + ퟏ

풃ퟏ + ퟏ

풄≥ ퟏ + ퟏ

풂풃풄

풂풃풄

References:

[1] http://www.artofproblemsolving.com/community/q1h1270987p6905229, [2] http://www.artofproblemsolving.com/community/c6h148824, [3] https://www.artofproblemsolving.com/wiki/index.php?title=Jensen%27s_Inequality [4] https://en.wikipedia.org/wiki/Jensen%27s_inequality, [5] http://www.artofproblemsolving.com/community/c6t243f6h1310196_easy_or_hard_rightorwrong [6] http://artofproblemsolving.com/community/c6t243f6h1313342_nice_one, [7] http://artofproblemsolving.com/community/c6t243f6h1313269_hard_one, [8] http://artofproblemsolving.com/community/c6t243f6h1313272_nice_one, [9] AoPS topic about USA TST 2006: http://www.artofproblemsolving.com/community/c6h148826p841252, [10] Vasile Cirtoaje, Algebraic Inequalities. GIL Publishing House, 2006, [11] Pham Kim Hung, Secrets in Inequalities, Volume 1. GIL Publishing House, 2007

FACTORIZAREA UNOR DETERMINANTI By Daniel Sitaru-Romania

Abstract: In this article we will show modalities of factorisation of some determinants and applications to them. Predarea noțiunii de determinant la clasa a XI – a este urmată de predarea proprietăților acestora.O clasă largă de aplicații la aceste lecții o constituie factorizarea determinanților ca o modalitate de a susține învățarea acestor proprietăți.Evidențierea modalităților de calcul al determinanților poate începe cu determinanții circulanți sau ciclici.Urmează prezentarea determinanților Vanderomonde și a celor pseudo – Vandermonde.Propunem câteva aplicații simple pentru rezolvarea ecuațiilor cu determinanți și câteva legături cu celelalte capitole din programa școlară. Propoziția 1: Dacă 푎, 푏, 푐 ∈ ℝ atunci:

퐷 =푎 푐 푏푏 푎 푐푐 푏 푎

=12

(푎 + 푏 + 푐)[(푎 − 푏) + (푏 − 푐) + (푎 − 푐) ]

Page 32: ROMANIAN MATHEMATICAL MAGAZINE R.M.M. · pentru ∈ z∗, pe care am rezolvat-o așa cum urmează, am păstrat algoritmul și pentru calculul altor sume, unele putând fi calculate

S.S.M.ROMÂNIA - Filiala Mehedinți 2017

32 ROMANIAN MATHEMATICAL MAGAZINE NR. 18

퐷 =푎 푏 푐푏 푐 푎푐 푎 푏

= −12

(푎 + 푏 + 푐)[(푎 − 푏) + (푏 − 푐) + (푎 − 푐) ]

Demonstrație:

퐷 =푎 푐 푏푏 푎 푐푐 푏 푎

= (푎 + 푏 + 푐)1 1 1푏 푎 푐푐 푏 푎

= (푎 + 푏 + 푐)1 0 0푏 푎 − 푏 푐 − 푏푐 푏 − 푐 푎 − 푐

=

= (푎 + 푏 + 푐)[(푎 − 푏)(푎 − 푐) − (푐 − 푏)(푏 − 푐)] == (푎 + 푏 + 푐)(푎 − 푎푐 − 푎푏 + 푏푐 − 푏푐 + 푐 + 푏 − 푏푐) =

=12

(푎 + 푏 + 푐)[(푎 − 푏) + (푏 − 푐) + (푐 − 푎) ]

퐷 =푎 푏 푐푏 푐 푎푐 푎 푏

= (푎 + 푏 + 푐)1 1 1푏 푐 푎푐 푎 푏

= (푎 + 푏 + 푐)1 0 0푏 푐 − 푏 푎 − 푏푐 푎 − 푐 푏 − 푐

=

= (푎 + 푏 + 푐) 푐 − 푏 푎 − 푏푎 − 푐 푏 − 푐 = (푎 + 푏 + 푐)[(푐 − 푏)(푏 − 푐) − (푎 − 푐)(푎 − 푏)] =

= −12

(푎 + 푏 + 푐)[(푎 − 푏) + (푏 − 푐) + (푐 − 푎) ]

Observăm că dacă 푎 + 푏 + 푐 > 0;푎 ≠ 푏 ≠ 푐 ≠ 푎 atunci 퐷 > 0;퐷 < 0. De asemenea, 푎 + 푏 + 푐 =0 implică 퐷 = 퐷 = 0. Propoziția 2: Dacă 푎, 푏, 푐 ∈ ℝ atunci:

퐷 = 푉푎푛푑(푎, 푏, 푐) =1 1 1푎 푏 푐푎 푏 푐

= (푏 − 푎)(푐 − 푎)(푐 − 푏)

퐷 =1 1 1푎 푏 푐푎 푏 푐

= (푎 + 푏 + 푐)(푏 − 푎)(푐 − 푎)(푐 − 푏)

Demonstrație:

퐷 =1 1 1푎 푏 푐푎 푏 푐

=1 0 0푎 푏 − 푎 푐 − 푎푎 푏 − 푎 푐 − 푎

= (푏 − 푎)(푐 − 푎 )− (푐 − 푎)(푏 − 푎 ) =

= (푏 − 푎)(푐 − 푎)(푐 + 푎푐 + 푎 )− (푐 − 푎)(푏 − 푎)(푏 + 푏푎 + 푎 ) = = (푏 − 푎)(푐 − 푎)(푐 + 푎푐 + 푎 − 푏 − 푎푏 − 푎 ) =

= (푏 − 푎)(푐 − 푎)[푐(푎 + 푏 + 푐) − 푏(푎 + 푏 + 푐)] == (푏 − 푎)(푐 − 푎)(푐 − 푏)(푎 + 푏 + 푐)

Observăm că dacă 0 < 푎 ≤ 푏 ≤ 푐 atunci 퐷 ≥ 0;퐷 ≥ 0 și 퐷 = (푎 + 푏 + 푐)퐷 . Propoziția 3: Dacă 푎, 푏, 푐 ∈ ℝ∗ atunci:

퐷 =

1 1 1푎 푏 푐1푎

1푏

1푐

=(푏 − 푎)(푐 − 푎)(푐 − 푏)

푎푏푐

퐷 =

1 1 1푎 푏 푐1푎

1푏

1푐

=(푏 − 푎)(푐 − 푎)(푐 − 푏)(푎푏 + 푎푐 + 푏푐)

푎 푏 푐

Demonstrație:

Page 33: ROMANIAN MATHEMATICAL MAGAZINE R.M.M. · pentru ∈ z∗, pe care am rezolvat-o așa cum urmează, am păstrat algoritmul și pentru calculul altor sume, unele putând fi calculate

S.S.M.ROMÂNIA - Filiala Mehedinți 2017

33 ROMANIAN MATHEMATICAL MAGAZINE NR. 18

퐷 =

1 1 1푎 푏 푐1푎

1푏

1푐

=

1 0 0푎 푏 − 푎 푐 − 푎1푎

1푏−

1푎

1푐−

1푎

=

=푏 − 푎 푐 − 푎1푏−

1푎

1푐−

1푎

=(푏 − 푎)(푎 − 푐)

푎푐−

(푐 − 푎)(푎 − 푏)푎푏

=

= (푏 − 푎)(푎 − 푐)1푎푐

−1푎푏

=(푏 − 푎)(푎 − 푐)(푏 − 푐)

푎푏푐=

(푏 − 푎)(푐 − 푎)(푐 − 푏)푎푏푐

퐷 =

1 1 1푎 푏 푐1푎

1푏

1푐

=

1 0 0푎 푏 − 푎 푐 − 푎1푎

1푏

−1푎

1푐−

1푎

=

=푏 − 푎 푐 − 푎푎 − 푏푎 푏

푎 − 푐푎 푐

=푏 − 푎 푐 − 푎

(푎 − 푏)(푎 + 푏)푎 푏

(푎 − 푐)(푎 + 푐)푎 푐

=

=(푏 − 푎)(푎 − 푐)(푎 + 푐)

푎 푐−

(푐 − 푎)(푎 − 푏)(푎 + 푏)푎 푏

=

= (푏 − 푎)(푎 − 푐)푎 + 푐푎 푐

−푎 + 푏푏

=(푏 − 푎)(푎 − 푐)

푎 푏 푐(푏 푎 + 푏 푐 − 푎푐 − 푏푐 ) =

=(푏 − 푎)(푎 − 푐)

푎 푏 푐푎(푏 − 푐 ) + 푏푐(푏 − 푐) =

(푏 − 푎)(푎 − 푐)(푏 − 푐)푎 푏 푐

(푎푏 + 푎푐 + 푏푐) =

=(푏 − 푎)(푐 − 푎)(푐 − 푏)(푎푏 + 푎푐 + 푏푐)

푎 푏 푐

Observăm că 퐷 = (푎푏 + 푎푐 + 푏푐) . Lecția poate fi continuată cu rezolvarea unor ecuații cu determinanți în care putem aplica propozițiile anterioare sau putem repeta raționamentele pentru cazurile concrete ale ecuațiilor care se cer a fi rezolvate. Aplicații:

Să se rezolve ecuațiile: 1. 1 1 1

푥 + 1 푥 + 2 3(푥 + 1) (푥 + 2) 27

= 0, 2. 1 1 13 푥 + 4 푥 + 5 = 0

3. 1 1 1

푥+ 2 푥 + 3 2

( ) ( )

= 0. Dacă 푎, 푏, 푐 sunt lungimile laturilor unui triunghi atunci din teorema

sinusurilor: 푎 = 2푅 sin퐴푏 = 2푅 sin퐵푐 = 2푅 sin 퐶

Prin înlocuire succesivă în formulele lui 퐷 ,퐷 ,퐷 ,퐷 ,퐷 ,퐷 obținem:

sin퐴 sin퐶 sin퐵sin퐵 sin퐴 sin 퐶sin 퐶 sin퐵 sin퐴

=12

(sin퐴 + sin퐵 + sin퐶) (sin퐴 − sin퐵)

sin퐴 sin퐵 sin퐶sin퐵 sin퐶 sin퐴sin 퐶 sin퐴 sin퐵

= −12

(sin퐴 + sin퐵 + sin퐶) (sin퐴 − sin퐵)

1 1 1

sin퐴 sin퐵 sin퐶sin 퐴 sin 퐵 sin 퐶

= (sin퐵 − sin퐴)(sin퐶 − sin퐴)(sin퐶 − sin퐵)

Page 34: ROMANIAN MATHEMATICAL MAGAZINE R.M.M. · pentru ∈ z∗, pe care am rezolvat-o așa cum urmează, am păstrat algoritmul și pentru calculul altor sume, unele putând fi calculate

S.S.M.ROMÂNIA - Filiala Mehedinți 2017

34 ROMANIAN MATHEMATICAL MAGAZINE NR. 18

1 1 1

sin퐴 sin퐵 sin 퐶sin 퐴 sin 퐵 sin 퐶

= (sin퐵 − sin퐴)(sin 퐶 − sin퐴)(sin 퐶 − sin퐵) sin퐴

1 1 1sin퐴 sin퐵 sin 퐶

1sin퐴

1sin퐵

1sin 퐶

=(sin퐵 − sin퐴)(sin퐶 − sin퐴)(sin 퐶 − sin퐵)

sin퐴 sin퐵 sin 퐶

1 1 1sin퐴 sin퐵 sin퐶

1sin 퐴

1sin 퐵

1sin 퐶

=(sin퐵 − sin퐴)(sin퐶 − sin퐴)(sin퐶 − sin퐵)∑ sin퐴 sin퐵

sin 퐴 sin 퐵 sin 퐶

Bibliografie: [1] Daniel Sitaru, Math Phenomenon, Editura Paralela 45, Pitești, 2016 [2] Daniel Sitaru, Radu Gologan, Leonard Giugiuc, 300 Romanian Mathematical Challenges, Editura Paralela 45, Pitești 2016 [3] Daniel Sitaru, Claudia Nănuți, Diana Trăilescu, Leonard Giugiuc, Inequalities, Editura Ecko-Print, Dr. Tr. Severin, 2015 [4] Colecția Gazeta Matematică seria A și B. [5] Colecția Didactica Matematică

ON SOME TRIANGLE INEQUALITIES

By Nguyen Viet Hung

High School For Gifted Students, Hanoi University of Science, Vietnam Email address: [email protected]

Abstract: In this article we will use two known triangle inequalities to give some other results. 1. Introduction: The following two inequalities have been very popular.

≥ 푦푧 cos퐴 + 푧푥 cos퐵 + 푥푦 cos 퐶 (1) , ≥ 푦푧 sin + 푧푥 sin + 푥푦 sin (2) where 퐴,퐵,퐶 are three angles of a triangle and 푥,푦, 푧 are any real numbers. We can rewrite these two inequalities as:

푥 + 푦 + 푧 ≥ 푦푧 + 푧푥 + 푥푦 (3)

≥ 푦푧 ( )( ) + 푧푥 ( )( ) + 푥푦 ( )( ) (4)

There are two popular proofs for (1) as follows: Using properties of vector. Let 퐼 be the incenter of triangle 퐴퐵퐶, and let 푋,푌,푍 be respectively feets of perpendicular lines from 퐼 to sides 퐵퐶,퐶퐴,퐴퐵. By full expanding the following selfevident

inequality: 푥퐼푋⃗ + 푦퐼푌⃗ + 푧퐼푍⃗ ≥ 0 we get the desired inequality. The equality occurs if and only if: 푥퐼푋⃗ + 푦퐼푌⃗ + 푧퐼푍⃗ = 0⃗. On the other hand, according to the porcupine theorem, we have: 푎퐼푋⃗ + 푏퐼푌⃗ + 푐퐼푍⃗ = 0⃗. Thus the necessary and sufficient conditions such that the equality occurs as: = = . Using algebraic techniques. The inequality (1) is equivalent to

2푦푧 cos퐴 + 2푧푥 cos퐵 − 2푥푦 cos(퐴 + 퐵) ≤ 푥 + 푦 + 푧 , 2푦푧 cos퐴 + 2푧푥 cos퐵 − 2푥푦 cos 퐴 cos퐵 ≤ 푥 + 푦 + 푧 − 2푥푦 sin퐴 sin퐵,

2푦푧 cos퐴 + 2푧푥 cos퐵 − 2푥푦 cos 퐴 cos퐵 + 푦 sin 퐴 + 푥 sin 퐵≤ 푥 + 푦 + 푧 + (푦 sin퐴 − 푥 sin퐵)

2푦푧 cos 퐴+ 2푧푥 cos퐵 − 2푥푦 cos퐴 cos퐵 ≤ 푥 cos 퐵 + 푦 cos 퐴+ 푧 + (푦 sin퐴− 푥 sin퐵) 2푧(푦 cos퐴 + 푥 cos퐵) ≤ (푦 cos 퐴+ 푥 cos퐵) + 푧 + (푦 sin퐴 − 푥 sin퐵) ,

(푦 cos퐴 + 푥 cos퐵 − 푧) + (푦 sin퐴 − 푥 sin퐵) ≥ 0

Page 35: ROMANIAN MATHEMATICAL MAGAZINE R.M.M. · pentru ∈ z∗, pe care am rezolvat-o așa cum urmează, am păstrat algoritmul și pentru calculul altor sume, unele putând fi calculate

S.S.M.ROMÂNIA - Filiala Mehedinți 2017

35 ROMANIAN MATHEMATICAL MAGAZINE NR. 18

Which is obviously true. The equility occurs if: = ,

푧 = 푦 cos퐴 + 푥 cos퐵

To do more clearly, we set = = 푘. Then 푥 = 푘. sin퐴 ,푦 = 푘. sin퐵 and 푧 = 푘(sin퐴 cos퐵 +sin퐵cos퐴=푘.sin퐶. So, the conditions above are equivalent to: 푥sin퐴=푦sin퐵=푧sin퐶. This is necessary and sufficient conditions to the equality happens. Applying (1) for a triangle which has three angles , , we obtain (2). Remark 1. From the second proof we observe that for any angles 훼,훽,훾 (the are not necessary three angles of a triangle) such that 훼 + 훽 + 훾 = 휋, the inequality below is also true (for all real numbers 푥,푦, 푧) : 푥 + 푦 + 푧 ≥ 2푦푧 cos훼 + 2푧푥 cos훽 + 2푥푦 cos훾 (5) 2. Some results: In all problems below, we use known notations of triangle 퐴퐵퐶 and note that 푆 denotes its area. In (3) we replace (푥,푦, 푧) by (√푎,√푏,√푐) to yield:

푎 + 푏 + 푐 ≥푏 + 푐 − 푎

√푎푐+푐 + 푎 − 푏

√푐푎+푎 + 푏 − 푐

√푎푏

This inequality has equivalent as:

풂ퟐ

√풃풄+

풃ퟐ

√풄풂+

풄ퟐ

√풂풃+ 풂 + 풃 + 풄 ≥

풂ퟑ

풃+

풃ퟑ

풄+

풄ퟑ

풂+

풃ퟑ

풂+

풄ퟑ

풃+

풂ퟑ

풄,

퐜퐨퐭푨√풃풄

+퐜퐨퐭푩√풄풂

+퐜퐨퐭푪√풂풃

≤ퟏퟐ풓

If we substitue (푎 , 푏 , 푐 ) for (푥,푦, 푧), we get 푎 + 푏 + 푐 ≥ 푏푐(푏 + 푐 − 푎 ) + 푐푎(푐 + 푎 − 푏 ) + 푎푏(푎 + 푏 − 푐 )

or 푎 + 푏 + 푐 + 푎푏푐(푎 + 푏 + 푐) ≥ 푎푏(푎 + 푏 ) + 푏푐(푏 + 푐 ) + 푐푎(푐 + 푎 )

This is Schur’s inequality of fourth degree that is well-known. We replace (푥,푦, 푧) in (1) by

√,√

,√

(and for the acute triangle 퐴퐵퐶) we have ퟏ

풔 − 풂+

ퟏ풔 − 풃

+ퟏ

풔 − 풄≥

ퟐ(풔 − 풃)(풔 − 풄)풄풚풄

퐜퐨퐬푨 ≥ퟒ풂퐜퐨퐬푨+

ퟒ풃퐜퐨퐬푩+

ퟒ풄퐜퐨퐬 푪

This can be rewritten by other forms as follows: 풓풂 풓풃 풓풄ퟒ푺

≥ 퐜퐨퐬푨풂

+ 퐜퐨퐬푩풃

+ 퐜퐨퐬 푪풄

. When (푥,푦, 푧) is replaced by 푟 , 푟 , 푟 (the triangle 퐴퐵퐶 is also acute) we obtain 풓풂 + 풓풃 + 풓풄

ퟐ≥ 풓풃풓풄 퐜퐨퐬푨 + 풓풄풓풂 퐜퐨퐬푩 + 풓풂풓풃 퐜퐨퐬 푪 ≥ 풉풂 퐜퐨퐬 푨+ 풉풃 퐜퐨퐬푩 + 풉풄 퐜퐨퐬푪

Which is the other form of one of the results above.

To be continue, we replace again (푥,푦, 푧) in (1) by , , to get: 풓풃풓풄풓풂

+풓풄풓풂풓풃

+풓풂풓풃풓풄

≥ ퟐ(풓풂 퐜퐨퐬푨 + 풓풃 퐜퐨퐬푩 + 풓풄 퐜퐨퐬푪)

In (1), (2) we replace respectively (푥,푦, 푧) by , , and , , and using: ퟏ풉풂

+ퟏ풉풃

+ퟏ풉풄

=ퟏ풓풂

+ퟏ풓풃

+ퟏ풓풄

=ퟏ풓

we obtain the following results: 퐜퐨퐬푨풉풃풉풄

+ 퐜퐨퐬 푩풉풄풉풂

+ 퐜퐨퐬 푪풉풂풉풃

≤ ퟏퟐ풓

, 퐬퐢퐧푨ퟐ풓풃풓풄

+퐬퐢퐧푩ퟐ풓풄풓풂

+퐬퐢퐧푪ퟐ풓풂풓풃

≤ ퟏퟐ풓

.

Chosing 푥 = ,푦 = , 푧 = and substitue it into (2) gives:

퐬퐢퐧 푨ퟐ

(풔 − 풃)(풔 − 풄) +퐬퐢퐧 푩

ퟐ(풔 − 풄)(풔 − 풂) +

퐬퐢퐧 푪ퟐ

(풔 − 풂)(풔 − 풃) ≤ퟏퟐ

ퟏ(풔 − 풂)ퟐ +

ퟏ(풔 − 풃)ퟐ +

ퟏ(풔 − 풄)ퟐ ,

which is equivalent to:

Page 36: ROMANIAN MATHEMATICAL MAGAZINE R.M.M. · pentru ∈ z∗, pe care am rezolvat-o așa cum urmează, am păstrat algoritmul și pentru calculul altor sume, unele putând fi calculate

S.S.M.ROMÂNIA - Filiala Mehedinți 2017

36 ROMANIAN MATHEMATICAL MAGAZINE NR. 18

(풔 − 풂) 퐬퐢퐧푨ퟐ

+ (풔 − 풃) 퐬퐢퐧푩ퟐ

+ (풔 − 풄)퐬퐢퐧푪ퟐ≤

≤(풔 − 풂)(풔 − 풃)(풔 − 풄)

ퟐퟏ

(풔 − 풂)ퟐ +ퟏ

(풔 − 풃)ퟐ +ퟏ

(풔 − 풄)ퟐ

On the other hand, it’s easy to show that: (푠 − 푎)(푠 − 푏)(푠 − 푐) ≤ We infer that:

(풔 − 풂) 퐬퐢퐧푨ퟐ

+ (풔 − 풃) 퐬퐢퐧푩ퟐ

+ (풔 − 풄) 퐬퐢퐧푪ퟐ≤풂풃풄ퟏퟔ

ퟏ(풔 − 풂)ퟐ +

ퟏ(풔 − 풃)ퟐ +

ퟏ(풔 − 풄)ퟐ .

Form (3) chosing 푥 = ,푦 = , 푧 = yields: 푎

푏 + 푐+

푏푐 + 푎

+푐

푎 + 푏≥

푏 + 푐 − 푎(푏 + 푎)(푐 + 푎) +

푐 + 푎 − 푏(푐 + 푏)(푎 + 푏) +

푎 + 푏 − 푐(푎 + 푐)(푏 + 푐).

This inequality has equivalent forms as follows: 퐜퐨퐭푨

(풂 + 풃)(풂 + 풄) +퐜퐨퐭푩

(풃+ 풄)(풃 + 풂) +퐜퐨퐭 푪

(풄 + 풂)(풄+ 풃) ≤ퟏퟒ푺

풂풃 + 풄

ퟐ+

풃풄 + 풂

+풄

풂 + 풃

ퟐ.

Chosing 푥 = ,푦 = , 푧 = and replace it into (4) we get:

(푠 − 푏)(푠 − 푐)(푎 + 푏)(푎 + 푐) +

(푠 − 푐)(푠 − 푎)(푏 + 푐)(푏 + 푎) +

(푠 − 푎)(푠 − 푏)(푐 + 푎)(푐 + 푏) ≤

12

푎푏 + 푐

+푏

푐 + 푎+

푐푎 + 푏

.

In (4) we replace (푥,푦, 푧) by √푠 − 푎,√푠 − 푏,√푠 − 푐 then (풔 − 풃)(풔 − 풄)

√풃풄+

(풔 − 풄)(풔 − 풂)

√풄풂+

(풔 − 풂)(풔 − 풃)

√풂풃≤풔ퟐ

.

This inequality has equivalent forms as follows

√풃풄 퐬퐢퐧ퟐ푨ퟐ

+ √풄풂 퐬퐢퐧ퟐ푩ퟐ

+ √풂풃퐬퐢퐧ퟐ푪ퟐ≤풂 + 풃 + 풄

ퟒ.

Chosing again 푥 =( )( )

,푦 =( )( )

, 푧 =( )( )

and then

respectively substitue it into (1), (4) and note that: 1

(푠 − 푏)(푠 − 푐) +1

(푠 − 푐)(푠 − 푎) +1

(푠 − 푎)(푠 − 푏) =1푟

we have the following results 퐜퐨퐬푨

(풔 − 풂) (풔 − 풃)(풔 − 풄)+

퐜퐨퐬푩(풔 − 풃) (풔 − 풄)(풔 − 풂)

+퐜퐨퐬 푪

(풔 − 풄) (풔 − 풂)(풔 − 풃)≤

ퟏퟐ풓ퟐ

,

ퟏ(풔 − 풂)√풃풄

+ퟏ

(풔 − 풃)√풄풂+

ퟏ(풔 − 풄)√풂풃

≤ퟏퟐ풓ퟐ

.

Now we chose 푥 = ,푦 = , 푧 = and substitue it into (2)

to get: 퐬퐢퐧 푩

ퟐ퐬퐢퐧 푪

퐬퐢퐧 푨ퟐ

+퐬퐢퐧 푪

ퟐ퐬퐢퐧 푨

퐬퐢퐧 푩ퟐ

+퐬퐢퐧 푨

ퟐ퐬퐢퐧 푩

퐬퐢퐧 푪ퟐ

≥ ퟐ퐬퐢퐧ퟐ푨ퟐ

+ ퟐ퐬퐢퐧ퟐ푩ퟐ

+ ퟐ퐬퐢퐧ퟐ푪ퟐ

When replace (푥,푦, 푧) by , , into (4) we have:

12푠 − 푎푎

+푠 − 푏푏

+푠 − 푐푐

≥(푠 − 푏)(푠 − 푐)

푏푐+

(푠 − 푐)(푠 − 푎)푐푎

+(푠 − 푎)(푠 − 푏)

푎푏

This is equivalent to: 풔 − 풂풂

+풔 − 풃풃

+풔 − 풄풄

≥ ퟐ 퐬퐢퐧ퟐ푨ퟐ

+ ퟐ 퐬퐢퐧ퟐ푩ퟐ

+ ퟐ 퐬퐢퐧ퟐ푪ퟐ

Page 37: ROMANIAN MATHEMATICAL MAGAZINE R.M.M. · pentru ∈ z∗, pe care am rezolvat-o așa cum urmează, am păstrat algoritmul și pentru calculul altor sume, unele putând fi calculate

S.S.M.ROMÂNIA - Filiala Mehedinți 2017

37 ROMANIAN MATHEMATICAL MAGAZINE NR. 18

or (풂 + 풃+ 풄) ퟏ풂

+ ퟏ풃

+ ퟏ풄

+ ퟐ(퐜퐨퐬푨 + 퐜퐨퐬푩 + 퐜퐨퐬푪) ≥ ퟏퟐ,

or (풂 + 풃+ 풄) ퟏ풂

+ ퟏ풃

+ ퟏ풄

+ ퟐ풓푹≥ ퟏퟎ.

Then next, we also replace (푥,푦, 푧) by √푏푐,√푐푎,√푎푏 into (4) then obtain

풂 (풔 − 풃)(풔 − 풄) + 풃 (풔 − 풄)(풔 − 풂) + 풄 (풔 − 풂)(풔 − 풃) ≤풂풃+ 풃풄 + 풄풂

ퟐ.

In (4) we chose 푥 = ,푦 = , 푧 = then

풂풔 − 풂

ퟐ+

풃풔 − 풃

+풄

풔 − 풄

ퟐ≥

퐬퐢퐧 푨ퟐ

+ퟐ

퐬퐢퐧 푩ퟐ

+ퟐ

퐬퐢퐧 푪ퟐ

≥ ퟏퟐ

Now we replace again (푥,푦, 푧) by (sin퐴 , sin퐵 , sin 퐶 ), where 퐴,퐵,퐶 are three angles of any triangle, into (1), we find that

퐬퐢퐧 푩 퐬퐢퐧푪 퐜퐨퐬푨 ≤ퟏퟐ퐬퐢퐧ퟐ푨 + 퐬퐢퐧ퟐ 푩 + 퐬퐢퐧ퟐ 푪

풄풚풄

Dividing both sides of this inequality by sin 퐴 sin퐵 sin 퐶 , we get cos 퐴sin퐴

+cos퐵sin퐵

+cos 퐶sin 퐶

≤12

sin퐴sin퐵 sin 퐶

+sin퐵

sin 퐶 sin퐴 +sin 퐶

sin퐴 sin퐵

Note that: sin퐴

sin퐵 sin 퐶=

sin(퐵 + 퐶 )sin퐵 sin퐶

=sin퐵 cos퐶 + cos퐵 sin 퐶

sin퐵 sin 퐶= cot퐵 + cot 퐶

Similarly: = cot퐶 + cot퐴 , = cot퐴 + cot퐵 Thus, we have a result: For any two triangles 퐴퐵퐶 and 퐴 퐵 퐶 , the following inequality holds

퐜퐨퐭푨 + 퐜퐨퐭푩+ 퐜퐨퐭푪 ≥퐜퐨퐬푨퐬퐢퐧푨

+퐜퐨퐬푩퐬퐢퐧푩

+퐜퐨퐬 푪퐬퐢퐧푪

. If we replace again (푥,푦, 푧) by (푎 , 푏 , 푐 ) into (1) and using formula 푎 + 푏 + 푐 = 4푆 (cot 퐴 + cot퐵 + cot 퐶 ) then to obtain:

2푆 (cot퐴 + cot퐵 + cot퐶 ) ≥ 푏 푐 cos퐴 + 푐 푎 cos퐵 + 푎 푏 cos퐶. This is equivalent to:

퐜퐨퐭푨 + 퐜퐨퐭푩 + 퐜퐨퐭푪 ≥퐜퐨퐬푨퐬퐢퐧 푨

+퐜퐨퐬푩퐬퐢퐧푩

+퐜퐨퐬푪퐬퐢퐧푪

. Thus, we get again the above result. We consider any point 푃 in triangle 퐴퐵퐶 and let 푃푋,푃푌,푃푍 be internal bisectors of ∠퐵푃퐶,∠퐶푃퐴,∠퐴푃퐵, respectively we set ∠퐵푃퐶 = 2훼,∠퐶푃퐴 = 2훽,∠퐴푃퐵 = 2훾. Using the known formulas about the length of bisectors in a triangle, we have

푃푋 =2푃퐵.푃퐶푃퐵 + 푃퐶

cos훼 ,푃푌 =2푃퐶.푃퐴푃퐶 + 푃퐴

cos훽 ,푃푍 =2푃퐴.푃퐵푃퐴 + 푃퐵

cos 훾

or

2 cos 훼 = 푃푋1푃퐵

+1푃퐶

, 2 cos 훽 = 푃푌1푃퐶

+1푃퐴

, 2 cos 훾 = 푃푍1푃퐴

+1푃퐵

. Applying (5) for 훼,훽, 훾 which are determined above, we have:

푷푿ퟏ푷푩

+ퟏ푷푪

풚풛+ 푷풀ퟏ푷푪

+ퟏ푷푨

풛풙 + 푷풁ퟏ푷푨

+ퟏ푷푩

풙풚 ≤ 풙ퟐ + 풚ퟐ + 풛ퟐ .

We continue take 푥 = √푃퐴,푦 = √푃퐵, 푧 = √푃퐶 then to get

푷푿√푷푪√푷푩

+√푷푩√푷푪

+푷풀√푷푨√푷푪

+√푷푪√푷푨

+푷풁√푷푩√푷푨

+√푷푨√푷푩

≤ 푷푨 +푷푩+ 푷푪.

which implies that: 푃퐴+ 푃퐵 + 푃퐶 ≥ 2(푃푋 + 푃푌 + 푃푍) ≥ 2(푃푃 + 푃푃 + 푃푃 ) where 푃 ,푃 ,푃 are feet of perpendicular lines from 푃 to the sides 퐵퐶,퐶퐴,퐴퐵, respectively. We have just received a result which is stronger than Erdos-Mordell inequality.

Page 38: ROMANIAN MATHEMATICAL MAGAZINE R.M.M. · pentru ∈ z∗, pe care am rezolvat-o așa cum urmează, am păstrat algoritmul și pentru calculul altor sume, unele putând fi calculate

S.S.M.ROMÂNIA - Filiala Mehedinți 2017

38 ROMANIAN MATHEMATICAL MAGAZINE NR. 18

We have know that if 푎, 푏, 푐 are side-lengths of a triangle then there exist positive real numbers 푢, 푣,푤 such that: 푎 = 푣 +푤, 푏 = 푤 + 푢, 푐 = 푢 + 푣 which is called Ravi’s substitutions. Using this substitutions, (3) can be written as:

푥 + 푦 + 푧 ≥ 푦푧 ⋅(푤 + 푢) + (푦 + 푣) − (푣 + 푤)

(푤 + 푢)(푢 + 푣) =

= 푦푧 ⋅2(푢 + 푢푣 + 푢푤 − 푣푤)

(푤 + 푢)(푢 + 푣) = 푦푧 ⋅2(푢 + 푣)(푢 +푤) − 4푣푤

(푢 + 푣)(푢 + 푤) =

= 푦푧 2 −4푣푤

(푢 + 푣)(푢 +푤)

It follows that:

푦푧푣푤

(푢 + 푣)(푢 +푤)+ 푧푥

푤푢(푣 +푤)(푣 + 푢) + 푥푦

푢푣(푤 + 푢)(푤 + 푢)

≥2(푥푦 + 푦푧 + 푧푥) − (푥 + 푦 + 푧 )

4=

(푥 + 푦 + 푧) − 2(푥 + 푦 + 푧 )4

Note that the equality occurs if: = = . We choose 푥 = 푦 = 푧 to get the known result: 푣푤

(푢 + 푐)(푢 +푤) +푤푢

(푣 + 푤)(푣 + 푢) +푢푣

(푤 + 푢)(푤 + 푣) ≥34

When choosing 푥 = 1,푦 = , 푧 = then we obtain: 푣푤

(푢 + 푣)(푢 + 푤) +2푤푢

(푣 +푤)(푣 + 푢) +3푢푣

(푤 + 푢)(푤 + 푣) >2324

(because the equality does not occur). Applying (3) for a triangle which has three side-lenghts 푚 ,푚 ,푚 we get:

푦푧5푎 − 푏 − 푐

푚 푚+ 푧푥

5푏 − 푐 − 푎푚 푚

+ 푥푦5푐 − 푎 − 푏

푚 푚≤ 4(푥 + 푦 + 푧 )

A simple consequent of this result as: ퟓ풂ퟐ −풃ퟐ − 풄ퟐ

풎풃풎풄+ퟓ풃ퟐ − 풄ퟐ −풂ퟐ

풎풄풎풂+ퟓ풄ퟐ −풂ퟐ − 풃ퟐ

풎풂풎풃≤ ퟏퟐ.

In (3) we replace (푥,푦, 푧) by (푥푎,푦푏, 푧푐) to give 푥 푎 + 푦 푏 + 푧 푐 ≥ 푦푧(푏 + 푐 − 푎 ) + 푧푥(푐 + 푎 − 푏 ) + 푥푦(푎 + 푏 − 푐 )

which is equivalent to: 풂ퟐ 풙ퟐ + ퟐ풚풛 + 풃ퟐ 풚ퟐ + ퟐ풛풙 + 풄ퟐ 풛ퟐ + ퟐ풙풚 ≥ 풂ퟐ + 풃ퟐ + 풄ퟐ (풙풚 + 풚풛+ 풛풙).

The equality holds for 푥 = 푦 = 푧. In (4) we replace (푥,푦, 푧) by 푥√푎,푦√푏, 푧√푐 to yield

푦푧 (푠 − 푏)(푠 − 푐) + 푧푥 (푠 − 푐)(푠 − 푎) + 푥푦 (푠 − 푎)(푠 − 푏) ≤푥 푎 + 푦 푏 + 푧 푐

2.

From here, choosing again (푥,푦, 푧) = , , we have:

(풔 − 풃)(풔 − 풄)풃풄

+(풔 − 풄)(풔 − 풂)

풄풂+

(풔 − 풂)(풔 − 풃)풂풃

≤ퟏퟐ

ퟏ풂

+ퟏ풃

+ퟏ풄

. By similar ways, you can also establish new results for yourself. To finish this article, we will give some problems that can be solved by using elementary inequality above. 3. Proposed problems: Exercise 1 (Murray-Klamkin). Let 퐴퐵퐶 be a triangle and let 푥,푦, 푧 be any real numbers. Prove that

풙ퟐ + 풚ퟐ + 풛ퟐ ≥ ퟐ(−ퟏ)풏 ퟏ(풚풛퐜퐨퐬 풏푨 + 풛풙 퐜퐨퐬풏푩 + 풙풚퐜퐨퐬풏푪) where 푛 is a natural number. The equality occurs if: = = . Hint: The desired inequality is equivalent to:

푥 + (−1) (푦 cos 푛퐶 + 푧 cos 푛퐵) + (푦 sin 푛퐶 − 푧 sin 푛퐵) ≥ 0. Exercise 2. Let 퐴퐵퐶 be a triangle and let 푛 be a natural number. Prove that:

Page 39: ROMANIAN MATHEMATICAL MAGAZINE R.M.M. · pentru ∈ z∗, pe care am rezolvat-o așa cum urmează, am păstrat algoritmul și pentru calculul altor sume, unele putând fi calculate

S.S.M.ROMÂNIA - Filiala Mehedinți 2017

39 ROMANIAN MATHEMATICAL MAGAZINE NR. 18

a. cos(2푛퐴) + cos(2푛퐵) + cos(2푛퐶) ≥ −

b. cos(2푛 + 1)퐴 + cos(2푛 + 1)퐵 + cos(2푛 + 1)퐶 ≤ Exercise 3. Prove that all any triangle 퐴퐵퐶, then:

√3 cos퐴 + 2 cos퐵 + 2√3 cos퐶 ≤ 4. Exercise 4. Let 퐴퐵퐶 be a triangle and let 푥,푦, 푧 be any real numbers. Prove that

푎푥 + 푏푦 + 푐푧4푆

≥푥푦푎푏

+푦푧푏푐

+푧푥푐푎

. Exercise 5. Prove that for all any triangle 퐴퐵퐶, then:

푎 + 푏 + 푐4푆

≥푎푏

+푏푐

+푐푎

.

Exercise 6 (IMO Shortlist, 1995). Given positive real numbers 푎, 푏, 푐. Find all triplets (푥,푦, 푧) of real numbers satisfying the following system of equation

푥 + 푦 + 푧 = 푎 + 푏 + 푐,4푥푦푧 − (푎 푥 + 푏 푦 + 푐 푧) = 푎푏푐.

Exercise 7 (China TST, 2007). Let 푥,푦, 푧 be positive real numbers such that: 푥 + 푦 + 푧 + 푥푦푧 = 4. Prove that:

푦푧푥

+푧푥푦

+푥푦푧≥ 푥 + 푦 + 푧.

Bibliography: [1] Math and Young Journal, Education Publishing House of Vietnam [2] Titu Andreescu, Zuming Feng, 103 trigonometry problems, Birkhouser, 2004 [3] Dragoslav S. Mitrinovici, J. Pecaric, V. Volenec, Recent Advances in Geometric Inequalities.

PROBLEMA COMENTATA GMB 5/2016

By Daniel Sitaru-Romania

Abstract: In this article it is solved and commented a problem published in GMB 5/2016 with three distinct lines of solving belongs to VII; X; XII – standard classes. Keywords: substitutions, inequalities, Viète. În GM5/2016 a fost publicată problema: „Fie 풂,풃, 풄 numere reale strict pozitive astfel încât 풂 + 풃+ 풄 + ퟐ ≤ 풂풃풄. Să se arate că 풂풃풄 ≥ ퟖ” (Autor Daniel Sitaru) Deși publicată la problemele de clasa a IX – a se poate da o rezolvare elementară care nu depășește cunoștințele clasei a VII – a. Plecăm de la ipoteza din problemă:

(푎+ 푏 + 푐) + 2 ≤ 푎푏푐 2(푎 + 푏 + 푐) + 2 ≤ 푎푏푐 + (푎 + 푏 + 푐)

2(푎 + 푏 + 푐) + 3 ≤ 푎푏푐 + (푎 + 푏 + 푐) + 1 (푎푏 + 푏푐 + 푐푎) + 2(푎 + 푏 + 푐) + 3 ≤ 푎푏푐 + (푎푏 + 푏푐 + 푐푎) + (푎 + 푏 + 푐) + 1

(푎 + 1)(푏 + 1) + (푏 + 1)(푐 + 1) + (푐 + 1)(푎 + 1) ≤ (푎 + 1)(푏 + 1)(푐 + 1) + + ≤ 1 (1)

Reducerea ipotezei la această inegalitatea sugerează folosirea următoarelor substituții: 푥 = ; 푦 = ; 푧 = de unde: 푎 = ; 푏 = ; 푐 =

Ipoteza prelucrată (1) devine: 푥 + 푦 + 푧 ≤ 1 sau 1− 푥 ≥ 푦 + 푧; 1 − 푦 ≥ 푥 + 푧; 1 − 푧 ≥ 푥 + 푦

În aceste condiții:

Page 40: ROMANIAN MATHEMATICAL MAGAZINE R.M.M. · pentru ∈ z∗, pe care am rezolvat-o așa cum urmează, am păstrat algoritmul și pentru calculul altor sume, unele putând fi calculate

S.S.M.ROMÂNIA - Filiala Mehedinți 2017

40 ROMANIAN MATHEMATICAL MAGAZINE NR. 18

푎푏푐 =1 − 푥푥

⋅1− 푦푦

⋅1 − 푧푧

≥(푦 + 푧)(푧 + 푥)(푥 + 푦)

푥푦푧≥⏞

≥2 푦푧 ⋅ 2√푥푧 ⋅ 2 푥푦

푥푦푧=

8푥푦푧푥 + 푦푧

= 8

O soluție de nivelul clasei a X – a, în care se utilizează inegalitatea mediilor aparține lui Marian Cucoaneș: 푎푏푐 ≥ 2 + 푎 + 푏 + 푐 ≥ 4√2푎푏푐, 푎푏푐 ≥ 4√푎푏푐 ⇒ (푎푏푐) ≥ 2 ⋅ 2푎푏푐,

(푎푏푐) ≥ 4 ⋅ 2 = 2 ⇒ 푎푏푐 ≥ 8 O soluție de nivelul clasei a XII – a sub semnătura Leonard Giugiuc utilizează relațiile lui Viète. Notând: 푎 + 푏 + 푐 = 3푠 și 푎푏푐 = 푝, din inegalitatea mediilor obținem

푝 ≤ 푠 ⇒ 3푠+ 2 ≤ 푠 ⇒ (푠 − 2)(푠 + 1) ≥ 0 ⇒ 푠 ≥ 2 Din ipoteza 3푠+ 2 ≤ 푝 de unde: 8 = 3 ⋅ 2 + 2 ≤ 3푠 + 2 ≤ 푝 ⇒ 푝 ≥ 8 ⇒ 푎푏푐 ≥ 8 Condiția 푎 + 푏 + 푐 + 2 ≤ 푎푏푐 implică (la fel ca în soluția autorului) relația:

11 + 푎

+1

1 + 푏+

11 + 푐

≤ 1

observă matematicianul indian Abhay Chandra. Urmărind această idee, prin aplicarea inegalității mediilor acesta obține:

2(푎 + 1)(푏 + 1)

≤1

1 + 푎+

11 + 푏

≤ 1 −1

1 + 푐=

푐푐 + 1

2(푏 + 1)(푐 + 1)

≤1

1 + 푏+

11 + 푐

≤ 1−1

1 + 푎≤

푎푎 + 1

2(푐 + 1)(푎+ 1)

≤1

1 + 푐+

11 + 푎

≤ 1−1

1 + 푏≤

푏푏 + 1

Prin înmulțirea acestor relații se obține: 8

(푎 + 1)(푏 + 1)(푐 + 1) ≤푎푏푐

(푎 + 1)(푏 + 1)(푐 + 1)

de unde: 푎푏푐 ≥ 8. Metodele semnalate pot fi utile în rezolvarea multor probleme de acest tip. Bibliografie: [1] Colecția „Gazeta matematică” seria A și B [2] Colecția „Didactica Matematică” [3] Daniel Sitaru – „Math Phenomenon” – Editura Paralela 45 – 2016 [4] Radu Gologan, Daniel Sitaru, Leonard Giugiuc – „300 Romanian Mathematical Challenges” – Editura Paralela 45 - 2016

REFINEMENTS OF THE INEQUALITY BETWEEN THE AM-GM

MEANS

By Mihaly Bencze-Romania

Abstract: In this paper we present some new refinements of the inequality between the AM-GM means. 2010 Mathematic Subject Classification 26D15. Kewords and phrases: AM-GM inequality

Main results: Theorem 1: If 푥,푦 > 0,푘 ∈ 푁, then:

푥 + 푦 ≥12

(푥 + 푦) 푥 + 푦 ≥ (푥푦) (푥 + 푦) ≥ 2(푥푦)

Proof: We have the followings: 푥 + 푦 ≥ (푥 + 푦) 푥 + 푦 ⇔

(푥 − 푦) (푥+ 푦)((푥 ) + (푥 ) 푦 + ⋯+ 푥 (푦 ) + (푦 ) ) ≥ 0

Page 41: ROMANIAN MATHEMATICAL MAGAZINE R.M.M. · pentru ∈ z∗, pe care am rezolvat-o așa cum urmează, am păstrat algoritmul și pentru calculul altor sume, unele putând fi calculate

S.S.M.ROMÂNIA - Filiala Mehedinți 2017

41 ROMANIAN MATHEMATICAL MAGAZINE NR. 18

and 푥 + 푦 ≥ 2푥 푦 , 푥 + 푦 ≥ 2 푥푦 which end the proof.

Theorem 2: If 푥 > 0(푖 = 1,2, … ,푛) then:

푥 ≥14

(푥 + 푥 )(푥 + 푥 ) ≥12

(푥 푥 ) (푥 + 푥 ) ≥ (푥 푥 ) ≥ 푛 푥

Proof: Using Theorem 1 we obtain the affirmations from Theorem 2.

Corollary 2.1. In all triangle 퐴퐵퐶 holds:

1.2푠(푠 − 3푟 − 6푅푟) ≥ ∑(푎 + 푏)(푎 + 푏 ) ≥ ((푠 + 푟 + 4푅푟) + 8푠 푅푟) ≥ ∑(푎푏) ≥ 12푠푅푟

2. 푠(푠 − 12푅푟) ≥ ∑푐((푠 − 푎) + (푠 − 푏) ) ≥ ∑푐 (푠 − 푎)(푠 − 푏) ≥ ∑ (푠 − 푎)(푠 − 푏) ≥ 3푠푟

3. (4푅 + 푟) − 12푠 푅 ≥ ∑(푟 + 푟 ) 푟 + 푟 ≥ 푠 푠 + 푟(4푅 + 푟) ≥ ∑(푟 푟 ) ≥ 3푠 푟

Corollary 2.2. If 푦 > 0(푖 = 1,2, … ,푛) then:

푦 ≥14

푦 + 푦 푦 + 푦 ≥12

(푦 푦 ) 푦 + 푦 ≥ (푦 푦 ) ≥ 푛 푦

Proof: In Theorem 2 we take: 푥 = 푦 (푖 = 1,2, … ,푛)

Reference: [1] Octogon Mathematical Magazine (1993 – 2016)

SOLVED PROBLEM

By Leonard Giugiuc-Romania

Let 풂,풃 and c be positive real numbers such that 풂ퟐ풃 + 풃ퟐ풄 + 풄ퟐ풂 + 풂ퟐ풃ퟐ풄ퟐ = ퟒ.

Proposed by Ardak Mirzakhmedov

Solution: Prove that: 푎 + 푏 + 푐 + 푎푏푐(푎 + 푏 + 푐) ≥ 2(푎푏 + 푏푐 + 푐푎). First, we’ll prove that 푎 푏 + 푏 푐 + 푐 푎 ≥ 3. We assume by absurd that 푎 푏 + 푏 푐 + 푐 푎 < 3. From the AM-GM:

푎 푏 + 푏 푐 + 푐 푎 ≥ 3 (푎 푏)(푏 푐)(푐 푎) = 3푎푏푐 ⇒ 1 > 푎푏푐 ⇒ 1 > 푎푏푐 > 푎 푏 푐 ⇒

4 = 3 + 1 > 푎 푏 + 푏 푐 + 푐 푎 + 푎 푏 푐 = 4, contradiction. Hence 푎 푏 + 푏 푐 + 푐 푎 ≥ 3. Note that since 푎 푏 + 푏 푐 + 푐 푎 ≥ 3, then 푎 푏 푐 ≤ 1 ⇒ 푎 푏 푐 ≤ 푎푏푐 ≤ 1. Now introduce the following:

Lemma 1: 4 ≥ 푎 푏 + 푏 푐 + 푐 푎 + 푎푏푐. Lemma 1’s proof: Let the function

푓: [0, ∞) → ℝ, 푓(푢, 푣,푤) = 4 − 푢푣푤 − (푢 푣 + 푣 푤 + 푤 푢).

Page 42: ROMANIAN MATHEMATICAL MAGAZINE R.M.M. · pentru ∈ z∗, pe care am rezolvat-o așa cum urmează, am păstrat algoritmul și pentru calculul altor sume, unele putând fi calculate

S.S.M.ROMÂNIA - Filiala Mehedinți 2017

42 ROMANIAN MATHEMATICAL MAGAZINE NR. 18

Since 푓 is cyclic and homogenous polynomial of degree three, then according to Cîrtoaje – Zhou theorem, 푓(푢, 푣,푤) ≥ 0∀푢, 푣,푤 ≥ 0 if and only if 푓(1,1,1) ≥ 0 and 푓(푢, 1,0) ≥ 0∀푢 ≥ 0. In our

case, 푓(1,1,1) = 0 ≥ 0. But 푓(푢, 1,0) = 4 − 푢 = (푢 − 2) (4푢 + 1) ≥ 0. In conclusion,

4 ≥ 푎 푏 + 푏 푐 + 푐 푎 + 푎푏푐. Back to main problem now. We have

4푎+ 푏 + 푐

3≥ 푎 푏 + 푏 푐 + 푐 푎 + 푎푏푐 ≥ 푎 푏 + 푏 푐 + 푐 푎 + 푎 푏 푐 = 4 ⇒

4 ≥ 4 ⇒ ≥ 1 ⇒ 푎 + 푏 + 푐 ≥ 3. Denote 푎 + 푏 + 푐 = 3푠 ⇒ 푠 ≥ 1. Also, since (푎 + 푏 + 푐) ≥ 3(푎푏 + 푏푐 + 푐푎), we get that ∃푡 ∈ [0, 푠) such that 푎푏+ 푏푐 + 푐푎 = 3(푠 − 푡 ). Let 푎푏푐 = 푝. The required inequality is equivalent to 푝푠 ≥ 푠 − 4푡 . If ≤ 푡 < 푠, then 푝푠 > 0 ≥ 푠 −4푡 , so our inequality was proved. It’s time for :

Lemma 2: If 0 ≤ 푡 < , then 푝 ≥ (푠 + 푡) (푠 − 2푡). Lemma 2’s proof: We consider the polynomial

푓:푅 → 푅, 푓(푥) = (푥 − 푎)(푥 − 푏)(푥 − 푐). Then 푓(푥) = 푥 − 3푠푥 + 3(푠 − 푡 )푥 − 푝 ⇒ 푓 ʹ(푥) =3[푥 − 2푠푥 + (푠 − 푡 )]. From here obtain that the critical points of 푓 are 푥 = 푠 − 푡 ≤ 푠 + 푡 = 푥 . We form the Rolle’s sequence −∞ < 푠 − 푡 ≤ 푠 + 푡 < ∞.

We have 푓(−∞) = −∞ < 0, 푓(∞) = ∞ > 0 and all the roots of the polynomial 푓 are real numbers, hence by consequence of Rolle’s theorem we get that 푓(푠 − 푡) ≥ 0 and 푓(푠 + 푡) ≤ 0. From here obtain immediately 푝 ≥ (푠 + 푡) (푠 − 2푡). In conclusion, 푝푠 ≥ 푠(푠 + 푡) (푠 − 2푡). It suffices to prove

푠(푠 + 푡) (푠 − 2푡) ≥ 푠 − 4푡 ⇔ 푠(푠 + 푡) ≥ 푠 + 2푡 ⇔ 푠푡 + 2(푠 − 1)푡 + 푠 − 푠 ≥ 0,

∀푡 ∈ 0, . It’s obvious,since 푠 > 0, 푠 − 1, 푠 − 푠 ≥ 0,that:

푠푡 + 2(푠 − 1)푡 + 푠 − 푠 ≥ 푠 − 푠 ≥ 0∀푡 ∈ 0, .The proof is complete.

Remark: The required inequality still holds if we replace the condition

푎 푏 + 푏 푐 + 푐 푎 + 푎 푏 푐 = 4 with the condition 푎 푏 + 푏 푐 + 푐 푎 + 푎 푏 푐 = 4. Indeed, using the same reasoning as above, obtain 푎 푏 + 푏 푐 + 푐 푎 ≥ 3. Now we use the well known Cîrtoaje’s inequality (푎 + 푏 + 푐 ) ≥ 3(푎 푏 + 푏 푐 + 푐 푎) and get from here 푎 + 푏 + 푐 ≥ 3.

From here we apply the above method and get the required result.

PROBLEME PROPUSE

Clasa I

I.1. Măriți numărul 3 cu jumătatea lui 8. Proposed by Maria Ungureanu-Romania

I.2. Suma a trei numere este 9. Care este al treilea număr știind că suma primelor două este 7.

Proposed by Maria Ungureanu-Romania

Page 43: ROMANIAN MATHEMATICAL MAGAZINE R.M.M. · pentru ∈ z∗, pe care am rezolvat-o așa cum urmează, am păstrat algoritmul și pentru calculul altor sume, unele putând fi calculate

S.S.M.ROMÂNIA - Filiala Mehedinți 2017

43 ROMANIAN MATHEMATICAL MAGAZINE NR. 18

I.3. Un termen al unei adunări este 3 iar al doilea cu 2 mai mare decât primul. Care este rezultatul adunării? Proposed by Maria Ungureanu-Romania

I.4. Aflați cifra 푎 astfel încât 93 > 9푎. Proposed by Maria Ungureanu-Romania

I.5. Pentru o prăjitură se folosesc 2 plicuri de zahăr vanilat. Câte pliculețe se folosesc pentru 3 prăjituri? Proposed by Maria Ungureanu-Romania

Clasa a II-a

II.1. O ladă cu gutui cântărește 80 kg. După ce s-au vândut 42 kg în ladă au rămas 30 kg de gutui. Cât cântărește lada? Proposed by Maria Ungureanu-Romania

II.2. Un autoturism consumă 3 litri de benzină la fiecare 50 km. Câți litri consumă la 200 km parcurși? Proposed by Maria Ungureanu-Romania

II.3. Ana și Ionel vor să împartă 22 bomboane astfel încât Ana să ia cu 2 mai multe decât Ionel. Câte bomboane ia fiecare? Proposed by Maria Ungureanu-Romania

II.4. Într-o livadă s-au plantat 45 de vișini pe 9 rânduri în mod egal. Câți vișini s-au plantat pe fiecare rând? Proposed by Maria Ungureanu-Romania

II.5. La o cofetărie s-au primit 55 prăjituri și cu 23 mai multe ciocolate. Câte ciocolate s-au primit?

Proposed by Maria Ungureanu-Romania

Clasa a III-a

III.1. Diferența dintre un număr și zecimea lui este 10. Să se afle numărul.

Proposed by Maria Ungureanu-Romania

III.2. La câtul numerelor 80 și 8 adăugați câtul numerelor 40 și 10.

Proposed by Maria Ungureanu-Romania

III.3. Aflați suma dintre treimea lui 9 și jumătatea lui 50.

Proposed by Maria Ungureanu-Romania

III.4. Scrieți 5 numere impare consecutive știind că primul este cel mai mic număr de trei cifre identice. Proposed by Maria Ungureanu-Romania

III.5. Aflați toate numerele naturale de trei cifre care au suma cifrelor 9 și apoi aranjați-le în ordine descrescătoare. Proposed by Maria Ungureanu-Romania

Clasa a IV-a IV.1. Ce cantitatea de mere s-a transportat în 24 lăzi, fiecare cântărind 36 kg dacă o ladă goală cântărește 2 kg? Proposed by Maria Ungureanu-Romania

Page 44: ROMANIAN MATHEMATICAL MAGAZINE R.M.M. · pentru ∈ z∗, pe care am rezolvat-o așa cum urmează, am păstrat algoritmul și pentru calculul altor sume, unele putând fi calculate

S.S.M.ROMÂNIA - Filiala Mehedinți 2017

44 ROMANIAN MATHEMATICAL MAGAZINE NR. 18

IV.2. Scrie fracțiile cu numărătorul 4 și numitorul un număr natural mai mic decât 7.

Proposed by Maria Ungureanu-Romania

IV.3. O bibliotecă are 5 rafturi iar pe fiecare raft sunt câte 24 cărți. Câte cărți se află în două biblioteci identice? Proposed by Maria Ungureanu-Romania

IV.4. Un avion a coborât cu 890 m. El zboară acum la 5120 m altitudine. La ce altitudine zbura înainte de coborâre? Proposed by Maria Ungureanu-Romania

IV.5. Câte numere naturale sunt cuprinse între 1235 și 1249?

Proposed by Maria Ungureanu-Romania

Clasa a V-a

V.1. Să se afle 푎푏 și 푥 din egalitatea: 푎푏 + 푎푏 = 4352.

Proposed by Ștefan Marica-Romania

V.2. Să se afle xy din relația: ab − xy = cd + xy unde ab și cd sunt pătrate perfecte.

Proposed by Ștefan Marica-Romania

V.3. Să se afle numerele abcb care verifică relațiile: a + 2b + c = a și a + b = 2c.

Proposed by Ștefan Marica-Romania

V.4. Să se arate că dacă: 푎푎 + 푏 = 2017 atunci 푎푏 este un pătrat perfect.

Proposed by Ștefan Marica-Romania

V.5. Să se determine 푎, 푏 ∈ ℕ astfel încât (푎 + 푏) ⋅ 푎푏 + 푏푎 să se reprezinte pătratul unui număr natural. Proposed by Ștefan Marica-Romania

V.6. Arătați că fracția ⋅

este echiunitară. Să se simplifice fracția: .

Proposed by Elena Rîmnicianu-Romania

V.7. Comparați: a. și 2 ; b. ( )

și ; c. și 1.

Proposed by Elena Rîmnicianu-Romania

V.8. Media aritmetică a numerelor 푎 și 푏 este 4,5, media aritmetică a numerelor 푏 și 푐 este 12,5 și media aritmetică a numerelor 푐 și 푎 este 16,5. Calculați media aritmetică a numerelor 푎, 푏 și 푐.

Proposed by Elena Rîmnicianu-Romania

V.9. Determinați numerele naturale de forma 푎푏푐푑 cu proprietatea 푎푏푐푑 + 푏푐푑 + 푐푑 + d = 2018.

Page 45: ROMANIAN MATHEMATICAL MAGAZINE R.M.M. · pentru ∈ z∗, pe care am rezolvat-o așa cum urmează, am păstrat algoritmul și pentru calculul altor sume, unele putând fi calculate

S.S.M.ROMÂNIA - Filiala Mehedinți 2017

45 ROMANIAN MATHEMATICAL MAGAZINE NR. 18

Argumentați că pentru 푎푏푐푑 + 푏푐푑 + 푐푑 + d = 2017 nu există soluție. Proposed by Laviniu Bejenaru-Romania

V.10. Determinați numerele naturale de forma 푎푏푐푑 cu proprietatea: 푎푏푐푑 + 푏푐푑 + 푐푑 + d + a + 푎푏 + 푎푏푐 = 2017. Proposed by Laviniu Bejenaru-Romania V.11. Determinați numerele naturale de forma 푎푏푐푑 cu proprietatea

푎푏푐푑 = 2푏푐푑 + 3푐푑 + 4d + 4a + 3푎푏 +2푎푏푐 . Proposed by Laviniu Bejenaru-Romania

V.12. Se consideră 45 de numere naturale nenule distincte mai mici sau egale cu 80. Demonstrați că există două din acestea, cu diferența egală cu 9. Proposed by Dan Nedeianu- Romania

V.13. Arătați că numărul 102400 … 001, care are în total 2016 zerouri, este compus. Proposed by Neculai Stanciu- Romania

V.14. Arătați că numărul 194800 … 007171, care are în total 2016 zerouri, este compus. Proposed by Neculai Stanciu- Romania

V.15. a. Să se arate că numărul: 푀 = 2002 + 2(1 + 2 + 3 + ⋯+ 2001) este un pătrat perfect. b. Să se arate că numerele 3푛 + 4; 4푛 + 5;푛 ∈ ℕ sunt prime între ele.

Proposed by Mihai Octavian Ungureanu; Daniel Stretcu V.16. Să se demonstreze că:

a. + + +⋯+ >

b. 1 + + +⋯+ < 2 Proposed by Mihai Octavian Ungureanu

V.17. Să se afle 푥 ∈ ℕ∗ știind că fracțiile: și sunt echivalente. Proposed by Mihai Octavian Ungureanu – Romania

V.18. Fie numerele 푛 ,푛 , … ,푛 ,푛 ,푛 . Știind că 푛 = ;푛 = ; 푛 = ; … ; 푛 = și că

푛 + 1 = 푛 + 1 ⋅ 2 ⋅ 3 ⋅… ⋅ 99 ⋅ 100 să se afle numerele 푛 ,푛 , … ,푛 ,푛 Proposed Andrei Bogdan Ungureanu – Romania

V.19. Fie 푥 ∈ ℕ și 퐴 = 9 ⋅ 36 + 16 ⋅ 3 ⋅ 12 + 9 ⋅ 16 ⋅ 8 ⋅ 3 : 6 , 퐵 = 169 ⋅ 6 . Să se arate că: a. 퐴 este pătrat perfect b. (퐴 − 퐵) este divizibil cu 푥.

Proposed by Manuela Prajea – Romania V.20. Să se afle 푥,푦, 푧, 푡 ∈ ℕ încât: 2007 + 2008 + 2009 = 2010

Proposed by Petre Stângescu – Romania V.21. Să se afle 푛, 푘 ∈ ℕ∗ și 푎 ∈ {1,2, … , 9}pentru care: 1 + 3 + 5 +⋯+ (2푘 − 1) = 푎푎… 푎

„ ”5

Proposed by Petre Stângescu – Romania V.22. Să se afle nouă numere naturale consecutive 푎 ,푎 , … ,푎 știind că:

푎 + 푎 + 푎 +. . +푎 = 푎푏 și 푎 + 푎 = 푏푎. Proposed by Ștefan Marica – Romania

V.23. Să se afle 푎푏푐푑 știind că au loc relațiile: 푎푏 + 푐푑 = 푝 푞 + 1;푎푏 − 푐푑 = 푞 unde 푝, 푞 sunt numere prime. Proposed by Ștefan Marica – Romania

V.24. Să se afle numerele 푎푏 și 푘 din relația 푎푎푏 + 푎푏 = 푘 .

Proposed by Ștefan Marica – Romania

Page 46: ROMANIAN MATHEMATICAL MAGAZINE R.M.M. · pentru ∈ z∗, pe care am rezolvat-o așa cum urmează, am păstrat algoritmul și pentru calculul altor sume, unele putând fi calculate

S.S.M.ROMÂNIA - Filiala Mehedinți 2017

46 ROMANIAN MATHEMATICAL MAGAZINE NR. 18

V.25. Să se afle numărul 푎푏푐푑 știind că: 푎푏 = 푐푑 + 3; 3푎 = 푑 − 푐; 푐 + 푑 = 푎 .

Proposed by Ștefan Marica – Romania

V.26. Să se afle numărul prim 푎푏푐푑 știind că 푎푏푐푑 + 푎 = 45 .

Proposed by Ștefan Marica – Romania

V.27. Să se afle numărul 푎푏 știind că: 푎푎 + 푏 = 2017.

Proposed by Ștefan Marica – Romania

V.28. Să se afle numerele prime 푢, 푣,푎푢,푎푣, 푏푢, 푏푣 știind că: 9(푢 + 푣) = 3(푎푢 + 푎푣) = 푏푢 + 푏푣 .

Proposed by Ștefan Marica – Romania

Clasa a VI-a

VI.1. Să se afle 푎 și 푏 astfel încât: = = ;푎, 푏, 푥,푦 ∈ ℕ.

Proposed by Ștefan Marica-Romania

VI.2. Să se afle numerele prime 푎푏푐 și 푐푏푎 astfel încât: 푎푏푐 + 푐푏푎 = 푑푏푑; 푎 ≠ 푏 ≠ 푐 ≠ 푑 ≠ 푎

Proposed by Ștefan Marica-Romania

VI.3. Să se afle 푎푏푐;푎 ≠ 푏 ≠ 푐 ≠ 푎 astfel încât: = .

Proposed by Ștefan Marica-Romania

VI.4. Să se afle numerele prime 푎푏; 푏푐; 푐푎 astfel încât: 푎푏 + 푏푐 + 푐푎 = 푎푎 .

Proposed by Ștefan Marica-Romania

VI.5. Să se arate că numărul 2016 nu se poate scrie ca suma a două pătrate de numere întregi.

Proposed by Nicolae Papacu-Romania

VI.6. Să se determine numere naturale nenule 푎, 푏, 푐 care verifică relația 푎 + 푏 + 푐 = 2016.

Proposed by Nicolae Papacu-Romania

VI.7. Să se arate că numărul 2016 nu se poate scrie ca suma a două cuburi de numere naturale.

Proposed by Nicolae Papacu-Romania

VI.8. Demonstrați că: ⋅ ⋅

+⋅ ⋅

+ ⋯+⋅ ⋅

< .

Proposed by Elena Rîmnicianu – Romania

Page 47: ROMANIAN MATHEMATICAL MAGAZINE R.M.M. · pentru ∈ z∗, pe care am rezolvat-o așa cum urmează, am păstrat algoritmul și pentru calculul altor sume, unele putând fi calculate

S.S.M.ROMÂNIA - Filiala Mehedinți 2017

47 ROMANIAN MATHEMATICAL MAGAZINE NR. 18

VI.9. În interiorul triunghiului dreptunghic isoscel 퐴퐵퐶([퐴퐵] ≡ [퐴퐶]) se construiesc triunghiurile dreptunghice isoscele 퐴퐵퐷 și 퐴퐶퐸 ( 푚(∢퐷) = 90° și 푚(∢퐸) = 90°). a. Să se demonstreze că punctele 퐷,퐴,퐸 sunt coliniare și că 퐷퐸 ∥ 퐵퐶; b. Dacă 퐴퐵 ∩ 퐸퐶 = {푀} să se arate că 퐴퐸 este linie mijlocie în 훥푀퐶퐵. Proposed by Elena Rîmnicianu-Romania

VI.10. Determinați perechile de numere 푥,푦 ∈ 푍 astfel încât |푥 − 4| + (2푥 − 푦) = 0.

Proposed by Elena Rîmnicianu-Romania

VI.11. Media aritmetică a numerelor 푎 și 푏 este 4,5, media aritmetică a numerelor 푏 și 푐 este 12,5 și media aritmetică a numerelor 푐 și 푎 este 16,5. Calculația media aritmetică a numerelor 푎,푏 și 푐.

Proposed by Elena Rîmnicianu-Romania

VI.12. Să se afle a 2016 - a zecimală a numărului .

Proposed by Elena Rîmnicianu-Romania

VI.13. Dacă = , ( ),

, să se arate că ∈ 푁. Proposed by Elena Rîmnicianu-Romania

VI.14. Determinați numerele prime de maxim două cifre x, y, z și t pentru care x y z t, astfel încât 푥 + 푦 + 푧 + 푡 = 푥푦 + 푦푧 + 푧푡 + 푡푥 − 2017 . Menționăm că printre numerele prime cerute cu maxim două cifre un singur număr este mai mare decât 50.

Proposed by Laviniu Bejenaru-Romania VI.15. Determinați în mulțimea numerelor naturale prime soluția (x, y, z) a ecuației

(ퟓ풙 − ퟐ풚)ퟐ − ퟕퟖퟒ = ퟓퟑퟐퟗ −(풚 + 풛)ퟐ . Proposed by Laviniu Bejenaru-Romania

VI.16. Determinați numerele naturale de forma 푎푏푐 cu proprietatea = p ,

unde p este un număr prim ce trebuie determinat. Proposed by Laviniu Bejenaru-Romania

VI.17. Determinați numerele naturale de forma 푎푏푐 cu proprietatea = p , unde p este un

număr prim ce trebuie determinat. Proposed by Laviniu Bejenaru-Romania

VI.18. Determinați numerele naturale de forma 푎푏푐푑 cu proprietatea = p ,

unde p este un număr prim ce trebuie determinat. Proposed by Laviniu Bejenaru-Romania

VI.19. Determinați numerele naturale de forma 푎푏푐푑 cu proprietatea p( )

= q ,

unde p, q sunt numere prime de cel mult două cifre ce trebuie determinate. Proposed by Laviniu Bejenaru-Romania

VI.20. Determinați 푥,푦 ∈ ℕ∗ știind că numărul + este natural.

Proposed by Dan Nedeianu-Romania VI.21. După eliminarea primei cifre zecimale nenule a numărului , rezultatul este de forma

⋅ , unde 푚,푛 ∈ ℕ∗. Să se determine 푚,푛 știind că sunt prime între ele. Proposed by Gheorghe Căiniceanu – Romania

Page 48: ROMANIAN MATHEMATICAL MAGAZINE R.M.M. · pentru ∈ z∗, pe care am rezolvat-o așa cum urmează, am păstrat algoritmul și pentru calculul altor sume, unele putând fi calculate

S.S.M.ROMÂNIA - Filiala Mehedinți 2017

48 ROMANIAN MATHEMATICAL MAGAZINE NR. 18

VI.22. Fie 푥,푦, 푧 ∈ ℕ∗ astfel ca: 푥 + 푦; 푦 + 푧;푧 + 푥 să fie proporționale cu 푧 + 푦, 푥 + 푧, respectiv 푦 + 푥. Să se arate că: a. 푥푦푧 este cub perfect, b. 푥 + 푦 + 푧 = 푥푦 + 푦푧 + 푧푥.

Proposed by Manuela Prajea – Romania VI.23 Să se afle 푙, 푘,푛 ∈ ℕ pentru care: 3푙 + 7푙 + 1 = (푘 + 2) + 3푘

Proposed by Petre Stângescu – Romania VI.24. Aflați 푛,푝 ∈ ℕ; 푝 − număr prim dacă: 2 + 3 = 7푝

Proposed by Petre Stângescu – Romania VI.25. Să se calculeze numerele:

a. 27 027027 … 027

⋅ 37 + 1 b. 702702702 … 702

⋅ 37 + 26

Proposed by Carmen Victorița – Chirfot – Romania VI.26. Să se calculeze

푁 = 270 … 270

, 270270 … + 702 … 702

, 702702 + 27 027 … 027

, 027027 …

Proposed by Carmen Victorița – Chirfot – Romania VI.27. Care este câtul și restul împărțirii numărului 10 la 111? Dar al lui 10 la 999?

Proposed by Carmen Victorița – Chirfot – Romania VI.28. Demonstrați inegalitatea:

+∙

+∙ ⋯ ∙

+<

12017

Proposed Laviniu Bejenaru – Romania VI.29. Demonstrate, for the natural numbers n and a, with a 3, the inequality:

+∙

+∙ ⋯ ∙

+<

1푎+ 1

Proposed Laviniu Bejenaru – Romania VI.30. Solve in ℤ the equation: (푥 + 1) + 푥 + 2 = 20162016.

Proposed by Rovsen Pirguliev – Azerbaidjian VI.31. Prove that the equation 푥 + 푥 − 푥 + 2016 = 0 has no solution.

Proposed by Rovsen Pirguliev – Azerbaidjian VI.32. Solve the equation: 4 = 192푥 − 512

Proposed by Rovsen Pirguliev – Azerbaidjian Clasa a VII-a

VII.1. În 훥퐴퐵퐶;푚 퐴 = 90° și 푀 mijlocul lui 퐴퐶 și 푁 mijlocul lui 퐵퐶. Fie {퐼} = 퐴푁 ∩퐵푀 și 퐷 ∈ [퐴푁];퐸 ∈ [퐵푁]. Să se arate că: ⋅ = 4. Proposed by Ștefan Marica-Romania

VII.2. În ΔABC; m A = 90° și N mijlocul lui BC. Fie G ∈ [AC] și F ∈ [AB. Să se arate că:

+ = 4. Proposed by Ștefan Marica-Romania

VII.3. Fie 푀 = {(푥,푦)|푥,푦 ∈ ℕ; 푥 − 푦 = 2016}. Să se afle 푐푎푟푑푀.

Proposed by Ștefan Marica-Romania

Page 49: ROMANIAN MATHEMATICAL MAGAZINE R.M.M. · pentru ∈ z∗, pe care am rezolvat-o așa cum urmează, am păstrat algoritmul și pentru calculul altor sume, unele putând fi calculate

S.S.M.ROMÂNIA - Filiala Mehedinți 2017

49 ROMANIAN MATHEMATICAL MAGAZINE NR. 18

VII.4. Să se scrie expresia: (푎 + 2푎푏 − 푏 ) + (푏 + 2푎푏 − 푎 ) ca o sumă de patru pătrate perfecte. Proposed by Ștefan Marica-Romania

VII.5. Să se arate că expresia 8 + 15 + 22 + 60; k ∈ ℕ este un multiplu de 7.

Proposed by Ștefan Marica-Romania VII.6. Să se determine numerele prime: ac și bc astfel încât ac + bc = 698.

Proposed by Ștefan Marica-Romania VII.7. Să se arate că dacă 푛 ∈ ℕ∗ atunci:

1 + (1 ⋅ 2) + 2 + (2 ⋅ 3) +⋯+ 푛 + 푛 (푛 + 1) ≤ 2 .

Proposed by Ștefan Marica-Romania

VII.8. Să se determine mulțimea triunghiurilor dreptunghice 퐴퐵퐶;푚 퐴 = 90° ale căror lungimi de laturi verifică simultan relațiile: 푎 + 푏 = 5푐; 3푏 − 2푎 = 2푐.

Proposed by Ștefan Marica-Romania

VII.9. Să se determine un triunghi 퐴퐵퐶 știind că 푆[퐴퐵퐶] = 푘 și 퐴퐵 + 퐵퐶 + 퐶퐴 = 푘 ; 푘 ∈ ℕ.

Proposed by Ștefan Marica-Romania VII.10. Știind că 푎푏 și 푏푎 sunt numere prime și 푎푏 + 푏푎 + 푛 = 푘 ; 푛, 푘 ∈ ℕ să se determine 푎푏.

Proposed by Ștefan Marica-Romania

VII.11. a. Determinați numerele naturale 푛 pentru care 4 + 12 este pătrat perfect.

b. Determinați numerele naturale 푛 care verifică relația 4 + 12 = 6 + 3 + 8 + 9 .

Proposed by Nicolae Papacu-Romania VII.12. Fie trapezul dreptunghic 푀퐴푇퐸,푀퐸 ∥ 퐴푇,푚(∢푀퐴푇) = 90°. Aflați perimetrul și lungimile diagonalelor trapezului în următoarele cazuri: a. 푀퐸 = 8 cm, 푀퐴 = 10 cm și 퐴푇 = 18 cm;

b. 푀퐸 = 퐴푀 = 8 cm și 푇퐸 = 10 cm. Proposed by Elena Rîmnicianu-Romania

VII.13. Să se determine cifra 푥 pentru care: a) 0, 푥(푥) + 0, (푥) ∈ 푄 . b) 0, 푥 − 0,0푥 ∈ 푄.

Proposed by Elena Rîmnicianu-Romania

VII.14. Se dă un triunghi în care laturile sunt direct proporționale cu 6,8,10. Știind că semiperimetrul triunghiului este de 120 cm, să se determine natura acestui triunghi și aria triunghiului.

Proposed by Elena Rîmnicianu-Romania

VII.15. Determinați numerele naturale x, y și z care sunt soluții ale ecuației

2 + 2 + 2 + 2 + 3 + 23 + 18 = 2017 . Proposed by Laviniu Bejenaru – Romania

Page 50: ROMANIAN MATHEMATICAL MAGAZINE R.M.M. · pentru ∈ z∗, pe care am rezolvat-o așa cum urmează, am păstrat algoritmul și pentru calculul altor sume, unele putând fi calculate

S.S.M.ROMÂNIA - Filiala Mehedinți 2017

50 ROMANIAN MATHEMATICAL MAGAZINE NR. 18

VII.17. Determinați numerele naturale prime m și n pentru care

( ) ∈ N.

Proposed by Laviniu Bejenaru – Romania VII.18. Câte triplete (p, q, r) de numere prime de cel mult două cifre verifică relația de divizibilitate 푝 + 2(푞 + 푟) |10(푝+ 푟)(푞 + 푟) + 2푞 + 푟

Proposed by Laviniu Bejenaru – Romania VII.19. Fie 푎, 푏, 푐,푑, 푒 ∈ ℤ∗ astfel ca:

(1 + 푎)(1 + 푏)(1 + 푐)(1 + 푑)(1 + 푒) = (1− 푎)(1− 푏)(1 − 푐)(1− 푑)(1− 푒) Să se calculeze (1 − |푎|)(1− |푏|)(1− |푐|)(1− |푑|)(1− |푒|).

Proposed by Dan Nedeianu – Romania VII.20. Să se arate că dacă 푥,푦, 푧, 푡 sunt numere strict pozitive și mai mici decât atunci

1 < 푥푦 + (1 − 푦)(1− 푧) + (1 − 푥)(1 − 푡) + 푧푡 < 2 Proposed by Manuela Prajea – Romania

VII.21. Se dau numerele 푁 = 푎푎 + (2푎 + 1); 푁 = (푎푎 + 1) − . Să se afle 푎 ∈ ℕ astfel încât 푁 = 푁 . Proposed by Ștefan Marica – Romania

VII.22. Să se arate că dacă 푎, 푏, 푐 ∈ (0,∞) atunci: 푎푐

+2푐푏

푐푏

+2푏푎

푏푎

+2푎푐

≥ 16√2

Proposed by Daniel Sitaru – Romania VII.23. Să se arate că dacă: 푎, 푏, 푐,푑, 푥,푦, 푧, 푡 ∈ (0,∞); 푎 + 푏 + 푐 + 푑 = 푥 + 푦 + 푧 + 푡 = 2 atunci:

푎푎+ 푥

+푏

푏 + 푦+

푐푐 + 푧

+푑푑 + 푡

≥ 1

Proposed by Daniel Sitaru – Romania VII.24. Să se arate că dacă 푎 > 0, 푏 > 0, 푐 > 0 atunci:

푎 +푏푐푎

푏 +푎푐푏

푐 +푎푏푐

≥ 8푎푏푐√푎푏푐

Proposed by Daniel Sitaru – Romania VII.25. Să se arate că dacă 푎, 푏, 푐,푑 ∈ (0,∞),푎 + 푏 + 푐 + 푑 = 2 atunci:

푎푏푎 + 푏

+푏푐푏 + 푐

+푐푑푐 + 푑

+푑푎푑 + 푎

≤ 1

Proposed by Daniel Sitaru – Romania VII.26. Să se arate că dacă 푥 , 푥 , … , 푥 ∈ (0,∞) și 푥 + 푥 +⋯+ 푥 = 2; 푛 ∈ ℕ∗; 푛 ≥ 2 atunci: + + ⋯+ + ≤ 1

Proposed by Daniel Sitaru – Romania VII.27. Să se afle 푥,푦, 푧, 푡 ∈ ℝ încât:

푥 + (4 − 푥) + 푦 + (4− 푦) = 9√2− 푧 + (5− 푧) − 푡 + (5− 푡) Proposed by Daniel Sitaru – Romania

VII.28. Să se arate că dacă 푎, 푏, 푐,푑 ∈ (0,∞) și 57푎푏푐푑 = 1 atunci: (5푎 + 푎 + 1)(5푏 − 푏 + 1)(7푐 + 푐 + 1)(7푑 − 푑 + 1) ≥ 9

Proposed by Daniel Sitaru – Romania

VII.29. Să se arate că în triunghiul 퐴퐵퐶 este valabilă relația: ∑ 푝 + √푎푏+ 푎푐 ≤ √2∑√푎 + 푏

Proposed by Daniel Sitaru – Romania VII.30. Să se arate că dacă 푎, 푏 ∈ (0,∞) atunci: 푎 (푏 + 2) + 푏 (푎 + 1) ≥ 푎 푏(3 + 2푏)

Proposed by Daniel Sitaru – Romania

Page 51: ROMANIAN MATHEMATICAL MAGAZINE R.M.M. · pentru ∈ z∗, pe care am rezolvat-o așa cum urmează, am păstrat algoritmul și pentru calculul altor sume, unele putând fi calculate

S.S.M.ROMÂNIA - Filiala Mehedinți 2017

51 ROMANIAN MATHEMATICAL MAGAZINE NR. 18

VII.31. Să se arate că dacă 푥,푦, 푧 ∈ (0,∞);푥 + 푦 + 푧 = 3 atunci: 1

푥 + 1+

1푦 + 1

+1

푧 + 1≥ 2 √3− 1

Proposed by Daniel Sitaru – Romania VII.32. În ∆퐴퐵퐶,∢퐵퐴푀 ≡ ∢푀퐴푁 ≡ ∢푁퐴퐶,퐵푀 = 푥,푀푁 = 푦,푁퐶 = 푧, 퐴퐵 = 푐,퐴퐶 = 푏, 푀 ∈ (퐵푁),푁 ∈ (푀퐶). Să se arate că: 푦 =

Proposed by Daniel Sitaru – Romania VII.33. Să se arate că dacă 푎, 푏, 푐 ∈ (0,∞); 푎 + 푏 + 푐 = 1 atunci:

1푎

+1푏

+1푐≥ 4

11 + 푎

+1

1 + 푏+

11 + 푐

Proposed by Daniel Sitaru – Romania VII.34. Să se arate că dacă 푥,푦, 푧, 푡 ∈ [1,∞) atunci:

푥푦푧 − 1푥푦푧

+푦푧푡 − 1푦푧푡

+√푧푡푥 − 1푧푡푥

+푡푥푦 − 1푡푥푦

≤ 2

Proposed by Daniel Sitaru – Romania VII.35. Să se arate că dacă 푎, 푏, 푐 ∈ (0,∞)

(푎 + 푏 + 푐 ) ∙1푎

+1푏

+1푐

> 2푎푐

+푐푏

+푏푎

Proposed by Daniel Sitaru – Romania VII.36. Să se arate că dacă 푎, 푏 ∈ ℕ∗; 푥,푦, 푧 > 0 atunci:

(푎 + 푏 )푥푦 + 푎푏(푥 + 푦 ) ≤ (푎 + 푏)(푥 + 푦 + 푧)

Proposed by Daniel Sitaru – Romania VII.37. Să se arate că dacă 푎, 푏, 푐 ∈ (0,∞) și 푛 ∈ ℕ∗ atunci:

푎√푏푐

+푏√푎푐

+푐√푎푏

≥ 푎 + 푏 + 푐

Proposed by Daniel Sitaru – Romania VII.38. Să se arate că dacă 푎, 푏, 푐 ∈ (0,∞) atunci:

푎(푏 + 푐) ≥ 12푎푏푐

Proposed by Daniel Sitaru – Romania VII.39. Să se arate că dacă 푎, 푏 ∈ (0,∞);푚,푛 ∈ ℕ∗ atunci:

1(푚푎 + 푛푏)(푛푎 +푚푏)

≥9

(푚 + 푛)(푎 + 푏 + 푐)

Proposed by Daniel Sitaru – Romania VII.40. Să se arate că dacă 푝 > 0;푎, 푏, 푐 ∈ (0,∞) ∖ {푝}, + + = atunci:

(푎 + 푏)(푏 + 푐)(푐 + 푎)(푎 − 푝)(푏 − 푝)(푐 − 푝) ≤

푎 + 푏 + 푐3푝

Proposed by Daniel Sitaru – Romania Clasa a VIII-a

VIII.1. Să se determine unghiurile 훥퐴퐵퐶 care verifică relațiile:

⎩⎪⎨

⎪⎧sin퐴 ⋅ sin퐵 − sin퐶 = √

sin퐵 ⋅ sin 퐶 − sin퐴 = √

sin퐶 ⋅ sin퐴 − sin퐴 = √

Proposed by Ștefan Marica-Romania

Page 52: ROMANIAN MATHEMATICAL MAGAZINE R.M.M. · pentru ∈ z∗, pe care am rezolvat-o așa cum urmează, am păstrat algoritmul și pentru calculul altor sume, unele putând fi calculate

S.S.M.ROMÂNIA - Filiala Mehedinți 2017

52 ROMANIAN MATHEMATICAL MAGAZINE NR. 18

VIII.2. Să se determine 푎, 푥,푦, 푧 ∈ ℝ astfel încât: 푎 = 19683푎 = 6561푎 = 2187

.

Proposed by Ștefan Marica-Romania

VIII.3. Se consideră numerele reale 푎 < 푏 < 푐 care verifică următoarele două proprietăți:

i. pentru orice 푥 ∈ [푎, 푏] și 푦 ∈ [푏, 푐], rezultă 푥 + 푦 ∈ [푎, 푐], ii. pentru orice 푥,푦 ∈ [푎, 푐], rezultă 푥 ⋅ 푦 ∈ [푎, 푐] . 1. Să se determine 푎, 푏, 푐 știind că sunt numere întregi. 2. Să se determine numerele 푎, 푏, 푐 știind că există 푥 ∈ [푎, 푐], 푥 ≠ 0 astfel încât ∈ [푎, 푐] și –푥 ∈ [푎, 푐].

Proposed by Nicolae Papacu-Romania VIII.4. Pentru numerele reale 푎, 푏, 푐,푑 ∈ (0,∞) se notează cu

퐸 = (푎 + 푏)(푎+ 푐) + (푏 + 푐)(푏 + 푑) + (푐 + 푑)(푐 + 푎) + (푑 + 푎)(푑 + 푏) Să se demonstreze că: 1. 퐸 ≤ 2(푎 + 푏 + 푐 + 푑) 2. 퐸 ≥ 2 √푎푏 + √푏푐 + √푐푑 + √푑푎

Proposed by Nicolae Papacu-Romania VIII.5. Fie funcția 푓:푁 → 푅, 푓(푥) = 2푥 − 1. Să se arate că 푓(1) + 푓(2) + 푓(3) +⋯+ 푓(2016) este pătrat perfect. Proposed by Elena Rîmnicianu-Romania VIII.6. Pentru un spectacol de circ se confecționează un cort în forma de piramida hexagonală regulată 푉퐴퐵퐶퐷퐸퐹 având muchiile laterale de 26 m și latura bazei de 20 m. a. Să se afle înălțimea și apotema piramidei; b. Știind că metrul pătrat de material folosit pentru acoperirea cortului costă 16,5 lei să se calculeze costul materialului; c. Să se calculeze aria totală și volumul piramidei date; d. Să se afle sinusul unghiului format de o față laterală cu planul bazei.

Proposed by Elena Rîmnicianu-Romania VIII.7. Un acvariu în formă de prismă hexagonală regulată 퐴퐵퐶퐷퐸퐹퐴ʹ퐵ʹ퐶 ʹ퐷ʹ퐸 ʹ퐹ʹ cu muchia bazei de 30 cm și unghiul format de diagonal 퐸ʹ퐵 cu planul bazei de 60°. Ce volum de apă este necesar pentru ca acvariul să fie umplut în proporție de 75%?

Proposed by Elena Rîmnicianu-Romania

VIII.8. Fie funcția 푓:푅 → 푅, 푓(푥) = 푥 − 40. a. Aflați sinusul unghiului format de graficul funcției cu axa absciselor. b. Aflați distanța de la origine la reprezentarea grafică a funcției.

Proposed by Elena Rîmnicianu-Romania VIII.9. Find the numbers 푎, 푏, 푐 ∈ 푁∗ such that ∈ 푁, ∈ 푁 and ∈ 푁.

Proposed by Gheorghe Alexe , George – Florin Șerban-Romania

VIII.10. Dacă 푥,푦 ∈ ℕ∗ astfel încât numărul 푥 + 푦 să fie un divizor comun al numerelor 2푥 푦 și 2푥푦 , atunci să se arate că 푥 = 푦. Proposed by Dan Nedeianu-Romania

VIII.11. Dacă 푥,푦 ∈ ℝ încât 푥 ∈ [−7,3] și 푥 − 4푦+ 3 = 0, să se arate că numărul 푎 = 2(푥 + 7) + 16(푦 + 1) + (푥 − 3) + 8(2푦 − 3) este irațional.

Proposed by Carmen Coadă – Romania VIII.12. În paralelipipedul dreptunghic 퐴퐵퐶퐷퐴 퐵 퐶 퐷 ;퐴퐶 ⊥ (퐴 퐵퐷). Să se arate că paralelipipedul este cub. Proposed by Manuela Prajea – Romania

Page 53: ROMANIAN MATHEMATICAL MAGAZINE R.M.M. · pentru ∈ z∗, pe care am rezolvat-o așa cum urmează, am păstrat algoritmul și pentru calculul altor sume, unele putând fi calculate

S.S.M.ROMÂNIA - Filiala Mehedinți 2017

53 ROMANIAN MATHEMATICAL MAGAZINE NR. 18

VIII.13. Să se arate că: 1 + ( ) ⋅ 1 + ( ) ⋅ … ⋅ 1 + ( ) ≤ 푒 ,푝 ∈ 푁∗, (∀)푛 ∈ 푁∗.

Proposed by Gheorghe Alexe , George – Florin Șerban-Romania VIII.14. Să se arate că dacă: 푎, 푏, 푐 ∈ ℝ∗ și + + = 1 + + + atunci 푎 + 푏 + 푐 > 0.

Proposed by Daniel Sitaru – Romania VIII.15. Să se arate că dacă: 푎, 푏, 푐,푑 ∈ (0,∞) atunci: ∑ ≥ (푎푏 + 푏푐 + 푐푑 + 푑푎)

Proposed by Daniel Sitaru – Romania VIII.16. Să se arate că dacă 푎 + 푏 = 2 atunci: (푎 + 푏 ) 1 + + ≥ 10

Proposed by Daniel Sitaru – Romania VIII.17. Să se arate că dacă 푎, 푏, 푐 ∈ (0,∞) atunci: ∑ + ≥ 2∑ √

Proposed by Daniel Sitaru – Romania VIII.18. Să se arate că dacă 푥,푦,푧 ∈ (0,∞) și 푥푦 + 푦푧 + 푧푥 = 4푥푦푧 atunci:

121

푥 + 4+

1푥 + 푦

≤ 12 + 71푥

Proposed by Daniel Sitaru – Romania VIII.19. Să se arate că dacă 푎, 푏, 푐 ∈ (0,∞) atunci: 3 + + + 4√2(푎푏 + 푏푐 + 푐푎) ≥ 24

Proposed by Daniel Sitaru – Romania VIII.20. Să se demonstreze că: √ + √ + ⋯+ √ <

√+

√+⋯+

Proposed by Daniel Sitaru – Romania VIII.21. Să se arate că dacă: 푎, 푏, 푐 ∈ (0,∞) atunci:

푎 푏 + 푎푏√푎푏 + 푎푏 ≤3√2

2푎 + 푏

Proposed by Daniel Sitaru – Romania VIII.22. Să se arate că dacă 푎, 푐 ∈ [3,5];푏,푑 ∈ [2,4] atunci:

푎 + 푏 + 푐 + 푑 + 81푎

+1푏

+1푐

+1푑

< 28

Proposed by Daniel Sitaru – Romania VIII.23. Fie 퐴 = ([푥 푥 푥 ]) − [푥 ] ∙ [푥 ] ∙ [푥 ] , 퐵 = max [푥 ] − 푥 unde 푥 , 푥 , 푥 ∈ ℝ∗ . Să se arate că |퐴| ≤ 퐵. Am notat cu [푥] − partea întreagă a numărului real 푥.

Proposed by Daniel Sitaru – Romania VIII.24. Să se arate că dacă 푎, 푏, 푐 ∈ (0,∞) atunci:

푎 푏푎 + 푏

+푏 푐푏 + 푐

+푐 푎푐 + 푎

≤1

푎 + 푏+

1푏 + 푐

+1

푐 + 푎

Proposed by Daniel Sitaru – Romania VIII.25. Să se arate că dacă 푎, 푏, 푐,푑 ∈ (0,∞); 푎 + 푏 + 푐 + 푑 = 1 atunci:

1푎

+1푏

+1푐

+1푑≥ 5

11 + 푎

+1

1 + 푏+

11 + 푐

Proposed by Daniel Sitaru – Romania VIII.26. Să se arate că dacă 푎, 푏, 푐 ∈ ℝ; 푎푏푐 = 1 atunci

푎(푎 − 3푏 − 3푐) + 푏(3푎 − 푏 − 푐) + 푐(3푎 − 푏 − 푐) ≥ 27 Proposed by Daniel Sitaru – Romania

VIII.27. Să se rezolve ecuația: 푥 − 2퐴푥 + 퐵 = 0; 퐴 = ℎ 푟 + ℎ 푟 + ℎ 푟 ; 퐵 = 푟(ℎ + ℎ + ℎ ) + 2푟(푟 + 푟 + 푟 )

Proposed by Daniel Sitaru – Romania VIII.28. Să se arate că dacă 푥,푦,푧 ∈ (0,∞) atunci

Page 54: ROMANIAN MATHEMATICAL MAGAZINE R.M.M. · pentru ∈ z∗, pe care am rezolvat-o așa cum urmează, am păstrat algoritmul și pentru calculul altor sume, unele putând fi calculate

S.S.M.ROMÂNIA - Filiala Mehedinți 2017

54 ROMANIAN MATHEMATICAL MAGAZINE NR. 18

13 푥 + 4 푥푧 ≥ 12 푥 푦푧

Proposed by Daniel Sitaru – Romania VIII.29. Să se arate că dacă 푎 > 0 și 푥,푦, 푧 ∈ [푎, 푏] atunci:

푎푏푥 + 푦푥푦

+푦 + 푧푦푧

+푧 + 푥푧푥

≤ 3(푎 + 푏)

Proposed by Daniel Sitaru – Romania VIII.30. Să se arate că dacă 푎, 푏, 푐 ∈ ℝ,푎 + 푏 + 푐 = 21 atunci:

4(푎 + 푏 + 푐 ) + 2(푎 + 푏 + 푐 ) ≥ 푎 + 푏 + 푐 Proposed by Daniel Sitaru – Romania

VIII.31. Să se arate că dacă 푎 ∈ ℝ atunci: 4푎 + 2푎 + 7 ≥ 푎 + 푎 Proposed by Daniel Sitaru – Romania

VIII.32. Să se arate că dacă 푎, 푏, 푐,푑 > 0 atunci: ( )( )≤ + + +

Proposed by Daniel Sitaru – Romania VIII.33. Să se arate că dacă 푎, 푏, 푐 ∈ ℝ,푎 + 푏 + 푐 = 2 atunci:

2(푎 + 푏 + 푐 ) + 10(푎 + 푏 + 푐 ) ≥ 5(푎 + 푏 + 푐 ) + 1 Proposed by Daniel Sitaru – Romania

VIII.34. Aria patrulaterului convex 퐴퐵퐶퐷 este 푆 = ( )( ) ;퐴퐵 = 푎;퐵퐶 = 푏; 퐶퐷 = 푐;퐴퐷 = 푑. Arătați că 퐴퐶 = 퐵퐷.

Proposed by Daniel Sitaru – Romania VIII.35. Să se afle 푥 , 푥 , … , 푥 ∈ (0,∞);푛 ∈ ℕ∗

⎩⎪⎨

⎪⎧푥푥

+푥푥

+ ⋯+푥푥

+푥푥

= 푛

푥푥

+푥푥

+ ⋯+푥푥

+푥푥

= 푛

푥 + 푥 + ⋯+ 푥 = 푛

Proposed by Daniel Sitaru – Romania VIII.36. Dacă 푥 , 푥 , … , 푥 ∈ (0,∞) și 푥 + 푥 + ⋯+ 푥 = 1. Să se arate că:

푥 푥 ≤10072015

Proposed by Daniel Sitaru – Romania VIII.37. Să se rezolve în numere naturale ecuația:

푥 + 푘3푘 + 2

=푘 + 1

푥 + 3푘 + 1

Proposed by Daniel Sitaru – Romania VIII.38. Să se arate că în orice triunghi este adevărată relația:

1푎(푏 + 푐)

+1

푏(푎 + 푐)+

1푐(푎 + 푏)

≥3푝

Proposed by Daniel Sitaru – Romania VIII.39. Să se arate că dacă: 푎 + 푏 + 푐 + 푑 = 8 ; 푎, 푏, 푐,푑 ∈ (0, ∞), atunci :

1(푎 + 1)(푎 + 4)

+1

(푏 + 1)(푏 + 4)+

1(푐 + 1)(푐 + 4)

+1

(푑 + 1)(푑 + 4)≤ 3

Proposed by Daniel Sitaru – Romania

Page 55: ROMANIAN MATHEMATICAL MAGAZINE R.M.M. · pentru ∈ z∗, pe care am rezolvat-o așa cum urmează, am păstrat algoritmul și pentru calculul altor sume, unele putând fi calculate

S.S.M.ROMÂNIA - Filiala Mehedinți 2017

55 ROMANIAN MATHEMATICAL MAGAZINE NR. 18

VIII.40. Să se arate că dacă: ; ; ∈ [2,4];푎, 푏, 푐 ∈ (0,∞), atunci:

푎푦 + 푏푦 + 푐푦 + 8(푎푥 + 푏푥 + 푐푥 ) ≤ 6(푎푥 푦 + 푏푥 푦 + 푐푥 푦 ) Proposed by Daniel Sitaru – Romania

VIII.41. Să se arate că dacă 푥,푦,푧 ∈ (0,∞) atunci : 푥

푥 + 푥 + 1+

푦푦 + 푦 + 1

+푧

푧 + 푧 + 1− 1 ≤ 푥 + 푦 + 푧

Proposed by Daniel Sitaru – Romania VIII.42. Fie 푥,푦, 푧 ∈ (0,∞). Să se arate că:

푥 + 푦 + 푧 ≤12푥

1푦

+1푧

+ 푦1푥

+1푧

+ 푧1푦

+1푥

Proposed by Daniel Sitaru – Romania VIII.43. Să se arate că în triunghiul 퐴퐵퐶 este valabilă relația:

푎푏

∙푏푐

∙푐푎

≥ 1

Proposed by Daniel Sitaru – Romania VIII.44. Să se arate că dacă 푎, 푏, 푐 ∈ (0,∞) ; 푎 ≠ 푏 ≠ 푐 ≠ 푎 atunci:

(푎 − 푐) + (푐 − 푏) + (푏 − 푎) ≥ 16 푎 1 +푎

푐 − 푏

Proposed by Daniel Sitaru – Romania VIII.45. Să se arate că dacă 푥 + 푦 + 푧 = 1 atunci:

푥 푦푥 + 1

+푦√푥 + 1푥 + 2

≤ 1

Proposed by Daniel Sitaru – Romania

Clasa a IX-a

IX.1. Să se afle 푥,푦 ∈ ℕ astfel încât: 푥 + 푦 + 2(푥 − 푦 + 1) = (푥 − 푦 + 2) .

Proposed by Ștefan Marica-Romania

IX.2. Să se determine mulțimea 퐴 = 푛 ∈ 푁| ∈ 푁 . Proposed by Ștefan Marica - Romania

IX.3. Să se rezolve ecuația 푝 + [푥] = [푝푥], unde 푝 este un număr natural. Proposed by Nicolae Papacu- Romania

IX.4. Fie triunghiul 훥퐴퐵퐶 ascuțitunghic. Să se demonstreze inegalitatea:

tg 퐴 ⋅ tg퐵 ⋅ tg 퐶 + tg 퐴 ⋅ tg 퐵 ⋅ tg퐶 + tg퐴 ⋅ tg 퐵 ⋅ tg 퐶 > 2 ⋅ (tg퐴 ⋅ tg퐵 + tg퐴 ⋅ tg퐶 + tg퐶 ⋅ tg퐵) − 1

Proposed by Gheorghe Alexe; George – Florin Șerban – Romania

IX.5. Dacă 푥, 푦 > 0, arătați că: + √ ⋅√

≥ .

Proposed by Gheorghe Alexe; George – Florin Șerban – Romania

IX.6. Prove that for any real numbers 푎, 푏 and 푐 the following inequality holds:

Page 56: ROMANIAN MATHEMATICAL MAGAZINE R.M.M. · pentru ∈ z∗, pe care am rezolvat-o așa cum urmează, am păstrat algoritmul și pentru calculul altor sume, unele putând fi calculate

S.S.M.ROMÂNIA - Filiala Mehedinți 2017

56 ROMANIAN MATHEMATICAL MAGAZINE NR. 18

8(푎 − 푏푐)(푏 − 푐푎)(푐 − 푎푏) ≤ (푎 + 푏 + 푐 )

Proposed by Richdad Phuc -Vietnam

IX.7. Pe un cerc de centru 푂 se construiesc punctele 퐴,퐵,퐶,퐷,퐸,퐹,퐺,퐻 în această ordine. Dacă segmentele [퐴퐷], [퐻퐸], [퐵퐺] și [퐶퐹] determină prin intersecția lor (două câte două) vârfurile unui dreptunghi 푃푄푅푆, să se arate că

푂푃⃗ + 푂푄⃗ + 푂푅⃗ + 푂푆⃗ = 푂퐴⃗ + 푂퐵⃗ + 푂퐶⃗ + 푂퐷⃗ + 푂퐸⃗ + 푂퐹⃗ + 푂퐺⃗ + 푂퐻⃗

Proposed by Dan Nedeianu- Romania

IX.8. Fie triunghiul 퐴퐵퐶 și 퐷 ∈ (퐵퐶),퐸 ∈ (퐴퐶) picioarele bisectoarelor sale din 퐴, respectiv din 퐵. Fie punctele 퐴ʹ ,퐵ʹ astfel încât 퐷 este mijlocul lui 퐴퐴ʹ și 퐸 este mijlocul lui 퐵퐵ʹ . Să se demonstreze că există o infinitate de triunghiuri neisoscele 퐴퐵퐶 pentru care punctele 퐴ʹ,퐵ʹ,퐶 ʹ sunt coliniare. Proposed by Dana Heuberger- Romania IX.9. Fie 푉퐴퐵퐶, numerele 푘, 푡 ∈ (0,1) și punctele 푀,푁 ∈ (퐵퐶),푃,푄 ∈ (퐴퐶) astfel încât = =

푘 și = = 푡. a) Să se arate că triunghiurile 퐴푀푁 și 퐵푃푄 au același centru de greutate dacă și

numai dacă 푡 = 푘. b) Dacă {퐸} = 퐴푀 ∩ 퐵푄, {퐹} = 퐴푁 ∩ 퐵푃 iar dreapta 퐶퐸 este mediană în triunghiul 퐴퐵퐶, să se arate că centrul de greutate al triunghiului 퐴푀푁 se află pe (퐸퐹).

Proposed by Dana Heuberger- Romania

IX.10. If 푥,푦, 푧 ∈ 0, , then: ( ) + ( ) + ( ) > .

Proposed by D. M. Bătinețu – Giurgiu, Neculai Stanciu- Romania IX.11. If 푥,푦, 푧 > 0, then: (푥 푦 + 푦 푧 + 푧 푥 )

( ) + ( ) + ( ) ≥

Proposed by D. M. Bătinețu – Giurgiu, Neculai Stanciu, Romania IX.12. Fie numerele reale 푎 ,푎 , … ,푎 în progresie aritmetică. Știind că 푎 = 1009, calculați suma 푆 = 푎 + 푎 + ⋯+ 푎 .

Proposed by Diana Trăilescu - Romania IX.13. Fie numerele reale 푎 ,푎 , … ,푎 în progresie geometrică. Știind că 푎 = 1, calculați produsul 푃 = 푎 푎 … 푎 .

Proposed by Diana Trăilescu – Romania IX.14. Dacă 푎, 푏 ∈ ℝ; |푎| + |푏| < 1 să se arate că:

√2 < (푎 + 1) + 푏 + 푎 + (푏 − 1) < 3√2 Proposed by Manuela Prajea – Romania

IX.15. Dacă |푢⃗| = 26; |푣⃗| = 10;|푢⃗ − 푣⃗| = 24 să se calculeze |푢⃗ + 푣⃗| Proposed by Dan Nănuți – Romania

IX.16. Să se arate că dacă 푎, 푏, 푐 ∈ ℝ∗; 푎 + 푏 + 푐 ≠ 0 și + + = atunci: 1

푎+

1푏

+1

푐=

1푎 + 푏 + 푐

Proposed by Mihai Octavian Ungureanu IX.17. Să se arate că √5,√6,√7 nu pot fi termenii unei progresii aritmetice (nu neapărat consecutivi). Proposed by Dorel Căpraru – Romania IX.18. Să se calculeze raportul dacă în 훥퐴퐵퐶 există relația:

Page 57: ROMANIAN MATHEMATICAL MAGAZINE R.M.M. · pentru ∈ z∗, pe care am rezolvat-o așa cum urmează, am păstrat algoritmul și pentru calculul altor sume, unele putând fi calculate

S.S.M.ROMÂNIA - Filiala Mehedinți 2017

57 ROMANIAN MATHEMATICAL MAGAZINE NR. 18

sin퐴2

∙ cos퐵2

= sin퐵2

∙ cos퐴2

Proposed by Claudia Nănuți - Romania IX.19. În 훥퐴퐵퐶 ascuțitunghic; 퐻 − ortocentrul, 푂 − centrul cercului circumscris. Să se arate că dacă: 퐴[퐻퐵퐶] ≥ 퐴[퐻퐶퐴] ≥ 퐴[퐻퐴퐵] atunci: 퐴[푂퐴퐵] ≥ 퐴[푂퐶퐴] ≥ 퐴[푂퐵퐶]

Proposed by Leonard Giugiuc – Romania IX.20. Let 푥;푦; 푧 > 0. Prove that:

푥푦

+푦푧

+푧푥

푥푦

+푦푧

+푧푥

≥푥 + 푥푦 + 푦

Proposed by Le Viet Hung – Vietnam IX.21. Given 푎, 푏, 푐 ∈ [1, +∞) prove that:

( )( )( ) √√ √ √

≥ 2.

Proposed by Daniel Sitaru – Romania IX.22. If 푎, 푏, 푐 > 0 and 푎 + 푏 + 푐 = 3 prove that:

푎1푏

+1푐

≥18

푎 + 푏 + 푐

where 푛 ≥ 0. Proposed by Marin Chirciu – Romania

IX.23. Given 푎, 푏, 푐 > 0 and 푎 + 푏 + 푐 = 6, prove: + + + 푎 + 푏 + 푐 ≥ 6 Proposed by Nguyen Phuc Tang - Dong Thap – Vietnam

IX.24. Let 푥,푦, 푧 be real numbers greater than 1 and satisfying 2 + 푥푦 + 푦푧 + 푧푥 ≤ 푥 + 푦 + 푧 + 푥푦푧. Prove that 푥푦 + 푦푧 + 푧푥 ≤ 푥푦푧(푥 + 푦 + 푧 − 3)

Proposed by Hung Nguyen Viet – Vietnam IX.25. Let 푎, 푏, 푐,푑 be non-negative real numbers such that 푎 + 푏 + 푐 + 푑 = 4. Prove that

2 + √푎 + √푏 + √푐 + √푑 ≥ 푎푏+ 푎푐 + 푎푑 + 푏푐 + 푏푑 + 푐푑 Proposed by Hung Nguyen Viet – Vietnam

IX.26. Let 푎, 푏, 푐 be non-negative real numbers such that 푎 + 푏 + 푐 = 3. Prove that

11 +23 √푎 + √푏 + √푐 ≥ 13푎푏푐

Proposed by Hung Nguyen Viet – Vietnam IX.27. Find all triads (푚,푛,푝) where 푚,푛 are two natural numbers and 푝 is a prime number satisfying the equation 푚 = 4(푝 − 1).

Proposed by Hung Nguyen Viet – Vietnam IX.28. Prove the following inequality holds for all positive real numbers 푎, 푏, 푐

푎 + 푏 + 푐 ≥12

(푎푏 + 푏푐 + 푐푎) +2(푎 + 푏 + 푐)(푎 푏 + 푏 푐 + 푐 푎 )

(푎 + 푏)(푏 + 푐)(푐 + 푎)

Proposed by Hung Nguyen Viet – Vietnam IX.29. Let 퐴퐵퐶 be a triangle with the known normal notations. Prove that for any point 푃 moving on

the incircle, 5푟 ≤ + + ≤ 푅. Proposed by Hung Nguyen Viet – Vietnam IX.30. Let 퐴퐵퐶 be a triangle with the centroid 퐺 and denote by 푆 its area. Prove that for any

point 푃 in the plane ⋅ + ⋅ + ⋅ ≥ 푆 . Proposed by Hung Nguyen Viet – Vietnam

IX.31. Let 푥,푦, 푧 be non-negative real numbers such that 푥푦 + 푦푧 + 푧푥 + 2푥푦푧 = 1. Find the maximum and minimum posible values of 푃 = 푥푦 + 푦푧 + 푧푥 + 푥 푦푧 + 푦√푧푥 + 푧 푥푦.

Proposed by Hung Nguyen Viet – Vietnam

Page 58: ROMANIAN MATHEMATICAL MAGAZINE R.M.M. · pentru ∈ z∗, pe care am rezolvat-o așa cum urmează, am păstrat algoritmul și pentru calculul altor sume, unele putând fi calculate

S.S.M.ROMÂNIA - Filiala Mehedinți 2017

58 ROMANIAN MATHEMATICAL MAGAZINE NR. 18

IX.32. Let 푎, 푏, 푐 and 푑 be real numbers such that 푎 + 푏 + 푐 + 푑 = 4 and 푎 + 푏 + 푐 + 푑 = 7. Show that ≤ 푎푏푐 + 푎푏푑 + 푎푐푑 + 푏푐푑 − 푎푏푐푑 ≤ .

Proposed by Leonard Giugiuc; Sladjan Stankovik-Macedonia IX.33. Să se arate că dacă 푎 ,푎 , … ,푎 ∈ ℝ atunci:

⎜⎛

⎟⎞≥ √푛 − 1 푎

Proposed by Daniel Sitaru – Romania IX.34. Să se arate că în triunghiul 퐴퐵퐶 este adevărată relația:

∑ √푎 퐴∑ √푎 퐴

≥∑ √푎 퐴∑ √푎퐴

Proposed by Daniel Sitaru – Romania IX.35. Să se arate că în triunghiul 퐴퐵퐶 este valabilă relația:

sin 퐴 + sin 퐵 + sin 퐶 ≤ 2 1 + cos(퐴 − 퐵) cos퐶 Proposed by Daniel Sitaru – Romania

IX.36. Să se arate că în triunghiul 퐴퐵퐶 este adevărată relația: 푎푏

푙 (푎 + 푏) cos≥ 1

Proposed by Daniel Sitaru – Romania IX.37. Să se arate că (∀)푥,푦 ∈ ℝ:

sin 푥 + sin 푦 + sin(푥 + 푦) ≤ 2 1− cos(푥 + 푦) cos(푥 − 푦) Proposed by Daniel Sitaru – Romania

IX.38. Să se rezolve ecuația: 푥 − 2 (1 + 푎 )(1 + 푏 )(1 + 푐 )푥 + (푎 + 푏 + 푐 − 푎푏푐) = 0

știind că 푎푏 + 푎푐 + 푏푐 = 1;푎, 푏, 푐 ∈ ℝ. Proposed by Daniel Sitaru – Romania

IX.39. Să se rezolve în numere reale sistemul: tg 푥+ tg푦 + tg 푧 = 6

tg푥 tg푦 + tg푥 tg 푧 + tg푦 tg푧 = 11푥 + 푦 + 푧 = 휋

Proposed by Daniel Sitaru – Romania IX.40. Să se demonstreze că:

13

+1

6 sin16

+1

3 cos>

518

Proposed by Daniel Sitaru – Romania IX.41. Să se arate că dacă 푎, 푏, 푐 ∈ (0,∞),푎 + 푏 + 푐 = 1 atunci:

푏 + 푐푎

+푎 + 푐푏

+푎 + 푏푐

푎 ∙ 푏 ∙ 푐 ≥ 2

Proposed by Daniel Sitaru – Romania IX.42. Să se determine funcțiile 푓:ℝ − {−1} → ℝ; 푛 ∈ ℕ,푛 ≥ 3 astfel încât:

2(푥 + 1)푓(3푥 − 2푥 ) + 2 푓 푥 − 푥 = 푛 − 푛 + 2

Page 59: ROMANIAN MATHEMATICAL MAGAZINE R.M.M. · pentru ∈ z∗, pe care am rezolvat-o așa cum urmează, am păstrat algoritmul și pentru calculul altor sume, unele putând fi calculate

S.S.M.ROMÂNIA - Filiala Mehedinți 2017

59 ROMANIAN MATHEMATICAL MAGAZINE NR. 18

(∀)푥 ∈ ℝ; 푖 ∈ 1,푛 Proposed by Daniel Sitaru – Romania

Clasa a X-a X.1. Rezolvați ecuația:

(1 + 3 + 5 + 7 + 9 )(2 + 4 + 6 + 8 + 10 ) = (11 + 14 )(13 + 17 )

Proposed by Nicolae Papacu- Romania

X.2. Fie numerele complexe nenule și distincte 푧 , 푘 = 1,푛,푛 ≥ 3, care verifică relațiile |푧 − 푧 | =|푧 |,푘 = 1,푛, unde 푧 = 푧 + 푧 +⋯+ 푧 . 1. Să se demonstreze că 푧 = 0.

2. Dacă 푛 = 3 și |푧 | = |푧 | = |푧 |, atunci să se arate că 푧 + 푧 + 푧 = 0, unde 푝 ∈ 푁∗ și 푝 ≡ 1 (mod 3) sau 푝 ≡ 2 (mod 3). Proposed by Nicolae Papacu- Romania

X.3. Să se arate că în orice ∆퐴퐵퐶 este valabilă relația: √ cos + √ cos + √ cos ≥

Proposed by D. M. Bătinețu – Giurgiu; Daniel Sitaru- Romania X.4. Să se arate că dacă 푎, 푏, 푐 ∈ (0,∞); 푎푏푐 = 1 și 푥,푦, 푧 ∈ 0, atunci:

1푎 (푏 + 푐) sin 2푥 cos 푥

+1

푏 (푐 + 푎) sin 2푦 cos 푦+

1푐 (푎 + 푏) sin 2푧 cos 푧

≥9√3

8

Proposed by D. M. Bătinețu – Giurgiu; Daniel Sitaru- Romania

X.5. Să se arate că în orice ∆퐴퐵퐶 are loc inegalitatea: + + ≥ .

Proposed by D. M. Bătinețu – Giurgiu; Daniel Sitaru- Romania

X.6. Fie 푚 ∈ [0,∞); 푛 ∈ ℕ∗ ∖ {1,2};푥 ∈ (0,∞); 푘 ∈ 1,푛;푆 = ∑ 푥 . Să se arate că:

푥1푥

≥ (푛 − 1)푆

Proposed by D. M. Bătinețu – Giurgiu; Daniel Sitaru- Romania

X.7. Fie 푚 ∈ [0,∞);푛 ∈ ℕ∗ ∖ {1,2};푥 ∈ (0,∞); 푘 ∈ 1,푛;푆 = ∑ 푥 . Să se arate că dacă 푥 = 푥 atunci:

푎푥 +푏

푥≥

(푎푆 + 푏 )푛 ⋅ 푆

Proposed by D. M. Bătinețu – Giurgiu; Daniel Sitaru- Romania

X.8. Fie 푛 ∈ ℕ∗ ∖ {1,2};푥 ∈ (0,∞);푘 ∈ 1,푛;푆 = ∑ 푥 Să se arate că:

Page 60: ROMANIAN MATHEMATICAL MAGAZINE R.M.M. · pentru ∈ z∗, pe care am rezolvat-o așa cum urmează, am păstrat algoritmul și pentru calculul altor sume, unele putând fi calculate

S.S.M.ROMÂNIA - Filiala Mehedinți 2017

60 ROMANIAN MATHEMATICAL MAGAZINE NR. 18

푥1푥

≥ (푛 − 1)푆

Proposed by D. M. Bătinețu – Giurgiu; Daniel Sitaru- Romania

X.9. Let a, b, c > 0, a ≤ b ≤ c, ab + bc + ca = 1. Prove that ( )

≥ a + b + c Proposed by Mihalcea Andrei Ștefan - Romania

X.10. Determinați cea mai mică valoare a funcției 푓: 0, → ℝ, 푓(푥) = 4 + 4√ .

Proposed by Dan Nedeianu- Romania

X.11. Să se demonstreze că dacă 푎 > 0,푎 ≠ 1, atunci există o infinitate de perechi de numere reale strict pozitive (푥,푦) astfel încât: a) log (푥 + 푦) = log 푥 + log 푦,

b) log (푥 + 푦) = (log 푥) ⋅ (log 푦). Proposed by Dana Heuberger- Romania

X.12. Fie triunghiul 퐴퐵퐶 și punctele 퐷,퐸,푀,푁 astfel încât 푀 e mijlocul lui (퐵퐶),푁 e mijlocul lui (퐴퐶)(퐷푀) ∩ (퐴퐵) ≠ ∅, (퐸푁)∩ (퐵퐶) ≠ ∅,퐴퐷 = 퐴퐵,퐵퐸 = 퐵퐶 și ∑퐷퐴퐵 ≡ ∑퐸퐵퐶. Să se demonstreze că 푉퐴퐷푀:푉퐵퐸푁 dacă și numai dacă ∆퐴퐵퐶 este echilateral.

Proposed by Dana Heuberger- Romania

X.14. Let 푥,푦, 푧 > 0 with 2 푥푦푧 + 푥 + 푦 + 푧 = 1. Prove that:

a. + + ≥ b. + + ≥ 12. Proposed by Mihalcea Andrei Ștefan - Romania

X.15. Let 푎, 푏, 푐 > 0, with 푎 + 푏 + 푐 = 5. Prove that: 21 + 2푎푏푐 ≥ 3(푎 푏 + 푏 푐 + 푐 푎 ).

Proposed by Mihalcea Andrei Ștefan - Romania X.16. Fie 퐴퐵퐶퐷 patrulater convex ortodiagonal inscriptibil, 퐴퐶 ∩ 퐵퐷 = {0}

Demonstrați că: ⋅⋅

≤ ⋅ ⋅⋅ ⋅

≤ + + +

Proposed by Mihalcea Andrei Ștefan - Romania

X.17. Let 푎, 푏, 푐 > 0, where 푎, 푏, 푐 are the sides of 훥퐴퐵퐶, 훥 = area of 훥퐴퐵퐶, 푠 = semiperimeter.

Prove that: ∑ [1 + (ℎ +푤 + 푚 )(2 +푤 )]( ) ≥ 3 + 3 .

Proposed by Mihalcea Andrei Ștefan - Romania

X.18. Să se rezolve ecuația: 푝 + (푝 + 1) +⋯+ (푝 + 푛 − 1) = (푛 − 1)푝 ; 푛 ∈ ℕ;푛 ≥ 2;푝 > 0 Proposed by Claudia Nănuți – Romania

X.19. Să se rezolve sistemul: 2 = 3(3푥) = (2푦)

Proposed by Claudia Nănuți – Romania

Page 61: ROMANIAN MATHEMATICAL MAGAZINE R.M.M. · pentru ∈ z∗, pe care am rezolvat-o așa cum urmează, am păstrat algoritmul și pentru calculul altor sume, unele putând fi calculate

S.S.M.ROMÂNIA - Filiala Mehedinți 2017

61 ROMANIAN MATHEMATICAL MAGAZINE NR. 18

X.20. Să se arate că în orice 훥퐴 퐴 퐴 avem:

4 cos퐴2

cos퐴2

cos퐴2− 3 sin퐴 sin퐴 sin퐴 ≥ max sin퐴 − sin퐴

Proposed by Daniel Sitaru, Claudia Nănuți – Romania X.22. Se dă progresia aritmetică: (푎 ) ;푎 = 3;푎 = 푎 + 4. Determinați 푞 ∈ ℕ∗ astfel încât progresia geometrică: (푏 ) ; 푏 = 3;푏 = 푏 ⋅ 푞, să aibă toți termenii printre cei ai progresiei aritmetice.

Proposed by Constantin Giugiuc – Romania X.23. Să se arate că dacă 푎, 푏, 푐 > 0 atunci:

(푎 + 푎 + 1)(푏 + 푏 + 1)(푐 + 푐 + 1) ≥ 27푎푏푐 Proposed by Dan Nănuți – Romania

X.24. Să se arate că dacă 푎, 푏, 푐 > 0 atunci: 푎푏

+푏푎

+ 1푏푐

+푐푏

+ 1푐푎

+푎푐

+ 1 ≥ 27푎푏푐

Proposed by Dan Nănuți – Romania X.25. Fie 퐴퐵퐶퐷 un patrulater inscriptibil și circumscriptibil, 푝 semiperimetrul acestuia, 푅 și 푟 razele cercurilor circumscris, respectiv înscris, 푎, 푏, 푐,푑 laturile acestuia (푎 și 푐 sunt laturi opuse). Demonstrați că:

a) 2 ≥ + + + ≥ 2√2 , b) 4 − 4 ≥ + + Proposed by Vasile Jiglău – Romania

X.26. Let 퐴퐵퐶퐷 be a triangle. Prove that

2(푎푏 + 푏푐 + 푐푎)− (푎 + 푏 + 푐 ) ≥ 2 푎푏 sin퐶2

+ 푏푐 sin퐴2

+ 푐푎 sin퐵2

≥ 6푆4푅푝≥ 4푆√3

Proposed by Vasile Jiglău – Romania X.27. Prove that if 푥,푦, 푧 > 0, 푥푦푧 = 8 then:

푥 + 푦 + 푧 ≥ 2푥 푦 + 푧 + 2푦√푧 + 푥 + 2푧 푥 + 푦 Proposed by Iuliana Trașcă – Romania

X.28. Let 푥,푦, 푧 be positive real numbers such that: 16(푎 + 푏 + 푐 ) + 27 = 128푎푏푐. Find the maximum value of the expression:

퐸 =1

푎 + 푏 ++

1

푏 + 푐 ++

1

푐 + 푎 +

Proposed by Iuliana Trașcă and Neculai Stanciu – Romania X.29. Let 훼,훽, 푥,푦, 푧 > 0. Prove that:

3 푥2016훼푦+ 훽푧

+3 푦

2016훼푧 + 훽푥+

3 푧2016훼푥 + 훽푦

≥2016

훼 + 2016 훽.

Proposed by Iuliana Trașcă – Romania X.30. Fie 푥 ∈ (−∞, 2016). Să se verifice inegalitatea:

√2016− 푥 ≤4031− 푥

2016

Proposed by Ovidiu Ticuși – Romania

X.31. Să se rezolve în 푅 sistemul: =3 + 4 = 19

Proposed by Carmen – Victorița Chirfot – Romania

Page 62: ROMANIAN MATHEMATICAL MAGAZINE R.M.M. · pentru ∈ z∗, pe care am rezolvat-o așa cum urmează, am păstrat algoritmul și pentru calculul altor sume, unele putând fi calculate

S.S.M.ROMÂNIA - Filiala Mehedinți 2017

62 ROMANIAN MATHEMATICAL MAGAZINE NR. 18

X.32. Fie 훥퐴퐵퐶 ascuțitunghic. Să se arate că: 푚 + sin 2퐴 ;푚 + sin 2퐵 ,푚 + sin 2퐶 pot fi lungimile laturilor unui triunghi.

Proposed by Leonard Giugiuc, Diana Trăilescu – Romania X.33. Se dă un triunghi ascuțitunghic 퐴퐵퐶 de arie și ortocentru 퐻. Să se calculeze:

퐴퐻 ⋅ 퐵퐶 + 퐵퐻 ⋅ 퐴퐶 + 퐶퐻 ⋅ 퐴퐵 Proposed by Leonard Giugiuc, Diana Trăilescu – Romania

X.34. Prove that in any triangle: 푎

푏 + 푐+ 푛 ⋅

푎푏 + 푏푐 + 푐푎푎 + 푏 + 푐

≤ 푛 +32

,푛 ≥ 1

Proposed by Marin Chirciu – Romania X.35. Să se calculeze 0! 1 + 1! 2 + 2! 3 + 3! 4 + 4! 5 + ⋯+ 푛! (푛 + 1) ,푛 ∈ ℕ∗.

Poposed by Carmen Victorița – Chirfot – Romania

X.36. a) Să se demonstreze că 1 + + ( ) = ( ) ,푛 ∈ ℕ∗.

b) Să se calculeze 1 + + + 1 + + +⋯+ 1 + + .

Poposed by Carmen Victorița – Chirfot – Romania X.37. Prove that if 푎, 푏, 푐,푑, 푥,푦, 푧 ∈ (0,∞):

푎 + 푏 + 푐 + 푑 = 푥 + 푦 + 푧 푎 + 푏 + 푐 + 푑 = 푥 + 푦 + 푧 then 푎 + 푏 + 푐 + 푑 ≥ 푥 + 푦 + 푧

Proposed by Richdad Phuc – Hanoi – Vietnam X.38. Solve the equation: 푒 + 푒 = 2푥 + 푦 + 2017

Proposed by Rovsen Pirguliev - Azerbaidjian X.39. In triangle 퐴퐵퐶∢퐶 = 90° and ∢퐴 = 훼. Prove that:

√> cos(훼 − 45°)

Proposed by Rovsen Pirguliev - Azerbaidjian

X.40. In the triangle prove the inequality: 푎푏푐 ≤ 4푅푠 푚 + 푚 +푚

Proposed by Rovsen Pirguliev - Azerbaidjian X.41. In all triangle 퐴퐵퐶 holds: = ⋅ − 1 − 1 − 1

Proposed by Rovsen Pirguliev - Azerbaidjian X.42. In all triangle 퐴퐵퐶 holds: 푝 + 푟 + 4푅푟 ≥ 4√3푆

Proposed by Rovsen Pirguliev - Azerbaidjian X.43. Let 퐴퐵퐶 be a triangle with the area 푆 and denote by 푟, 푟 , 푟 , 푟 inradii, exradii respectively. Prove that

(푟 + 푟 푟 )(푟 + 푟 푟 )(푟 + 푟 푟 ) ≥103

(푟푆)

Proposed by Hung Nguyen Viet – Vietnam X.44. Let 푥 , 푥 , … , 푥 (푛 ≥ 2) be positive real numbers such that 푥 + 푥 +⋯+ 푥 ≤ 푥 Find the minimum value of 퐸 = 푥 + 푥 + ⋯+ 푥 + + ⋯+ where 푘 is a positive

integer. Proposed by Hung Nguyen Viet – Vietnam

X.45. Prove that the following inequality holds for all real numbers 푎, 푏, 푐,푑 (6푎+ 7푏 + 8푐 + 9푑) ≥ 140(푎푏 + 푎푐 + 푎푑 + 푏푐 + 푏푑 + 푐푑). When does the equality occurs?

Proposed by Hung Nguyen Viet – Vietnam X.46. Solve in the set of real numbers the following equation

Page 63: ROMANIAN MATHEMATICAL MAGAZINE R.M.M. · pentru ∈ z∗, pe care am rezolvat-o așa cum urmează, am păstrat algoritmul și pentru calculul altor sume, unele putând fi calculate

S.S.M.ROMÂNIA - Filiala Mehedinți 2017

63 ROMANIAN MATHEMATICAL MAGAZINE NR. 18

2푥 − 푥 + 13푥 + 2푥 + 1

=23,

where {푎} = 푎 − ⌊푎⌋ denotes fractional part of 푎. Proposed by Hung Nguyen Viet – Vietnam

X.47. Prove that in any acute triangle 퐴퐵퐶: √sin 2퐵 sin 2퐶

cos 퐴+√sin 2퐶 sin 2퐴

cos퐵+√sin 2퐴 sin 2퐵

cos 퐶≥ 3√3

Proposed by Hung Nguyen Viet – Vietnam X.48. Let 퐴퐵퐶 be a triangle inscribed in the circle (푂,푅) and circumscribed out the circle (퐼, 푟). 퐼 , 퐼 , 퐼 are excenters of 퐴,퐵,퐶, respectively. Prove that. a. 2퐵퐶√2 ≤ 퐼퐼 + 퐼 퐼 ≤ 4푅√2

b. √≤ + ≤ √

Proposed by Hung Nguyen Viet – Vietnam X.49. Find all pairs (푥,푦) of integers satisfying the following inequality

(3푥 + 2푥 + 1)(푥 + 5푦 − 4푥푦 + 2푥 − 6푦+ 1) ≤ 4푥 + 3푥 + 2 Proposed by Hung Nguyen Viet – Vietnam

X.50. Determine measurments of three angles of triangle 퐴퐵퐶 satisfying

4(cot퐴+ cot퐵 + cot퐶) =√6 + √2

sin퐴+

2√2sin퐵

−2

sin 퐶

Proposed by Hung Nguyen Viet – Vietnam X.51. Let 퐴퐵퐶 be a triangle with the area 푆 and known familiar notations. Prove that:

(푟 + 푟 + 푟 )(푚 + 푚 + 푚 ) ≥ 9√3푆 Proposed by Hung Nguyen Viet – Vietnam

X.52. Prove that in any triangle 퐴퐵퐶:

(푎 + 푏 + 푐) ⋅푂퐼푅

= 푎 ⋅ cos(∠퐴푂퐼) + 푏 ⋅ cos(∠퐵푂퐼) + 푐 ⋅ cos(∠퐶푂퐼)

Proposed by Hung Nguyen Viet – Vietnam X.53. Let 푎, 푏, 푐,푑 be positive real numbers. Prove that:

√푎 + √푑 √푏 + √푐 ≤ √푎 + 푏 + 푐 √푎 + 푏 + 푑 + √푎 + 푐 + 푑 Proposed by Hung Nguyen Viet – Vietnam

X.54. Let 푎, 푏, 푐 be non-negative real numbers. Prove that:

9(푎 + 푏 + 푐) ≥푎 + 푏 + 푐

3+ 26

푎푏 + 푏푐 + 푐푎3

Proposed by Hung Nguyen Viet – Vietnam X.55. Prove that in any triangle the following relationship holds:

≤ ( )⋅ , where 0 ≤ 푛 ≤ .

Proposed by Marin Chirciu – Romania X.56. Let 푚 ,푚 ,푚 be the lengths of the medians of a triangle 퐴퐵퐶. Prove that

94푅 + 푟

≤1푚

+1푚

+1푚

≤1푟

,

where 푅 and 푟 are the circumradius and inradius of 퐴퐵퐶 respectively. Proposed by Martin Lukarevski – Stip – Macedonia

X.57. Prove that if 푎, 푏, 푐,푑 ∈ ℝ;푎 + 푏 ≠ 0; 푐 + 푑 ≠ 0 then:

Page 64: ROMANIAN MATHEMATICAL MAGAZINE R.M.M. · pentru ∈ z∗, pe care am rezolvat-o așa cum urmează, am păstrat algoritmul și pentru calculul altor sume, unele putând fi calculate

S.S.M.ROMÂNIA - Filiala Mehedinți 2017

64 ROMANIAN MATHEMATICAL MAGAZINE NR. 18

(푎푑 − 푏푐)(3(푎 + 푏 )(푐 + 푑 ) − 4(푎푑 − 푏푐) )

(푎 + 푏 )(푐 + 푑 )≤ 1

Proposed by Daniel Sitaru – Romania

X.58. Let 퐴퐵퐶 be an arbitrary triangle and 푋푌푍 is the Kiepert triangle of 퐴퐵퐶. If 퐾(휃) is a Kiepert perspector and 휔 is first Brocard angle then prove that a) ( )

( ) = (3 tan 휃 + 2 tan휃 ⋅ cot휔 + 1)

b) if 휃 = 휔 then 푋푌푍 is Gallatly – Kiepert triangle takes the name. Prove that

퐴푟푒푎(푋푌푍)퐴푟푒푎(퐴퐵퐶) = 3 ⋅

푎 푏 + 푏 푐 + 푐 푎(푎 + 푏 + 푐 )

Proposed by Mehmet Șahin – Ankara – Turkey

X.59. Let 푥 and 푦 be complex numbers such that |푥 + 푦| = 16 and

|푥 + 푦 | = 20. Find the extrema of |푥 + 푦 |. Proposed by Leonard Giugiuc-Romania

Clasa a XI-a

XI.1. Fie matricele 퐴,퐵 ∈ 푀 (ℝ). 1. Să se determine 푎, 푏 ∈ ℝ astfel încât 퐴퐵 = 푎 4

3 7 și

퐵퐴 = 2 34 푏 . 2. Dacă 퐴퐵 = 퐵퐴 și 퐴 + 퐵 − 퐴퐵 + 퐼 = 푂 să se arate că det(퐴 + 퐵) ≥ 0.

Proposed by Nicolae Papacu- Romania

XI.2. Fie matricele inversabile 퐴,퐵 ∈ (ℝ)− {퐼 } și 푝 ≥ 1 un număr natural nenul. Dacă 퐵 = 퐼 și 퐴퐵 = 퐵 퐴, arătați că 1. 퐵 = 퐼 . 2. 퐴퐵 = 퐵퐴 dacă și numai dacă 3|(푝 − 2).

Proposed by Nicolae Papacu- Romania

XI.3. Find:

훺 = lim→

1푖(푖 + 1)(푖 + 2)(푖 + 3)

Proposed by Daniel Sitaru – Romania

XI.4 Find:

훺 = lim→∞

푛∫ √푥 푑푥

∫ √푥 푑푥, [∗] − 푔푟푒푎푡푖푛푡푒푔푒푟푓푢푛푐푡푖표푛

Proposed by Daniel Sitaru – Romania

Page 65: ROMANIAN MATHEMATICAL MAGAZINE R.M.M. · pentru ∈ z∗, pe care am rezolvat-o așa cum urmează, am păstrat algoritmul și pentru calculul altor sume, unele putând fi calculate

S.S.M.ROMÂNIA - Filiala Mehedinți 2017

65 ROMANIAN MATHEMATICAL MAGAZINE NR. 18

XI.5. Find:

훺 = lim→∞

Proposed by Daniel Sitaru – Romania

XI.6. Calculate:

훺 = lim→∞

⎩⎨

⎧ 푛 + 11

푛 + 12

… 푛 + 1푛 + 1

푒( )

(푛 + 1)−

푛1

푛2 … 푛

푒 푛⎭⎬

Proposed by Soumitra Mandal –Chandar Nagore – India

XI.7. If 푎, 푏, 푐,푑 ∈ ℝ, 푎 ≤ 푏 ≤ 푐 ≤ 푑 then: 푒 − 푒 + 푒 − 푒 ≥ 2 √푒 − √푒 .

Proposed by Daniel Sitaru – Romania

XI.8. Fie (푏 ) ; 푏 = ∏ (2푘 − 1)‼ . Să se calculeze:

퐿 = lim→

푛푏

Proposed by D. M. Bătinețu – Giurgiu; Daniel Sitaru- Romania XI.9. Dacă 퐴 ∈ ℳ (ℝ),푇푟퐴 = (푇푟퐴) ≠ 0, să se arate că matricea 퐴 este inversabilă.

Proposed by Dan Nedeianu- Romania XI.10. Fie (푎 ) ⊆ 푅 un șir astfel încât:

lim→

푎 = 1; lim→

푎 = 2; lim→

푎 = 3 Să se calculeze:

lim→

푎 + 푎 + ⋯+ 푎푛

Proposed by Diana Trăilescu – Romania XI.11. Fie 푎 ∈ ℝ ∖ {−1}. Să se studieze convergența șirului (푥 ) definit prin:

푥 =푎

1 + 푎;푛 ∈ ℕ

Proposed by Gheorghe Căiniceanu – Romania XI.12. Fie 퐴 ∈ 푀 (ℂ); 퐴 ≠ 푂 ; det퐴 = 푇푟퐴 = 0. Să se arate că ecuația 푥 = 퐴 nu are soluții în 푀 (ℂ).

Proposed by Dorel Căpraru – Romania XI.13. Fie 퐴 ∈ 푀 (ℝ); det퐴 = 1;푇푟퐴 = 푡 ∈ ℝ. Să se calculeze 푇푟(퐴 ) în funcție de 푡.

Proposed by Diana Trăilescu – Romania

Page 66: ROMANIAN MATHEMATICAL MAGAZINE R.M.M. · pentru ∈ z∗, pe care am rezolvat-o așa cum urmează, am păstrat algoritmul și pentru calculul altor sume, unele putând fi calculate

S.S.M.ROMÂNIA - Filiala Mehedinți 2017

66 ROMANIAN MATHEMATICAL MAGAZINE NR. 18

XI.14. Fie 휎 ∈ 푆 . Să se verifice inegalitatea: [휎(푖) − 2016] ⋅ [휎(푗)− 2016] ≥ 0,∀푖, 푗 ∈ {1,2, … ,2016}

Proposed by Ovidiu Ticuși – Romania XI.15. Să se arate că orice primitivă a funcției 푓:ℝ → ℝ,

푓(푥) = (푒 − 푥 + 2016)푒 log (푥 − 1)este injectivă. Proposed by Ovidiu Ticuși – Romania

XI.16. Fie 퐴 ∈ 푀 (ℂ); det퐴 = 푇푟퐴 = 0. Să se arate că există 푘 ∈ ℂ încât 퐴 = 푘퐴. Proposed by Leonard Giugiuc, Diana Trăilescu – Romania

XI.17. Fie 퐴 ∈ 푀 (ℂ); 푟푎푛푔(퐴퐵 − 퐵퐴) = 1. Se cere (퐴퐵 − 퐵퐴) . Proposed by Leonard Giugiuc, Diana Trăilescu – Romania

XI.18. Fie 퐴 ∈ 푀 (ℂ); 퐴 =1 휀 휀휀 1 휀휀 휀 1

; 휀 = − + 푖 √ . Dacă 퐴 =푎 푏 푐푐 푎 푏푏 푐 푎

; 푛 ≥ 1 să

se calculeze 푎 + 푏 휀 + 푐 휀 . Proposed by Leonard Giugiuc, Diana Trăilescu – Romania

XI.19. Să se calculeze: lim → ln푛 + ln푝 − ln푛 + ln푞 , unde 푝, 푞 ∈ 푅∗ . Poposed by Carmen Victorița – Chirfot – Romania

XI.20. Să se calculeze: lim→

ln푛 + ln푝, unde 푝 ∈ 푅∗ .

Poposed by Carmen Victorița – Chirfot – Romania

XI.21. Să se rezolve ecuația 2 √푥 − 1 = 푥 − 2√푥 + 1, unde 푥 ∈ 푅 . Poposed by Carmen Victorița – Chirfot – Romania

XI.22. Dacă 푏 ⊂ (0,∞);푛 ≥ 1 încât:

lim→

푏푛푏

= 푏 ∈ (0,∞)

să se calculeze:

훺 = lim→

푛 + 1푏

−푛푏

⋅ √푛

Proposed by D.M. Bătinețu – Giurgiu, Neculai Stanciu – Romania

XI.23. Dacă 푥 ⊂ (0,∞); 푛 ≥ 1 încât

lim→

푥푛푥

= 푥 ∈ (0,∞)

să se calculeze: 훺 = lim

→푥 − 푥 ⋅ 푛

Proposed by D.M. Bătinețu – Giurgiu, Neculai Stanciu – Romania

XI.24. Dacă 푎 ⊂ (0,∞); 푛 ≥ 1 încât:

lim→

푎푎 ⋅ 푛

= 푎 ∈ (0,∞)

să se calculeze:

훺 = lim→

(푛 + 1)!푎 − 푛! ⋅ 푎 tg휋푎

Proposed by D.M. Bătinețu – Giurgiu, Neculai Stanciu – Romania

Page 67: ROMANIAN MATHEMATICAL MAGAZINE R.M.M. · pentru ∈ z∗, pe care am rezolvat-o așa cum urmează, am păstrat algoritmul și pentru calculul altor sume, unele putând fi calculate

S.S.M.ROMÂNIA - Filiala Mehedinți 2017

67 ROMANIAN MATHEMATICAL MAGAZINE NR. 18

XI.25. Să se calculeze:

훺 = lim→

(2푛 − 1)‼(2푛 + 1)‼푛 + 1

−(2푛 − 1)‼

Proposed by D.M. Bătinețu – Giurgiu, Neculai Stanciu – Romania XI.26. Să se calculeze:

훺 = lim→

√푛!(2푛 + 1)‼

(푛 + 1) −(2푛 − 1)‼푛

Proposed by D.M. Bătinețu – Giurgiu, Neculai Stanciu – Romania XI.27. Dacă (푥 ) ; 푥 = ∑ să se calculeze:

훺 = lim→

(2푛 + 1)‼ − (2푛 − 1)‼ ⋅ 푒 ⋅ sin휋√푛!

Proposed by D.M. Bătinețu – Giurgiu, Neculai Stanciu – Romania XI.28. If 푎, 푏, 푐 ∈ (0, +∞), then:

푎 푐푏

≥푎푐푏

Proposed by Daniel Sitaru – Romania XI.29. Compute:

lim→

tan1

푛 + 푖− tan

1푛 + 푖 + 1

cos1

푛 + 1+ cos

1푛 + 2

+⋯+ cos1

푛 + 푖

Proposed by Daniel Sitaru – Romania XI.30. Compute:

lim→

(1 + 2 + ⋯+ 푘 )푛 (1 + 2 +⋯+ 푛 ) ; 푝 ∈ ℕ

Proposed by Daniel Sitaru -Romania XI.31. If 푎, 푏, 푐, 휂 ∈ ℝ then: |(푎 − 푏)(푏 − 푐)(푐 − 푎)| ≤ ∑|(푎 − 푏)(푐 + 푏 + 휂)(푐 + 푎 + 휂)|

Proposed by Daniel Sitaru -Romania XI.32. Find 푥 ∈ ℝ such that: 퐴 √ , √ , ,퐵 √ , , √ , 퐶 , , √ ,퐷 0, , 푥 are cospherical points. Proposed by Daniel Sitaru -Romania

XI.33 If 푥,푦, 푧, 푡,푎, 푏, 푐 ∈ (0,∞), 푥푦푧푡 = 푎 then: 3푎∑ ≥ 4 푎 + 푎 .

Proposed by Daniel Sitaru -Romania

XI.34. Fie 퐴,퐵 ∈ 푀 (푅), două matrice care verifică relația 퐴 ⋅ 퐵 = 퐵 ⋅ 퐴. Să se arate că:

det (퐴 + 퐵) + 5det 퐴 + det (퐴− 퐵) + 5det 퐴 ≥ 3|det퐴+ det퐵|

Arătați că egalitatea are loc pentru o infinitate de matrice 퐴,퐵 ∈ 푀 (푅).

Proposed by Gheorghe Alexe; George – Florin Șerban – Romania

XI.35. Let 퐴퐵퐶 be a triangle such that: + (− sin퐴 + sin퐵 + sin 퐶) = 2.

Prove that ∢퐴 ≤ . Proposed by George Apostolopoulos – Messolonghi - Greece

Page 68: ROMANIAN MATHEMATICAL MAGAZINE R.M.M. · pentru ∈ z∗, pe care am rezolvat-o așa cum urmează, am păstrat algoritmul și pentru calculul altor sume, unele putând fi calculate

S.S.M.ROMÂNIA - Filiala Mehedinți 2017

68 ROMANIAN MATHEMATICAL MAGAZINE NR. 18

XI.36. Let 푠 is symedian and 푟 , 푟 are exradius and inradius triangle of 퐴퐵퐶 respectively. Prove

that: + + ≥ . Proposed by Mehmet Șahin – Ankara – Turkey

XI.37. Let 푎, 푏, 푐 be positive real numbers such that 푎 + 푏 + 푐 = 3. Prove that:

+ + ≥ . Proposed by George Apostolopoulos – Messolonghi - Greece

XI.38. Let 푚 ,푚 ,푚 be the lengths of medians of a triangle 퐴퐵퐶 with inradius 푟. Prove that:

≥ 4푟. Proposed by George Apostolopoulos – Messolonghi - Greece

XI.39. Let 푎, 푏 and 푐 be the lengths of the sides of a triangle with circumradius 푅. Prove that:

+ + ≤ √ 푅. Proposed by George Apostolopoulos – Messolonghi - Greece

XI.40. Let 퐴퐵퐶 be an arbitrary triangle and 퐼 , 퐼 , 퐼 are excenters of 퐴퐵퐶. 퐼 퐵퐶, 퐼 퐶퐴, 퐼 퐴퐵 are the

extriangles of 퐴퐵퐶. Let ℎ (푖 = 1,2,3, … ,9) the altitudes of extriangles. Prove that:

ℎ = 푟, ,

where 푟 , 푟 , 푟 are exradii of 퐴퐵퐶. Proposed by Mehmet Șahin – Ankara – Turkey

XI.41. Let 푎, 푏, 푐 be the side lengths of a triangle 퐴퐵퐶 with inradius 푟. Prove that:

1푎

tan퐴2

+1푏

tan퐵2

+1푐

tan퐶2≤

푅48푟

Proposed by George Apostolopoulos – Messolonghi - Greece

XI.42. Prove that for all 푥 ∈ ℝ we have: cos(sin 푥) > |sin(cos 푥)|.

Proposed by Abdallah El Farisi – Bechar – Algerie

XI.43. Let 훼 and 훽 be positive real numbers with 훼 > 훽. Let 푎, 푏 and 푐 be real numbers such that

푎 + 푏 + 푐 = 2훼 + 훽 and 푎푏푐 = 훼 훽. Find the range of 푎 + 푏 + 푐.

Proposed by Leonard Giugiuc-Romania XI.44. Let 훼 and 훽 be positive real numbers with 훼 > 훽.Let 푎, 푏 and 푐 be positive real numbers such

that 푎 + 푏 + 푐 = 2훼 + 훽 and 푎푏푐 = 훼 훽. Find the range of 푎 + 푏 + 푐.

Proposed by Leonard Giugiuc-Romania XI.45. Fie 푥,푦, 푧 ∈ (1,∞). Să se arate că dacă 푥 + 푦 + 푧 = 3휋 atunci:

푥 + 푦 + 푧 > 81 Proposed by Daniel Sitaru – Romania

XI.46. Să se demonstreze că: √2 + 2 √6 < √4 + 2 √5

Proposed by Daniel Sitaru – Romania

Page 69: ROMANIAN MATHEMATICAL MAGAZINE R.M.M. · pentru ∈ z∗, pe care am rezolvat-o așa cum urmează, am păstrat algoritmul și pentru calculul altor sume, unele putând fi calculate

S.S.M.ROMÂNIA - Filiala Mehedinți 2017

69 ROMANIAN MATHEMATICAL MAGAZINE NR. 18

XI.47. Să se calculeze:

lim→

7푘 + 푘 + 3푘 + 1푛 + 푘 + 5

Proposed by Daniel Sitaru – Romania XI.48. Să se arate că dacă 푥,푦, 푧 ∈ 0, atunci:

(tg푥 + 2 sin 푥) > 3(푥 + 푦 + 푧)

Proposed by Daniel Sitaru – Romania XI.49. Să se arate că în orice triunghi ascuțitunghic 퐴퐵퐶 este valabilă relația:

cos퐴 + 4 cos퐵 + 4 sin퐶2≤ 9 cos

휋 + 퐵 − 퐶3

Proposed by Daniel Sitaru – Romania XI.50. Să se arate că dacă 푥,푦, 푧 ∈ (0,∞) și 푥 + 푦 + 푧 = 1 atunci:

푥 + 푦 + 푧 +1

27≥

18

[(1 − 푥) + (1− 푦) + (1− 푧) ]

Proposed by Daniel Sitaru – Romania XI.51. Să se demonstreze că dacă 푥,푦, 푧 ∈ (−∞, 0) atunci:

1ln(1− 푥) +

1ln(1− 푦) +

1ln(1− 푧) < 3−

1푥−

1푦−

1푧

Proposed by Daniel Sitaru – Romania XI.52. Să se arate că (∀)푥,푦, 푧 ∈ (0,1)

푒 + 푒 + 푒 <1

(1− 푥)(1− 푦) +1

(1− 푦)(1− 푧) +1

(1− 푧)(1− 푥)

Proposed by Daniel Sitaru – Romania XI.53. Să se arate că dacă 푎, 푏, 푐 ∈ (0,∞) atunci:

푎 < 3 +32

12√푎

Proposed by Daniel Sitaru – Romania XI.54. Să se arate că dacă 푥,푦, 푧 ∈ (0,∞) atunci:

(푥 + 푦) < 1 +32

1푥 + 푦

Proposed by Daniel Sitaru – Romania XI.55. Să se arate că dacă 푥,푦, 푧 ∈ (0,∞);푥 + 푦 + 푧 = 1 atunci:

푒 − 2 푒 + 3√푒 ≥ 0

Proposed by Daniel Sitaru – Romania XI.56. Să se demonstreze că:

ln 3 + ln 4 + ln 5 +103

ln 3 ln 5 ≤1

15ln(3 ∙ 5 ) ln 3 ln 4 ln 5

Proposed by Daniel Sitaru – Romania XI.57. Să se calculeze:

퐿 = lim→

2√4√5 + √3

Proposed by Daniel Sitaru – Romania

Page 70: ROMANIAN MATHEMATICAL MAGAZINE R.M.M. · pentru ∈ z∗, pe care am rezolvat-o așa cum urmează, am păstrat algoritmul și pentru calculul altor sume, unele putând fi calculate

S.S.M.ROMÂNIA - Filiala Mehedinți 2017

70 ROMANIAN MATHEMATICAL MAGAZINE NR. 18

XI.58. Să se calculeze:

퐿 = lim→

√2 ∙ √3! ∙ √4! ∙ … ∙ √푛!(푛 + 1)!

Proposed by Daniel Sitaru – Romania XI.59. Să se arate că:

tg12

+ tg13

+ ⋯+ tg1

10< 1 +

12

+ ⋯+19

Proposed by Daniel Sitaru – Romania XI.60. Să se calculeze:

퐿 = lim→

1푛

푘∑ 푗

Proposed by Daniel Sitaru – Romania XI.61. Fie 퐴,퐵 ∈ 푀 (ℂ); det(−2퐴 + 퐵) = 0; det(2퐴 + 퐵) = 0. Să se calculeze: 4 det퐴 + det퐵.

Proposed by Daniel Sitaru – Romania XI.62. Să se demonstreze că:

1휋

+1푒

+1훾

>13

+12

+1

휋 + 훾 + 푒 − 5

unde

훾 = lim→

1 +12

+ ⋯+1푛− ln푛

Proposed by Daniel Sitaru – Romania XI.63. Să se arate că dacă 푥,푦, 푧 ∈ (0,∞); 푥 + 푦 + 푧 = 1 atunci:

푥푦 2 − 1 > ln 2

Proposed by Daniel Sitaru – Romania XI.64. Să se calculeze:

퐿 = lim→

푛 −arctg

arctg

Proposed by Daniel Sitaru – Romania

Clasa a XII-a

XII.1. Fie 푝 ≥ 2 un număr natural și șirul (퐼 ) , 퐼 = ∫ ⋅

⋯ √

푑푥 . a. Calculați 퐼 ,푛 ≥ 1

b. Calculați lim → lim → 퐼 . Proposed by Nicolae Papacu- Romania

XII.2. Fie (퐴, +,⋅) un inel cu 1 ≠ 0. Dacă 푥,푦 ∈ 퐴 astfel încât 푥 + 푦 = 1 și 푥 = 푥 să se arate că elementele 1 − 푥푦 și 1 − 푦푥 sunt inversabile.

Proposed by Nicolae Papacu- Romania

Page 71: ROMANIAN MATHEMATICAL MAGAZINE R.M.M. · pentru ∈ z∗, pe care am rezolvat-o așa cum urmează, am păstrat algoritmul și pentru calculul altor sume, unele putând fi calculate

S.S.M.ROMÂNIA - Filiala Mehedinți 2017

71 ROMANIAN MATHEMATICAL MAGAZINE NR. 18

XII.3. Find: 훺 = ∫ 푥 + √1 + 푥 푑푥, 푥 ∈ ℝ

Proposed by Nishant Kumar – Jamshedpur – India

XII.4. Se consideră funcția 푓: (0,∞) → ℝ, 푓(푥) = ( ) −( ). Să se calculeze ∫|푓(푥)|푑푥.

Proposed by Dan Nedeianu- Romania

XII.5. Fie inelul (퐴, +,⋅). Dacă există 푘 ∈ ℕ∗ astfel încât pentru orice 푎, 푏 ∈ 퐴 avem

(푎 + 푏) = 푎 + 푏 și (푎 + 푏) = 푎 + 푏 , atunci arătați că inelul este comutativ.

Proposed by Dana Heuberger- Romania

XII.7. If 푎 ∈ 0, and 푓:푅 → 푅 is a continuous and odd function, then compute

(푥 + 푥 + 82) ⋅ arccos sin 푓(푥) 푑푥.

D. M. Bătinețu – Giurgiu, Neculai Stanciu - Romania XII.8. Fie 푎 ∈ ℝ∗ și 푓:ℝ → ℝ o funcție cu proprietatea că 푔:ℝ → ℝ; 푔(푥) = (푥 + 푎)푓(푥) și ℎ:ℝ → ℝ ℎ(푥) = 푥푓(푥 + 푎) admit primitive pe ℝ. Să se arate că: a. funcția ℎ ∘ 푣 admite primitive pe ℝ, unde 푣:ℝ → ℝ; 푣(푥) = 푥 − 푎 b. funcția 푓 admite primitive pe ℝ

Proposed by Gheorghe Căiniceanu – Romania XII.9. Fie (퐴, +,⋅) un inel cu patru elemente în care 푥 + 푥 = 0, (∀)푥 ∈ 퐴. Să se arate că (퐴, +,⋅) este corp comutativ.

Proposed by Diana Trăilescu – Romania XII.10. Să se calculeze:

퐼 = (sin 2푥 + cos 2푥) ln(1 + tg푥)푑푥

Proposed by Viorel Sahagia – Romania XII.11. Pe mulțimea 푀 (ℂ); 푛 ∈ ℕ∗ se definesc legile de compoziție:

퐴 ∗ 퐵 = 퐴+ 퐵 − 2017퐼 ;퐴 ∘ 퐵 = 퐴 + 퐵 + 2018퐼 (∀)퐴,퐵 ∈ 푀 (ℂ). Să se arate că (푀 (ℂ),∗) și (푀 (ℂ),∘) sunt grupuri abeliene izomorfe.

Proposed by Claudia Nănuți – Romania XII.12. Fie 퐺 = {푓 :ℝ → ℝ|푓 (푥) = 푥 + 푡; 푡 ∈ ℝ}, 퐻 = {푓 :ℝ → ℝ|푓 (푥) = 푥푡; 푡 ∈ ℝ∗} Să se arate că (퐺,∘) și (퐻,∘) sunt grupuri abeliene neizomorfe.

Proposed by Daniel Sitaru, Claudia Nănuți – Romania

XII.13. Să se calculeze: ∫ 2 푑푥 Proposed by Leonard Giugiuc, Diana Trăilescu – Romania

XII.14. Dacă 푎, 푏 ∈ ℝ,푎 < 푏 să se calculeze: lim → ∫ (sin 푥)(arctg(푛푥))푑푥

Proposed by Dan Nedeianu – Romania

Page 72: ROMANIAN MATHEMATICAL MAGAZINE R.M.M. · pentru ∈ z∗, pe care am rezolvat-o așa cum urmează, am păstrat algoritmul și pentru calculul altor sume, unele putând fi calculate

S.S.M.ROMÂNIA - Filiala Mehedinți 2017

72 ROMANIAN MATHEMATICAL MAGAZINE NR. 18

XII.15. Prove that:

1√2

≤ 푥 + √푥 푑푥 ≤4√2

5

Proposed by Rovsen Pirguliev – Azerbaidjian

XII.16. If 푎, 푏 ∈ ℝ;푎 < 푏;푓,푔: [푎, 푏] → ℝ are continuous functions such that: 푓(푎+ 푏 − 푥) = 푓(푥),푔(푎 + 푏 − 푥)푔(푥) = 1, (∀)푥 ∈ [푎, 푏] then:

푓(푥) ⋅ 푔(푥)1 + 푔(푥) 푑푥 =

12

푓(푥)푑푥

Proposed by D. M. Bătinețu – Giurgiu; Neculai Stanciu – Romania

XII.17. If 0 < 푎 < 푏 then:

2휋

ln푏푎푏 − 푎 <

휋2

푑푥arctan 푥

<휋2

ln푏푎

+ 푏 − 푎

Proposed by Daniel Sitaru -Romania

XII.18. If [푎, 푏] ⊂ 0, then:

sin 푥 푑푥 > 푏 + 1 − 푎 + 1

Proposed by Daniel Sitaru -Romania

XII.19. If 푓,푔: [푎, 푏] → (0,∞) integrable, such that 푓(푥) + 푔(푥) ≤ 8 then:

푓(푥) 푔(푥) + 푔(푥) 푓(푥)

푓(푥) − 푓(푥)푔(푥) + 푔(푥)푑푥 ≤ 4(푏 − 푎)

Proposed by Daniel Sitaru -Romania

XII.20. If 푓: [0,1] → (0,∞), 푓 derivable, 푓 ʹ continuous, 푓 ʹ(푥) = 푓 ʹ(1− 푥),∀푥 ∈ [0,1] then:

푓(푥)푑푥 ≥ 푓(0) ⋅ 푓(1)

Proposed by Daniel Sitaru -Romania

XII.21. If 0 ≤ 푥 ≤ 푎, 0 ≤ 푦 ≤ 푏, 푐 > 0:

Page 73: ROMANIAN MATHEMATICAL MAGAZINE R.M.M. · pentru ∈ z∗, pe care am rezolvat-o așa cum urmează, am păstrat algoritmul și pentru calculul altor sume, unele putând fi calculate

S.S.M.ROMÂNIA - Filiala Mehedinți 2017

73 ROMANIAN MATHEMATICAL MAGAZINE NR. 18

Ω = 푥 + 푦 − 2푎푥 + 푎 푑푥 푑푦,Ω = 푥 + 푦 − 2푏푦 + 푏 푑푥 푑푦

then Ω + Ω ≤ (푎 + 푏)푐

Proposed by Daniel Sitaru -Romania

XII.22. If 0 ≤ 푥 ≤ 3, 0 ≤ 푦 ≤ 4,푎 > 0

훺 = 푥 + 푦 − 6푥 + 9푑푥 푑푦,훺 = 푥 + 푦 − 8푦 + 16푑푥 푑푦

then: 훺 + 훺 > 5푎

Proposed by Daniel Sitaru -Romania

XII.23. Calculate:

(푥 + 34푦 − 10푥푦 − 6푦 + 2) 푑푥푑푦 ≥ 1

Proposed by Sameer Shihab-Riyadh-Saudi Arabia

XII.24. If 푛 ∈ ℕ,푛 ≥ 2,푛 − fixed, 푓: [1,푛] → (0,∞), 푓 − integrable, 푖 ∈ 1,푛 − 1

훺 = 푓(푥) 푑푥, 0 ≤ 훼 ≤ min 푓(푥)푑푥 ≤ max 푓(푥)푑푥 ≤ 훽

then:

(푛 − 1)훼훽 + 푓(푥) 푑푥 ≤ (훼 + 훽)훺

Proposed by Daniel Sitaru -Romania

XII.25. If 푓: [푎, 푏] → (0,∞),푎 < 푏, continuous,

푚 = min푓(푥) ,푀 = max 푓(푥) ,푛 ∈ ℕ∗ then:

푚푀

( )

(푏 − 푎) ≤ 푓 (푥) 푑푥1

푓 (푥)푑푥 ≤푀푚

( )

(푏 − 푎)

Proposed by Daniel Sitaru -Romania

XII.26. If 푒 ≤ 푎 ≤ 푐 ≤ 푏 ≤ 푒 then:

Page 74: ROMANIAN MATHEMATICAL MAGAZINE R.M.M. · pentru ∈ z∗, pe care am rezolvat-o așa cum urmează, am păstrat algoritmul și pentru calculul altor sume, unele putând fi calculate

S.S.M.ROMÂNIA - Filiala Mehedinți 2017

74 ROMANIAN MATHEMATICAL MAGAZINE NR. 18

(푏 − 푎)푥

log푥푑푥 ≤ (푐 − 푎)

푥log 푥

푑푥

Proposed by Daniel Sitaru -Romania

XII.27. If 0 < 푎 < 푏 < 1 then:

1푏 − 푎

1 +1

sin 푥1 +

1cos 푥

푑푥 ≥ 1 +4휋

Proposed by Daniel Sitaru -Romania

XII.28. If 푎, 푏, 푐 ∈ (2,∞),훺(푎) = ∫ 푑푥 then:

2푏푐훺(푎) + 2푐훺(푏) + 2푎푏훺(푐) < 푎 + 푏 + 푐

Proposed by Daniel Sitaru -Romania

XII.29. Calculate:

log (1 + 푒 ) 푑푥 < log(1 + 푒 )푑푥 +1

12

Proposed by Daniel Sitaru -Romania

XII.30. If 푎, 푏, 푐 ∈ (0,∞) then:

1 +푏푎

푒 푑푥 + 1 +푎푏

푒 푑푥 ≥ 푒 + 푒 푑푥

Proposed by Daniel Sitaru -Romania

XII.31. Let 푓: [0,1] → (−1,1) be a continuous function so that: ∫ 푓(푥) 푑푥 ∉ {−1,1}.Prove that:

푒∫ ( )

1 + 푒∫ ( )≤

푒 ( )

1 + 푒 ( ) 푑푥

Proposed by Soumitra Mandal – Chandar Nagore – India

XII.32. If 푎, 푏 ∈ ℝ then: 2(푏 − 푎) √푒 ≤ 8 √푒 − √푒 ≤ 푒 + 푒 (푏 − 푎) .

Proposed by Daniel Sitaru -Romania

XII.33. If 푎 ∈ ℝ, 푓: [푎,푎 + 2] → ℝ, 푓 ∈ 퐶 ([푎,푎 + 2]), 6 ≤ 푓 (푥) ≤ 12 then:

1 + 푓(푎 + 1) ≤12

푓(푥) 푑푥 ≤ 2 + 푓(푎 + 1)

Proposed by Daniel Sitaru -Romania

Page 75: ROMANIAN MATHEMATICAL MAGAZINE R.M.M. · pentru ∈ z∗, pe care am rezolvat-o așa cum urmează, am păstrat algoritmul și pentru calculul altor sume, unele putând fi calculate

S.S.M.ROMÂNIA - Filiala Mehedinți 2017

75 ROMANIAN MATHEMATICAL MAGAZINE NR. 18

XII.34. If 0 < 푎 < 푏 < then:

7 푥 tan 푥 sin푥2

+ tan푥2 √sin 푥 tan 푥 푑푥 > 푏 − 푎

Proposed by Daniel Sitaru -Romania XII.35. Solve the next system of equations in the set of the complex numbers

푥 + 푦 + 푧 = 푥푦푧푥 + 푦 + 푧 = 푥푦 + 푥푧 + 푦푧

푥 + 푦 + 푧 = 18

Proposed by Kovács Béla – Romania XII.36. Să se calculeze:

4푥 − 5푥 + 10푥 + 4푥 + 25

푑푥;푥 ∈ (0,∞)

Proposed by Daniel Sitaru – Romania XII.37. Să se calculeze:

퐼 =2 tg 푥1 + 2

+3 tg 푥1 + 3

푑푥

Proposed by Daniel Sitaru – Romania XII.38. Să se arate că:

1 + 푥 sin 푥 푑푥 >5휋288

Proposed by Daniel Sitaru – Romania XII.39. Să se arate că dacă [푎, 푏] ⊂ 0, atunci:

sin 푥 푑푥 > 푏 + 1 − 푎 + 1

Proposed by Daniel Sitaru – Romania XII.40. Fie 0 < 훼 ≤ ; 푟 = 1

Să se calculeze:

lim→

푆[퐴 퐵 퐴 ]

Proposed by Daniel Sitaru – Romania XII.41. Să se arate că:

Page 76: ROMANIAN MATHEMATICAL MAGAZINE R.M.M. · pentru ∈ z∗, pe care am rezolvat-o așa cum urmează, am păstrat algoritmul și pentru calculul altor sume, unele putând fi calculate

S.S.M.ROMÂNIA - Filiala Mehedinți 2017

76 ROMANIAN MATHEMATICAL MAGAZINE NR. 18

sin(sin 푥)푑푥 ≤휋3

+ 1 −√3

Proposed by Daniel Sitaru – Romania XII.42. Să se calculeze:

lim→

∫ 푡 (푡 + 푡 ln 푡 + 1)푑푡푥

Proposed by Daniel Sitaru – Romania XII.43. Să se calculeze:

√푥 + √푥 − 2 + √푥 − √푥 − 2

√푥 + √2푑푥

Proposed by Daniel Sitaru – Romania XII.44. Să se calculeze:

L = lim→∞

n12

sin 푥 dx −1

nsin

in

sinjn

Proposed by Daniel Sitaru – Romania XII.45. Să se arate că:

√푥|sin 푥 − 푥| <1

27

Proposed by Daniel Sitaru – Romania XII.46. Să se arate că:

√푥 sin 푥 푑푥 >49

135

Proposed by Daniel Sitaru – Romania XII.47. Să se arate că:

휋24

<푥

sin 푥푑푥 <

휋12

Proposed by Daniel Sitaru – Romania XII.35. Să se calculeze:

퐼 =푥 sin푥

√1 + cos 푥푑푥

Proposed by Daniel Sitaru – Romania XII.48. Să se demonstreze că:

(tg(sin 푥) + 2 sin(sin 푥))푑푥 >3 √3 − 1

2

Proposed by Daniel Sitaru – Romania XII.49. Fie

Page 77: ROMANIAN MATHEMATICAL MAGAZINE R.M.M. · pentru ∈ z∗, pe care am rezolvat-o așa cum urmează, am păstrat algoritmul și pentru calculul altor sume, unele putând fi calculate

S.S.M.ROMÂNIA - Filiala Mehedinți 2017

77 ROMANIAN MATHEMATICAL MAGAZINE NR. 18

퐼 =√푥 − 푘 1 + √푘 + 1 − 푥

√푥 − 푘 + 2 (푘 + 1− 푥)(푥 − 푘) + √푘 + 1− 푥푑푥

Să se calculeze:

퐿 = lim→

Proposed by Daniel Sitaru – Romania XII.50. Să se calculeze:

1(푥 + 2) + (푥 + 4) + 14

푑푥; 푥 ∈ ℝ Proposed by Daniel Sitaru – Romania

XII.51. Să se demonstreze că:

1ln(1 − 푥)푑푥 < 1− ln

32

Proposed by Daniel Sitaru – Romania

XII.52. Să se arate că:

푒 푑푥 < ln3√5

5

Proposed by Daniel Sitaru – Romania XII.53. Să se calculeze:

ln(1 + 푥 )− 2푥 arctg 푥(1 + 푥 ) ln (1 + 푥 )

푑푥

Proposed by Daniel Sitaru – Romania XII.54. Să se arate că dacă 0 < 푎 < 푏 atunci:

2 √푏 + 1− √푎 + 1푏 − 푎

< ln푏 + √푏 + 1푎 + √푎 + 1

Proposed by Daniel Sitaru – Romania XII.55. Let 훼 be a real number with 훼 > 2 and let 푎 and 푏 be real numbers with 0 < 푎 ≤ 푏.

Prove that + ≤ (푏 − 푎)(푏 − 푎 ).

Proposed by Leonard Giugiuc-Romania XII.56. Let 푛 be an integer, 푛 ≥ 2 and let 푎 ,푎 , … ,푎 be real numbers situated in the interval 2 + √3,∞ . Prove the inequality:

(푎 − 푎 + 1)(푎 − 푎 + 1) … (푎 − 푎 + 1) ≥ 3 2 + √3푎 푎 …푎

2 + √3−

푎 푎 …푎

2 + √3+ 1 .

Proposed by Leonard Giugiuc – Romania; Michael Rozenberg – Israel

Page 78: ROMANIAN MATHEMATICAL MAGAZINE R.M.M. · pentru ∈ z∗, pe care am rezolvat-o așa cum urmează, am păstrat algoritmul și pentru calculul altor sume, unele putând fi calculate

S.S.M.ROMÂNIA - Filiala Mehedinți 2017

78 ROMANIAN MATHEMATICAL MAGAZINE NR. 18

XII.57. Find : 퐼 = ∫ 푑푥 . Proposed by Shivam Sharma-New Delhi-India

AUTORII MATERIALELOR DIN RMM-18

Nr.crt. Numele și prenumele Nr.crt. Numele și prenumele 1 DANIEL SITARU 29 ȘTEFAN MARICA 2 GHEORGHE CĂINICEANU 30 ANGELICA BOITAN 3 CLAUDIA NĂNUȚI 31 NICOLAE PAPACU 4 DIANA TRĂILESCU 32 DANIEL MACKLEEN 5 MIHAI OCTAVIAN UNGUREANU 33 LAVINIU BEJENARU 6 DAN NĂNUȚI 34 MARIN CHIRCIU 7 LEONARD GIUGIUC 35 IOAN ȘERDEAN 8 EMILIA RĂDUCAN 36 NGUYEN ANH DUC 9 OVIDIU TICUȘI 37 PETRE STÂNGESCU

10 MANUELA PRAJEA 38 GEORGE APOSTOLOPOULOS 11 ELENA RÎMNICEANU 39 ROVSEN PIRGULIEV 12 DAN NEDEIANU 40 GHEORGHE ALEXE 13 DANIEL STRETCU 41 GEORGE - FLORIN ȘERBAN 14 MIHALY BENCZE 42 CARMEN COADĂ 15 D.M.BĂTINEȚU-GIURGIU 43 RICHDAD PHUC 16 NECULAI STANCIU 44 NGUYEN VIET HUNG 17 ABDALLAH EL FARISSI 45 DANA HEUBERGER 18 GABRIELA BONDOC 46 LE VIET HUNG 19 IULIANA TRAȘCĂ 47 NGUYEN PHUC TANG 20 DOREL CĂPRARU 48 ANDREI ȘTEFAN MIHALCEA 21 NISHANT KUMAR 49 CONSTANTIN GIUGIUC 22 LĂDARU DANIELA 50 VASILE JIGLĂU 23 CARMEN-VICTORIȚA CHIRFOT 51 MARTIN LUKAREVSKI 24 MARIA UNGUREANU 52 MEHMET ȘAHIN 25 MARIAN DINCĂ 53 SOUMITRA MANDAL 26 SAMEER SHIHAB 54 BELA KOVACS 27 ARDAK MIRZAKHMEDOV 55 SHIVAM SHARMA 28 SLADJAN STANKOVIK 56 MICHAEL ROZENBERG

NOTĂ: Website-ul SSMR-Filiala Mehedinți are adresa http//:www.ssmrmh.ro Pentru a publica probleme propuse, articole și note matematice în RMM puteți trimite materialele pe mailul: [email protected]